You are on page 1of 124

‫جتميعية فروض و إختبارات‬

‫الفصل األول‬
‫للسنوات األوىل‬
‫جذع مشرتك علوم و تكنولوجيا‬
‫مأخوذة من خمتلف ثانويات الوطن‬
‫مجعها األستاذ ‪ :‬قرايدية مسري‬
‫ثانوية سعدي الصديق ‪ -‬تبسة ‪-‬‬
‫السّنة الدّراسيّة‪:‬‬ ‫ثانوية سعدي الصديق‬

‫املدّة‪ :‬ساعتان‬ ‫املستوى‪ :‬أوىل جذع مشرتك علوم و تكنولوجيا‬


‫أجِب على التّمرين الثالث إجباريّاً و تمرينين إختياريّاً‬
‫التّمرين األوّل‪ 60( :‬نقاط)‬
‫اخرت اإلجابة الصّحيحة مع التّعليل‬
‫‪ A‬هي‬ ‫‪2‬‬ ‫‪1‬‬ ‫‪2‬‬ ‫‪2 3‬‬ ‫(‬ ‫‪2‬‬ ‫‪ )1‬كتابة العدد ‪2)2‬‬

‫‪1‬‬ ‫ج) ‪2 3‬‬ ‫‪2 2‬‬ ‫ب) ‪1‬‬ ‫‪2 3‬‬ ‫أ) ‪1‬‬
‫‪ J‬فإنّ‬ ‫‪ I‬و ‪3, 3‬‬ ‫‪2,‬‬ ‫‪ )2‬إذا كان‬
‫‪I‬‬ ‫‪J‬‬ ‫‪2, 3‬‬ ‫ج)‬ ‫‪I‬‬ ‫‪J‬‬ ‫ب) ‪2, 3‬‬ ‫‪I‬‬ ‫‪J‬‬ ‫‪3,2‬‬ ‫أ)‬
‫‪ )3‬إذا كان ‪ N 0, 0003478‬و ‪ M 1780 105‬فإنّ رتبة مقدار ‪ M N‬هي‬
‫ج) ‪6 104‬‬ ‫ب) ‪6 103‬‬ ‫أ) ‪3 104‬‬
‫‪ )4‬إذا كان ‪ x‬عدداً حقيقيّاً حيثُ ‪ x 1‬فإنّ‬
‫‪2‬‬ ‫‪2‬‬
‫) ‪(1 3x‬‬ ‫ج) ‪4‬‬ ‫) ‪(1 3x‬‬ ‫ب) ‪4‬‬ ‫أ) ‪(1 3x )2 4‬‬
‫‪ )5‬يُمكن التّعبري عن الكتابة ‪ x 4 5‬بـ‬
‫‪4‬‬ ‫‪x‬‬ ‫ج) ‪5‬‬ ‫‪9‬‬ ‫‪x‬‬ ‫ب) ‪1‬‬ ‫‪4‬‬ ‫‪x‬‬ ‫‪5‬‬ ‫أ)‬
‫التّمرين الثّاني‪ 60( :‬نقاط)‬
‫ليكن ) ( مُستقيماً مُزوّداً مبعلم ) ‪. (O, I‬‬
‫‪ )1‬علِّم النّقطتني ‪ A‬و ‪ B‬ذواتي الفاصلتني ‪ 3‬و ‪ 5‬على التّوالي‪ ،‬و النّقطة ‪ G‬منتصف القطعة ‪. AB‬‬
‫لتكن ‪ M‬نقطة متحرّكة على املستقيم ) ( فاصلتها ‪. x‬‬
‫‪ )2‬عيِّن قيمة العدد احلقيقي ‪ x‬يف احلاالت اآلتية‬
‫‪x 5‬‬ ‫ج) ‪x 3‬‬ ‫‪x 5‬‬ ‫ب) ‪x 3 8‬‬ ‫أ) ‪x 3 4‬‬

‫التّمرين الثّالث‪ 68( :‬نقاط)‬


‫بتمثيليهمـــا البيـــانيّني ) ‪ (C‬و ) ‪(C‬‬
‫‪g‬‬ ‫‪f‬‬
‫لـــتكن ‪ f‬و ‪ g‬دالّـــتني معـ ـرّفتني علـــى ‪1, 4‬‬
‫على التّوالي و املوضّحني يف الشكل املرفق‪ .‬بقراءة بيانيّة‪:‬‬
‫‪ )1‬عيِّن )‪ g( 1) ، g(3) ، g(1) ، g(0) ، f (3) ، f (1) ، f (0‬و )‪. g(2‬‬
‫‪ )2‬عيٍّن اجتاه تغيُّر الدّالة ‪ g‬على اجملال ‪ُ 1, 3‬ث ّم شكل جدول تغيُّراتها‪.‬‬
‫‪ )3‬حدِّد القيمتني العظمى و الصغرى للدّالة ‪ g‬على اجملال ‪. 1, 3‬‬
‫‪ )4‬شكل جدول إشارة الدّالة ‪ g‬على اجملال ‪. 1, 3‬‬
‫‪ )5‬حدِّد وضعيّة ) ‪ (C f‬بالنّسبة إىل ) ‪ (C g‬على اجملال ‪. 1, 3‬‬
‫‪ )6‬استنتج حلول املرتاجحة ) ‪. g(x ) f (x‬‬
‫‪ )7‬ما هو عدد حلول املعادلة ‪ g(x ) 0‬؟‬
‫التّمرين الرّابع‪ 60( :‬نقاط)‬
‫أنبوب اختبار علـى شـكل أسـطوانة دورانيّـة نصـف قطرهـا ‪ 1 r 1,2‬و ارتفاعهـا ‪ ، 9,6 h 9, 8‬مملـوء إىل اايـة ثةثـة‬
‫أرباع حجمه بالزيت‪.‬‬
‫‪ )1‬أحسُب ‪ t‬ارتفاع الزّيت يف األنبوب‪.‬‬
‫‪ )2‬أضفنا املاء يف هذا األنبوب فازداد ارتفاع املزيج بـ ‪ . 1,6 l 1, 8‬ما هو حجم املزيج النّاتج؟‬
‫‪ ،V‬الوحدة هي السّنتيمرت‪.‬‬ ‫‪ ، 3,14‬حجم األسطوانة الدّورانيّة هو ‪hr 2‬‬ ‫مةحظة‪ :‬نأخذ ‪3,15‬‬
‫التّمرين األوّل‪:‬‬
‫الكتاب ـ ـ ـ ـ ــة العلميّ ـ ـ ـ ـ ــة ل ـ ـ ـ ـ ــ ‪ N 0, 0003478‬هـــــــــــي‬ ‫‪ )1‬اإلختيار (أ) ألنّ‬
‫‪ N 3, 478 10 4‬و الكتابـ ـ ـ ـ ــة العلميّـــــــ ــة للعــــــــــدد‬
‫‪A‬‬ ‫‪2‬‬ ‫‪1‬‬ ‫‪2‬‬ ‫‪2 3‬‬ ‫(‬ ‫‪2‬‬ ‫‪2)2‬‬
‫‪ M 1780 105‬هـي ‪ ، M 1,780 108‬إذن رتبـة‬
‫‪2‬‬ ‫‪1‬‬ ‫‪2 3‬‬ ‫‪2‬‬ ‫‪2‬‬ ‫‪2‬‬ ‫‪2 3‬‬ ‫‪1‬‬
‫مقـــ ــدار ‪ N‬هـ ـ ــي ‪ 3 10 4‬و رتب ـ ـــة مقــــــدار ‪ M‬هــــــي‬
‫‪ 2 108‬و منـ ـ ـ ـ ـ ــه رتبـ ـ ـ ـ ـ ــة مقـ ـ ـ ـ ـ ــدار ‪ M N‬هــــــــــــي‬ ‫‪ )2‬اإلختيار (ب) ألنّ‬
‫‪ 2 108 3 10 4‬أي ‪. 6 104‬‬ ‫‪x‬‬ ‫‪J‬‬ ‫‪ x‬و ‪3, 3‬‬ ‫‪ x‬معناه ‪2‬‬ ‫‪I‬‬ ‫‪2,‬‬
‫‪ )4‬اإلختيار (أ) ألنّ‬ ‫‪ x‬معنـــــــــاه‬ ‫‪I‬‬ ‫‪ ، 3‬و منـــــــــه ‪J‬‬ ‫‪x‬‬ ‫معنـــ ـــــاه ‪3‬‬
‫‪ 1 3x‬يكافئ ‪. (1 3x )2 4‬‬ ‫‪ x 1‬يكافئ ‪2‬‬ ‫‪3‬‬ ‫‪x‬‬ ‫‪3‬‬
‫‪ )5‬اإلختيار (ب) ألنّ‬ ‫‪.x‬‬ ‫أي ‪2, 3‬‬ ‫‪2‬‬ ‫‪x‬‬ ‫‪ et‬أي ‪3‬‬
‫بتطبيق خاصيّة اإلنتقال من القيمة املطلقة إىل احلصـر‬ ‫‪x‬‬ ‫‪2‬‬
‫( ‪ x c r‬تك ـ ـ ــافئ ‪ ) c r x c r‬جنـــــــد أنّ‬
‫‪.‬‬ ‫‪9‬‬ ‫‪x‬‬ ‫‪ x‬يك ـ ـ ـ ـ ـ ـ ـ ـ ـ ـ ـ ـ ـ ـ ـ ـ ـ ـ ـ ـ ـ ـ ـ ـ ـ ـ ـ ـ ـــافئ ‪1‬‬ ‫‪4‬‬ ‫‪5‬‬ ‫‪ )3‬اإلختيار (ج) ألنّ‬

‫التّمرين الثّاني‪:‬‬
‫‪ x‬هــي املســافة بــني النّقطــتني ‪ B‬و ‪M‬‬ ‫ب) الكتابــة ‪5‬‬ ‫‪ )1‬التّعليم يف الرّسم املرفق‪.‬‬
‫أي ‪ x 5 x 3 8‬تكـ ـ ـ ـــــــــــــــــــــــــــــــــــــــــــــــــــافئ‬
‫‪ ، MA MB 8‬و من ــه ‪ M‬تكـــون داخـــل القطعـــة‬
‫‪.x‬‬ ‫‪ AB‬أي ‪5, 3‬‬ ‫‪ x‬هــي املســافة بــني النّقطــتني ‪ A‬و ‪M‬‬ ‫‪ )2‬أ) الكتابــة ‪3‬‬

‫‪ MB‬أي أنّ‬ ‫‪MA‬‬ ‫‪ x‬تعـ‬ ‫‪5‬‬ ‫‪x‬‬ ‫ج) الكتابة ‪3‬‬


‫‪ x‬أو‬ ‫‪ MA‬إذن ‪1‬‬ ‫‪4‬‬ ‫‪ x‬تكــــــــــافئ‬ ‫‪3‬‬ ‫أي ‪4‬‬
‫‪.x‬‬ ‫‪7‬‬
‫‪.x‬‬ ‫‪ M‬تكون على يسار ‪ G‬أي ‪, 1‬‬

‫التّمرين الثّالث‬
‫‪ )1‬الصور موضحة يف اجلدول‬
‫‪x‬‬ ‫‪0 1 3‬‬ ‫‪x‬‬ ‫‪0 1 3‬‬ ‫‪1 2‬‬
‫‪f (x ) 0‬‬ ‫‪2 0 g(x ) 0‬‬ ‫‪2 0‬‬ ‫‪4‬‬ ‫‪4‬‬
‫القيم ــة احلديّ ــة الص ــغرى للدال ــة ‪ g‬علـــى ‪ 1, 3‬هـــي‬ ‫‪ )2‬الدّالـ ـ ـــة ‪ g‬متزايـــــــدة لامـــــ ـاً علـــــــى ‪ 1, 0 2, 3‬و‬
‫‪ x‬أو ‪. x 2‬‬ ‫‪ 4‬من أجل ‪1‬‬ ‫متناقصة لاماً على ‪ ، 0,2‬و جدول تغيّراتها هو‬
‫‪ )4‬مبــا أنّ ) ‪(C g‬يقــع ــت اــور الفواصــل فــإنّ الدّالــة ‪g‬‬
‫موضــ ــحة يف‬ ‫ســـــالبة و إش ـ ــارتها عل ـ ــى اجمل ـ ــال ‪1, 3‬‬
‫اجلدول‬
‫هــي ‪0‬‬ ‫‪ )3‬القيمــة احلديّــة الكــاى للدالــة ‪ g‬علــى ‪1, 3‬‬
‫‪.x‬‬ ‫‪ x‬أو ‪3‬‬ ‫من أجل ‪0‬‬

‫‪1‬‬
‫) ‪(C‬‬ ‫‪f‬‬
‫) ‪ g(x‬هي فواصـل نقـ‬ ‫‪ )6‬حلول املرتاجحة ) ‪f (x‬‬ ‫‪ )5‬التّمثيــــــــــل ) ‪ (C f‬يقــــــــــع فــــــــــو ) ‪ (C g‬مــــــــــن أجــــــــــل‬
‫‪.x‬‬ ‫‪1, 0‬‬ ‫اليت تقع فو ) ‪ (C g‬أي ‪1, 3‬‬ ‫‪.x‬‬ ‫‪1, 0‬‬ ‫‪1, 3‬‬

‫) ‪ g(x‬هـ ــي فواصــــل نقـــ تقــــا ع‬ ‫‪ )7‬حلــــول املعادلـ ــة ‪0‬‬ ‫‪.x‬‬ ‫ت ) ‪ (C g‬من أجل ‪0,1‬‬ ‫التّمثيل ) ‪ (C f‬يقع‬
‫‪.x‬‬ ‫‪ x‬أو ‪3‬‬ ‫) ‪ (C g‬مع اور الفواصل أي ‪0‬‬ ‫‪،x‬‬ ‫‪1 ،x‬‬ ‫التمثيــل ) ‪ (C f‬يقطــع ) ‪ (C g‬مــن أجــل ‪0‬‬
‫‪.x‬‬ ‫‪3‬‬

‫التّمرين الرّابع‬
‫‪ )2‬بعـــــ ـ ـ ــد زيـ ـ ـ ـ ـ ــادة املـ ـ ـ ـ ـ ــاء يُصـ ـ ـ ـ ـ ــب ارتفـــــــــ ــاع املــــــــــــزيج‬ ‫‪ )1‬مب ـــا أنّ األنبــــوب مملــــوء إىل اايــــة ثةثــــة أرباعــــه فــــإنّ‬
‫‪ 7,20 1,6 t l 7, 35 1, 8‬و منـــــــــــــــــــــــــــــــــه‬ ‫‪3‬‬
‫‪ ، 8, 80 t l 9,15‬و عليــه يكــون حجــم املــزيج هــو‬ ‫‪ 9,6‬فــــــــــــــــــــــإنّ‬ ‫‪h‬‬ ‫‪ ، t‬و مبــــــــــــــــــــــا أنّ ‪9, 8‬‬ ‫‪h‬‬
‫‪4‬‬
‫‪ V‬ومنـ ـ ـ ـ ـ ـ ـ ـ ـ ـ ـ ـ ـ ـ ـ ـ ـ ـ ـــــــــــــــــــــــــــــــــــــــــــــــــــــــــــه‬ ‫‪(t l )r 2‬‬ ‫‪3‬‬ ‫‪3‬‬
‫‪. 7,20‬‬ ‫‪t‬‬ ‫أي أنّ ‪7, 35‬‬ ‫‪9, 6‬‬ ‫‪t‬‬ ‫‪9, 8‬‬
‫‪ 3,14 8, 80 12 V 3,15 9,15 1,22‬أي‬ ‫‪4‬‬ ‫‪4‬‬
‫أنّ ‪( . 27,63 V 41,50‬الوحـ ـ ــدة هـــ ــي السّــ ـــنتيمرت‬ ‫(الوحدة هي السّنتيمرت)‬
‫املكعّب)‬

‫‪2‬‬
‫السّنة الدّراسيّة‪2016/2015 :‬‬ ‫ثانويّة شريّط لزهر – احلمّامات‬

‫الفرض األول للثالثي األول يف مادّة الرياضيات‬


‫املستوى‪ :‬أوىل جذع مشرتك علوم وتكنولوجيا‬
‫‪2‬‬ ‫‪1‬‬

‫‪5‬‬
‫أمتِم اجلدول اآلتي (عيّن طبيعة األعداد املوضّحة يف اجلدول)‬

‫‪2‬‬
‫‪5‬‬ ‫‪3‬‬ ‫‪7 3‬‬ ‫‪3‬‬ ‫‪2‬‬
‫‪1‬‬ ‫‪5 72‬‬
‫‪8‬‬
‫‪2‬‬ ‫‪4‬‬
‫(‬ ‫‪3‬‬ ‫‪2)2‬‬ ‫‪2 6‬‬

‫‪22‬‬ ‫‪2 2‬‬


‫‪11‬‬ ‫‪42‬‬ ‫‪3‬‬
‫‪6‬‬
‫أمتِم اجلدول اآلتي‬
‫‪ I‬هو جمموعة األعداد‬
‫اجملال ‪I‬‬ ‫اجملال ‪J‬‬ ‫‪I‬‬ ‫‪J‬‬ ‫‪I‬‬ ‫‪J‬‬ ‫احلقيقيّة ‪ x‬حيث‬
‫‪2,1‬‬ ‫‪0, 3‬‬

‫‪2, 7‬‬ ‫‪5‬‬ ‫‪x‬‬ ‫‪1‬‬


‫‪,0‬‬ ‫‪0,‬‬

‫‪2,‬‬ ‫‪x‬‬ ‫‪4‬‬

‫‪5‬‬
‫‪.b‬‬ ‫‪ a‬و ‪0, 0043‬‬ ‫‪ )1‬ليكن ‪ a‬و ‪ b‬عددين حقيقيّني حيث ‪715,24‬‬

‫أ) الكتابة العلميّة للعدد ‪ a‬هي ‪ .....................................‬ورتبة مقداره هي ‪...................................‬‬

‫ب) الكتابة العلميّة للعدد ‪ b‬هي ‪ ......................................‬ورتبة مقداره هي ‪...................................‬‬

‫ج) رتبة مقدار العدد ‪ a b‬هي ‪....................................‬‬

‫اقلب الصفحة‬ ‫مُالحظة‪ :‬ال تؤخذ يف احلسبان اإلجابات النّاقصة وغري الواضحة‬
‫‪http://imadmaths.blogspot.com/‬‬
‫‪ )2‬أمتم اجلدول اآلتي‬
‫حتليل العددين ‪ a‬و ‪ b‬إىل جداء عوامل أوّليّة‬
‫‪a‬‬ ‫‪2016‬‬ ‫‪..............................‬‬

‫‪b‬‬ ‫‪2408‬‬ ‫‪..............................‬‬

‫)‪PGCD(a,b‬‬ ‫‪..............................‬‬

‫)‪PPCM (a,b‬‬ ‫‪..............................‬‬

‫الطريقة‬

‫‪ Q‬يُكتب على شكل‬ ‫‪14, 717171...‬‬ ‫العدد‬

‫‪Q‬‬ ‫كسر بـ‬

‫‪4‬‬
‫‪5, 3‬‬ ‫‪a‬‬ ‫‪ 1, 8‬مع مســتطيل طول ‪5, 4‬‬ ‫الشــّكل املرفق يُمثّل احتاد نصــد داةر نصــد قطرها ‪r 1, 9‬‬
‫)‬ ‫وعرض ‪ . 2, 4 b 2,7‬عيّن حصراً ملساحة الشكل‪( .‬نأخذ ‪3,14‬‬

‫‪....................................................................................................................................................................................................‬‬

‫‪........................................................................................................................................ ............................................................‬‬

‫‪....................................................................................................................................................................................................‬‬

‫‪....................................................................................................................................................................................................‬‬

‫‪...................................................................................................................................................................................................‬‬
‫اقلب الصفحة‬ ‫مُالحظة‪ :‬ال تؤخذ يف احلسبان اإلجابات النّاقصة وغري الواضحة‬
‫‪http://imadmaths.blogspot.com/‬‬
‫التصحيح النموذجي‬
‫‪5‬‬
‫أمتِم اجلدول اآلتي (عيّن طبيعة األعداد املوضّحة يف اجلدول)‬

‫‪2‬‬
‫‪5‬‬ ‫‪3‬‬ ‫‪7 3‬‬ ‫‪3‬‬ ‫‪2‬‬ ‫‪‬‬
‫‪1‬‬ ‫‪5 72‬‬
‫‪8‬‬ ‫‪‬‬
‫‪2‬‬ ‫‪4‬‬
‫(‬ ‫‪3‬‬ ‫‪2)2‬‬ ‫‪2 6‬‬ ‫‪‬‬

‫‪22‬‬ ‫‪2 2‬‬ ‫‪‬‬

‫‪11‬‬ ‫‪42‬‬ ‫‪3‬‬ ‫‪‬‬

‫‪6‬‬
‫أمتِم اجلدول اآلتي‬
‫‪ I‬هو جمموعة‬
‫اجملال ‪I‬‬ ‫اجملال ‪J‬‬ ‫‪I‬‬ ‫‪J‬‬ ‫‪I‬‬ ‫‪J‬‬ ‫األعداد‬
‫احلقيقيّة ‪ x‬حيث‬
‫‪2,1‬‬ ‫‪0, 3‬‬ ‫‪0,1‬‬ ‫‪2, 3‬‬ ‫‪2‬‬ ‫‪x‬‬ ‫‪1‬‬

‫‪5,1‬‬ ‫‪2, 7‬‬ ‫‪5,1‬‬ ‫‪2, 7‬‬ ‫‪5‬‬ ‫‪x‬‬ ‫‪1‬‬


‫‪,0‬‬ ‫‪0,‬‬ ‫}‪{0‬‬ ‫‪,‬‬ ‫‪x‬‬ ‫‪0‬‬
‫‪, 4‬‬ ‫‪2,‬‬ ‫‪, 4‬‬ ‫‪2,‬‬ ‫‪x‬‬ ‫‪4‬‬

‫‪5‬‬
‫‪.b‬‬ ‫‪ a‬و ‪0, 0043‬‬ ‫‪ )1‬ليكن ‪ a‬و ‪ b‬عددين حقيقيّني حيث ‪715,24‬‬

‫أ) الكتابة العلميّة للعدد ‪ a‬هي ‪ 7,1524 102‬ورتبة مقداره هي ‪. 7 102‬‬

‫ب) الكتابة العلميّة للعدد ‪ b‬هي ‪ 4, 3 10 3‬ورتبة مقداره هي ‪. 4 10 3‬‬

‫‪ 7 102 4 10‬أي ‪. 3‬‬ ‫‪3‬‬


‫ج) رتبة مقدار العدد ‪ a b‬هي‬

‫‪http://imadmaths.blogspot.com/‬‬
‫‪ )2‬أمتم اجلدول اآلتي‬
‫حتليل العددين ‪ a‬و ‪ b‬إىل جداء عوامل أوّليّة‬

‫‪2408‬‬ ‫‪2016‬‬
‫‪a‬‬ ‫‪2016‬‬ ‫‪25‬‬ ‫‪32‬‬ ‫‪7‬‬
‫‪2‬‬ ‫‪2‬‬
‫‪1204‬‬ ‫‪2‬‬ ‫‪1008‬‬ ‫‪2‬‬
‫‪602‬‬ ‫‪504‬‬
‫‪b‬‬ ‫‪2408‬‬ ‫‪23‬‬ ‫‪7 43‬‬
‫‪2‬‬ ‫‪2‬‬
‫‪301‬‬ ‫‪7‬‬ ‫‪252‬‬ ‫‪2‬‬
‫)‪PGCD(a,b‬‬ ‫‪23‬‬ ‫‪7‬‬ ‫‪56‬‬
‫‪43‬‬ ‫‪43‬‬ ‫‪126‬‬ ‫‪2‬‬
‫‪1‬‬ ‫‪63‬‬ ‫‪3‬‬
‫‪21‬‬ ‫‪3‬‬
‫)‪PPCM (a,b‬‬ ‫‪25‬‬ ‫‪32‬‬ ‫‪7‬‬ ‫‪43‬‬ ‫‪86688‬‬
‫‪7‬‬ ‫‪7‬‬
‫‪1‬‬
‫الطريقة‬
‫لدينا ‪.Q 14,717171... 14 0,717171...‬‬
‫جنــــــــــد أنّ‬ ‫بــــــــــوضـــــــــــــــع ‪x 0,717171...‬‬
‫ومــــــــــــــــــــنــــــــــــــــــ ـ‬ ‫‪ Q‬يُكتب على شكل كسر بـ ‪100x 71,717171...‬‬ ‫‪14, 717171...‬‬ ‫العدد‬
‫‪ 100x 71 0,717171...‬ومـــنـ ـ ‪100x 71 x‬‬
‫‪71‬‬ ‫‪1457‬‬
‫‪ x‬ومــــــنـــــ‬ ‫‪ 99x‬ومــــــنـــــ‬ ‫‪71‬‬ ‫ومــــــنـــــ‬ ‫‪Q‬‬
‫‪99‬‬ ‫‪99‬‬
‫‪71‬‬ ‫‪1457‬‬
‫‪.Q‬‬ ‫‪14‬‬
‫‪99‬‬ ‫‪99‬‬

‫‪4‬‬
‫* حصر مساحة املستطيل هو ‪. 12,72 ab 14,58‬‬
‫‪ 10,17‬ومن حصـــــر نصـــــد‬ ‫‪r2‬‬ ‫* لدينا ‪ 1, 8 r 1, 9‬ومن ‪ 3,24 r 2 3, 61‬ومن ‪11, 34‬‬
‫‪r2‬‬
‫‪. 5, 09‬‬ ‫الدّاةر هو ‪5, 67‬‬
‫‪2‬‬
‫‪r2‬‬
‫‪. 17, 81‬‬ ‫‪ab‬‬ ‫* حصر مساحة الشكل هو ‪20,25‬‬
‫‪2‬‬

‫أستاذ المادّة‪ :‬عماد عقوني‬ ‫‪http://imadmaths.blogspot.com/‬‬


‫السّنة الدّراسيّة‪2016/2015 :‬‬ ‫ثانويّة شريّط لزهر – احلمّامات‬

‫الفرض الثاني للثالثي األول يف مادّة الرياضيات‬


‫املستوى‪ :‬أوىل جذع مشرتك علوم وتكنولوجيا‬

‫‪12‬‬
‫) ‪(C‬‬‫‪f‬‬
‫نعترب الدددّالددة ‪ f‬املعرّفدددة ثتمهيلددديدددا ال يدددان‬
‫املوضّح يف الرّسم املُلحق‪ .‬ثقراءة ثيانيّة‪:‬‬
‫‪ )1‬عيّن جمموعة تعريف الدّالة ‪. f‬‬
‫‪ )2‬عيّن صور األعداد‪ 1 ، 0 :‬و ‪ 1‬ثواسطة الدّالة ‪. f‬‬
‫‪1‬‬
‫و ‪ 3‬وفق الدّالة ‪f‬‬ ‫‪ )3‬عيّن سدددددواثق األعداد‪، 2 :‬‬
‫‪2‬‬
‫(إن وُجدت)‪.‬‬
‫‪ )4‬عيّن اجتاه تغيّر الدّالة ‪ ، f‬وشكل جدول تغيّراتيا‪.‬‬
‫‪ )5‬حدددل ثدديددداندديددّا املددعدددادلدددة ‪ f (x ) 0‬واملددرتاجددحددتددن‬
‫‪ f (x ) 0‬و ‪. f (x ) 0‬‬
‫‪ )6‬عيّن إشارة الدّالة ‪ f‬على جمموعة تعريفيا‪.‬‬

‫‪8‬‬

‫الشددددّكل املرفق يُمهّل مهلها ‪ ABC‬متقايس األضدددددلع لول ضدددددلع ‪ . 10‬لتكن ‪ P‬و ‪ Q‬نقطتن من ‪ AC‬و‬
‫‪ BC‬على التّرتيددحب ثيددو يكون نددل من املهلهن ‪ BMQ‬و ‪ APM‬متقددايس األضدددددلع‪ ،‬حيددو ‪ M‬نقطددة‬
‫‪. AM‬‬ ‫متحرّنة على القطعة ‪ AB‬وال تنط ق على أي من النّقطتن ‪ A‬و ‪ . B‬نضع ‪x‬‬

‫‪ )1‬أحسحب مساحة املهلو ‪ ، ABC‬ومساحيت املهلهن ‪ MQB‬و ‪ APM‬ثداللة ‪. x‬‬


‫‪ )2‬إذا علمتَ أنّ الرّثاع ‪ CPMQ‬متوازي أضلع‪ ،‬أث ت أنّ ‪ S‬مساحة‬
‫‪3‬‬
‫‪.S‬‬ ‫املهلو ‪ PMQ‬تُعطى ثالعلقة ‪25 (x 5)2‬‬
‫‪4‬‬
‫) ‪. f (x‬‬ ‫‪ )3‬نعترب الدّالة ‪ f‬املعرّفة على ‪ 0,10‬ثد ‪25 (x 5)2‬‬
‫أ) أدرس اجتاه تغيّر الدّالة ‪ f‬على اجملالن ‪ 0, 5‬و ‪. 5,10‬‬
‫ب) شكل جدول تغيّرات الدّالة ‪ f‬على جمموعة تعريفيا‪.‬‬
‫ج) استنتج أنرب مساحة للمهلو ‪ PMQ‬ومن أجل أي قيمة لد ‪x‬‬
‫ت لغيا؟‬

‫أستاذ المادّة‪ :‬عماد عقوني‬


‫التصحيح النموذجي‬
‫‪12‬‬
‫‪5‬‬
‫‪.‬‬ ‫‪,1‬‬ ‫‪ )1‬جمموعة تعريف الدّالة ‪ f‬ه‬
‫‪2‬‬
‫‪ )2‬صورة العدد ‪ 0‬ثالدّالة ‪ f‬ه ‪. f (0) 0‬‬
‫صورة العدد ‪ 1‬ثالدّالة ‪ f‬ه ‪. f (1) 2‬‬
‫صورة العدد ‪ 1‬ثالدّالة ‪ f‬ه ‪. f ( 1) 2‬‬
‫‪ )3‬ليس للعدد ‪ 3‬سواثق وفق الدّالة ‪. f‬‬
‫‪.x‬‬ ‫‪2‬‬ ‫‪x‬و‬ ‫سواثق العدد ‪ 2‬وفق الدّالة ‪ f‬ه ‪1 ، x 1‬‬
‫‪5‬‬ ‫‪1‬‬
‫‪.x‬‬ ‫وفق الدّالة ‪ f‬ه‬ ‫سواثق العدد‬
‫‪2‬‬ ‫‪2‬‬
‫‪5‬‬
‫ومتناقصة متاما على ‪ ، 1, 52; 0,25‬وجدول تغيّراتيا هو‪:‬‬ ‫‪; 1, 52‬‬ ‫‪ )4‬الدّالة ‪ f‬متزايدة متاما على ‪0,25;1‬‬
‫‪2‬‬

‫‪x‬و‬ ‫‪0, 48 ، x‬‬ ‫) ‪ f (x‬ه فواصدددددددل نقط تقددالع املنحنى ) ‪ (C f‬مع حمور الفواصدددددددل أي ‪0‬‬ ‫‪ )5‬حلول املعددادلددة ‪0‬‬
‫‪.x‬‬ ‫‪2, 42‬‬
‫) ‪ f (x‬هد د فدددواصددددددددل ندددقدددط املدددندددحدددندددى ) ‪ (C f‬الدددواقدددعدددة فدددو حمدددور الدددفدددواصددددددددل أي‬ ‫حدددلدددول املدددرتاجدددحدددة ‪0‬‬

‫‪.x‬‬ ‫‪2, 42; 0‬‬ ‫‪0, 48;1‬‬

‫دددت حمدددور الدددفدددواصددددددددل أي‬ ‫) ‪ f (x‬هد د فدددواصددددددددل ندددقدددط املدددندددحدددندددى ) ‪ (C f‬الدددواقدددعدددة‬ ‫حدددلدددول املدددرتاجدددحدددة ‪0‬‬

‫‪.x‬‬ ‫;‪2, 5‬‬ ‫‪2, 42‬‬ ‫‪0; 0, 48‬‬

‫‪ )6‬إشارة الدّالة ‪ f‬موضّحة يف اجلدول‬

‫‪8‬‬
‫‪ABC‬‬ ‫‪ ، h‬إذن مسددددداحة املهلو‬ ‫‪ 25‬ومن ‪5 3‬‬ ‫‪h2‬‬ ‫‪ )1‬ليكن ‪ h‬ارتفاع املهلو ‪ ، ABC‬حسدددددحب فيهالورس لدينا ‪100‬‬
‫‪. SABC‬‬ ‫‪25 3 u.c 2‬‬ ‫ه‬

‫أستاذ المادّة‪ :‬عماد عقوني‬


‫‪2‬‬
‫‪3‬‬ ‫‪10‬‬ ‫‪x‬‬
‫‪ ، y‬إذن‬ ‫‪(10‬‬ ‫ومن ) ‪x‬‬ ‫‪y2‬‬ ‫‪(10‬‬ ‫ليكن ‪ y‬ارتفاع املهلو ‪ ، MBQ‬حسدددددحب فيهالورس لدينا ‪x )2‬‬
‫‪2‬‬ ‫‪2‬‬

‫‪3‬‬
‫‪. SMBQ‬‬ ‫‪(10‬‬ ‫مساحة املهلو ‪ MBQ‬ه ‪x )2 u.c 2‬‬
‫‪4‬‬
‫‪2‬‬
‫‪3‬‬ ‫‪x‬‬
‫‪AMP‬‬ ‫‪ ، t‬إذن مسددددداحة املهلو‬ ‫ومن ‪x‬‬ ‫‪t2‬‬ ‫ليكن ‪ t‬ارتفاع املهلو ‪ ، AMP‬حسدددددحب فيهالورس لدينا ‪x 2‬‬
‫‪2‬‬ ‫‪2‬‬

‫‪3 2‬‬
‫‪. SMBQ‬‬ ‫ه ‪x u.c 2‬‬
‫‪4‬‬

‫‪ )2‬لدينا مساحة املهلو ‪ PMQ‬ه‬


‫‪1‬‬ ‫‪1‬‬ ‫‪3‬‬ ‫‪3 2‬‬
‫‪S‬‬ ‫‪S‬‬ ‫‪S BMQ‬‬ ‫‪S AMP‬‬ ‫‪25 3‬‬ ‫‪(10‬‬ ‫‪x )2‬‬ ‫‪x‬‬
‫‪2 ABC‬‬ ‫‪2‬‬ ‫‪4‬‬ ‫‪4‬‬
‫‪1 3‬‬ ‫‪1 3‬‬
‫‪100‬‬ ‫‪(10‬‬ ‫‪x )2‬‬ ‫‪x2‬‬ ‫‪20x‬‬ ‫‪2x 2‬‬
‫‪2 4‬‬ ‫‪2 4‬‬
‫‪3‬‬ ‫‪3‬‬
‫‪20x‬‬ ‫‪2x 2‬‬ ‫‪25‬‬ ‫‪25‬‬ ‫‪25‬‬ ‫‪(x‬‬ ‫‪5)2‬‬
‫‪4‬‬ ‫‪4‬‬
‫) ‪. f (x‬‬ ‫‪25‬‬ ‫‪(x‬‬ ‫‪ )3‬لدينا ‪5)2‬‬
‫دراسة تغيّرات الدّالة ‪f‬‬ ‫أ)‬
‫‪.a‬‬ ‫‪ ‬على اجملال ‪ : 0, 5‬ليكن ‪ a‬و ‪ b‬عددين حقيقيّن من ‪ 0, 5‬حيو ‪b‬‬

‫لددددديددددنددددا ‪ 0 a b 5‬يسدددددددددتددددلددددزم ‪ 5 a 5 b 5 0‬يسدددددددددتددددلددددزم ‪ (a 5)2 (b 5)2‬يسدددددددددتددددلددددزم‬


‫‪ (a 5)2‬يستلزم ‪ 25 (a 5)2 25 (b 5)2‬يستلزم )‪ ، f (a) f (b‬إذن الدّالة ‪ f‬متزايدة على اجملال‬ ‫‪(b 5)2‬‬
‫‪. 0, 5‬‬
‫‪.a‬‬ ‫‪ ‬على اجملال ‪ : 5,10‬ليكن ‪ a‬و ‪ b‬عددين حقيقيّن من ‪ 5,10‬حيو ‪b‬‬

‫لددددديددددنددددا ‪ 5 a b 10‬يسدددددددددتددددلددددزم ‪ 0 a 5 b 5 5‬يسدددددددددتددددلددددزم ‪ (a 5)2 (b 5)2‬يسددددددددتددددلددددزم‬


‫‪ (a 5)2‬يسدددددتلزم ‪ 25 (a 5)2 25 (b 5)2‬يسدددددتلزم )‪ ، f (a) f (b‬إذن الدّالة ‪ f‬مت ناقصدددددة على‬ ‫‪(b 5)2‬‬
‫اجملال ‪. 0, 5‬‬
‫ب) جدول تغيّرات الدّالة ‪ f‬هو‬

‫‪ x‬وعلي‬ ‫ج) القيمة احل ّد يّة العظمى للدّالة ‪ f‬على اجملال ‪( 0,10‬جمموعة تعريفيا) ه ‪ 25‬وت لغيا من أجل ‪5‬‬

‫‪25 3 2‬‬
‫‪.x‬‬ ‫وت لغيا من أجل ‪5‬‬ ‫أنرب مساحة للمهلو ‪ PMQ‬ه ‪u.c‬‬
‫‪4‬‬

‫أستاذ المادّة‪ :‬عماد عقوني‬


‫السّنة الدّراسيّة‪8112/8112 :‬‬ ‫ثانويّة سعدي الصديق ‪ -‬تبسة ‪-‬‬

‫املدّة‪ :‬ساعتان‬ ‫املستوى‪ :‬أوىل جذع مشرتك علوم و تكنولوجيا‬


‫التّمرين األوّل‪ 60( :‬نقاط)‬
‫أجب بصحيح أو خاطئ مع التّعليل‬
‫‪ 3‬طبيعي ‪.‬‬ ‫‪5‬‬ ‫‪3‬‬ ‫‪ )1‬العدد ‪5‬‬
‫‪ x‬فإ ّن ‪. d(x;1) 2‬‬ ‫‪ )2‬إذا كان ‪2; 4‬‬
‫) ‪ f (x‬زوجيّة ‪.‬‬ ‫‪x2‬‬ ‫‪x‬‬ ‫بــ ‪3 :‬‬ ‫‪ )3‬الدّالة ‪ f‬املعرفة على اجملال ‪4; 4‬‬
‫‪ )4‬العدد ‪ 983‬أوّلي ‪.‬‬
‫‪2‬‬
‫‪. 17 (x‬‬ ‫)‪3‬‬ ‫‪ )5‬إذا كان ‪ x 1‬فإن ‪1 :‬‬

‫‪ 6‬يساوي ‪. 6‬‬ ‫‪6‬‬ ‫‪6‬‬ ‫‪6‬‬ ‫‪ )6‬العدد ‪9‬‬

‫التّمرين الثاني‪ 60( :‬نقاط)‬


‫‪A‬‬ ‫‪2‬‬ ‫‪5‬‬ ‫‪2 5‬‬ ‫‪3‬‬ ‫‪7‬‬ ‫‪ )1‬عبّر عن العدد ‪ A‬دون رمز القيمة املطلقة حيث ‪3 5 :‬‬

‫‪J‬‬ ‫;‪1‬‬ ‫‪،I‬‬ ‫‪ I‬حيث ‪2;4 :‬‬ ‫‪J ،I‬‬ ‫‪ )2‬عيّن اجملموعات ‪J‬‬
‫‪1 3‬‬
‫‪b‬‬ ‫‪ a‬و ;‬ ‫‪ )3‬ليكن ‪ a‬و ‪ b‬عددان حقيقيان حيث ‪2; 3 :‬‬
‫‪2 2‬‬
‫‪a4 1‬‬
‫‪ -‬أعط حصراً للعبارتني ‪، 2a 2 4b :‬‬
‫‪2b 1‬‬
‫‪3‬‬ ‫‪ )4‬أدرس إشارة كال من العبارتني ‪ 2x 2 :‬و ‪2x‬‬
‫‪ 3 2x‬بإستعمال طريقة فصل احلاالت ‪.‬‬ ‫‪2x‬‬ ‫حل املرتاجحة ‪2 :‬‬

‫التّمرين الثالث‪ 60( :‬نقاط)‬


‫نعترب الدّالة العددية ‪ f‬املعرفة بتمثيلها البياني املوضّح يف الشكل املقابل ‪ .‬بقراءة بيانية أجب عن مايلي ‪:‬‬
‫‪ )1‬عيّن جمموعة تعريف الدّالة ‪. f‬‬
‫‪ )8‬عيّن صور األعداد‪ 3 ، 1 :‬و ‪ 4‬بواسطة الدّالة ‪. f‬‬
‫‪ )3‬عيّن سوابق األعداد ‪ 5 ، 0‬و ‪ 3‬وفق الدّالة ‪ f‬يف حال‬
‫وجودها ‪.‬‬
‫‪ )4‬صف سلوك الدّالة ‪ f‬على جمموعة تعريفها ‪.‬‬
‫‪ )5‬شكّل جدول تغيّرات الدّالة ‪. f‬‬
‫‪ )6‬عيّن القيمة احلدية العظمى و القيمة احلدية الصغرى‬
‫للدّالة ‪. f‬‬
‫‪ )7‬هل الدّالة ‪ f‬زوجية على اجملموعة ‪ 3; 3‬؟ ملاذا ؟‪.‬‬

‫‪1‬‬
‫السّنة الدّراسيّة‪2016/2015 :‬‬ ‫ثانويّة شريّط لزهر – احلمّامات‬

‫اختبار الثالثي األول يف مادّة الرياضيات‬


‫املدّة‪ :‬ساعتان‬ ‫املستوى‪ :‬أوىل جذع مشرتك علوم وتكنولوجيا‬

‫‪5‬‬
‫أجب بـ صحيح أو خاطئ مع التّعليل‬
‫طبيعي‪.‬‬ ‫‪2‬‬ ‫‪5‬‬ ‫‪2‬‬ ‫‪ )1‬العدد‬
‫‪5‬‬
‫)‪.d(x;1‬‬ ‫‪ x‬فإنّ ‪2‬‬ ‫‪ )2‬إذا كان ‪2, 4‬‬
‫) ‪ f (x‬زوجيّة‪.‬‬ ‫‪x2‬‬ ‫بـ ‪x‬‬ ‫‪4, 4‬‬ ‫‪ )3‬الدّالّة ‪ f‬املعرّفة على‬
‫‪ )4‬العدد ‪ 983‬أوّلي‪.‬‬
‫‪. 17 (x‬‬ ‫‪3)2‬‬ ‫‪ )5‬إذا كان ‪ x 1‬فإنّ ‪1‬‬

‫‪7‬‬
‫نعترب الـ ـدّالــّّة العـــدديــّة ‪ f‬املعرّفـــة على ‪6,5‬‬
‫بتمثيلها البياني املوضّح يف الشكل املرفق‪ .‬بقراءة بيانيّة‪:‬‬
‫‪ )1‬عيّن صور األعداد‪ 4 ، 1 :‬و ‪ 4‬بواسطة الدّالّة ‪. f‬‬
‫‪ )2‬عيّن ســـــوابق األعداد‪ 5 ، 3 :‬و ‪ 5‬وفق الدّا لّة ‪ f‬يف‬
‫حال وجودها‪.‬‬
‫‪ )3‬عيّن اجتاه تغيّر الدّالّة ‪ f‬على جمموعة تعريفها‪.‬‬
‫‪ )4‬شكّل جدول تغيّرات الدّالّة ‪. f‬‬
‫‪ )5‬حـــل‪ ،‬بــيـــانــيــّا‪ ،‬املــعـــادلـــة ‪ f (x ) 0‬واملــرتاجــحــتـ‬
‫‪ f (x ) 0‬و ‪. f (x ) 0‬‬
‫‪ )6‬عيّن إشارة الدّالّة ‪ f‬على جمموعة تعريفها‪.‬‬
‫‪ )7‬هل الدّالّة ‪ f‬زوجيّة على اجملموعة ‪ 3, 3‬؟‬

‫‪4‬‬
‫املســـــتقيد العدد مزوّد مبعلد ) ‪ ،(O, i‬نعترب النّقط ‪ C ، B ، A‬و ‪ D‬ذات الفواصـــــل ‪ 1 ، 5 ، 2‬و ‪ 3‬على‬
‫التّرتيب‪ ،‬ولتكن ‪ M‬نقطة من احملور احلقيقي فاصلتها ‪. x‬‬
‫‪ )1‬عبّر عن ‪ x 1 ، x 5 ، x 2‬و ‪ 3 x‬مبسافة‪.‬‬
‫‪ ، x‬باستعمال مفهوم املسافة‪.‬‬ ‫‪5‬‬ ‫‪3‬‬ ‫‪ x‬و ‪x‬‬ ‫‪2‬‬ ‫‪x‬‬ ‫‪ )2‬حل املعادلة ‪1‬‬
‫‪ ، 3 x‬باستعمال مفهوم املسافة‪.‬‬ ‫‪x‬‬ ‫‪ )3‬حل املرتاجحة ‪2‬‬

‫إقلب الصفحة‬ ‫صفحة ‪ 1‬من ‪2‬‬


‫‪4‬‬
‫‪ 32,2 cm‬و ‪ ، 34, 2 cm‬ويقوم‬ ‫يف حمــل للمــتكوخلت اةفيفــة‪ ،‬يطـــــنع الطّبـّا بيتزا طرهــا ‪ r‬يرتاوح ب‬
‫بتقسيمها إىل مثاني طع (نفرتض أنّها متساوية)‪.‬‬
‫ن طبق البيتزا دائر الشكل‪ ،‬أعطِ حطرا ملساحة وج طعة واحدة من الطّبق؟ نتخذ ‪3,14‬‬
‫‪ )1‬إذا افرتضنا أ ّ‬
‫‪ 1, 4 cm‬و ‪ ، 1, 6 cm‬أعطِ حطــرا‬ ‫‪ )2‬إذا افرتضــنا أ ّن طبق البيتزا املطــنوع أســطواني الشــكل ارتفاع ‪ h‬يرتاوح ب‬
‫حلمجد طعة واحدة من الطّبق‪.‬‬

‫أساتذة المادّة‬ ‫صفحة ‪ 1‬من ‪2‬‬


‫التصحيح النموذجي‬
‫‪5‬‬
‫‪0.5‬‬ ‫‪ )1‬خاطئ‬
‫‪0.5‬‬ ‫‪.‬‬ ‫‪ 2‬وخل نستطيع حساب اجلذر التّربيعي للعدد احلقيقي السالب يف‬ ‫‪5‬‬ ‫ن‪0‬‬
‫أل ّ‬
‫‪0.5‬‬ ‫‪ )2‬خاطئ‬
‫‪4‬‬ ‫)‪( 2‬‬ ‫‪2 4‬‬
‫‪ r‬إذن ميكن التّعبري عن اجملال‬ ‫‪ c‬ونطـــط طره هو ‪3‬‬ ‫هو ‪1‬‬ ‫ألنّ مركز اجملال ‪2, 4‬‬
‫‪2‬‬ ‫‪2‬‬
‫‪0.5‬‬
‫)‪. d(x,1‬‬ ‫بالكتابة ‪3‬‬ ‫‪2, 4‬‬
‫‪0.5‬‬
‫‪ )3‬خاطئ‬
‫‪0.5‬‬
‫ليست متناظرة بالنّسبة إىل الطفر‪.‬‬ ‫ن اجملموعة ‪4, 4‬‬
‫أل ّ‬
‫‪0.5‬‬ ‫‪ )4‬صحيح‬
‫لدينا اجلدول‬

‫‪37‬‬ ‫‪31‬‬ ‫‪29‬‬ ‫‪23‬‬ ‫‪19‬‬ ‫‪17‬‬ ‫‪13‬‬ ‫‪11‬‬ ‫العدد ‪ 983‬يقبل القسمة على ‪7 5 3 2‬‬
‫خل‬ ‫خل‬ ‫خل‬ ‫خل‬ ‫خل‬ ‫خل‬ ‫خل‬ ‫خل‬ ‫خل خل خل خل‬ ‫اإلجابة‬
‫ن العدد ‪ 983‬عدد أوّلي‪0.5 .‬‬
‫من خالل اجلدول نستنتج أ ّ‬
‫‪0.5‬‬ ‫‪ )5‬صحيح‬
‫‪0.5‬‬ ‫‪. 17 (x‬‬ ‫‪3)2‬‬ ‫‪ (x‬ومن ‪1‬‬ ‫‪3)2‬‬ ‫‪ (x‬ومن ‪16‬‬ ‫‪3)2‬‬ ‫‪ x‬ومن ‪16‬‬ ‫‪3‬‬ ‫‪ x‬ومن ‪4‬‬ ‫لدينا ‪1‬‬

‫‪7‬‬
‫‪0.5‬‬ ‫)‪. f (1‬‬ ‫‪ )1‬صورة العدد ‪ 1‬بواسطة الدّالّة ‪ f‬هي ‪3‬‬
‫‪0.5‬‬ ‫)‪. f (4‬‬ ‫صورة العدد ‪ 4‬بواسطة الدّالّة ‪ f‬هي ‪4‬‬
‫)‪. f ( 4‬‬
‫‪0.5‬‬ ‫صورة العدد ‪ 4‬بواسطة الدّالّة ‪ f‬هي ‪3‬‬
‫‪0.5 . x‬‬ ‫‪x‬و‪1‬‬ ‫‪ )2‬سابقتا العدد ‪ 3‬وفق الدّالّة ‪ f‬هما ‪1‬‬
‫العدد ‪ 5‬ليست ل صور وفق الدّالّة ‪0.5 . f‬‬

‫‪0.5 . x‬‬ ‫‪x‬و‪3‬‬ ‫سابقتا العدد ‪ 5‬وفق الدّالّة ‪ f‬هما ‪3‬‬


‫‪0.5‬‬
‫‪. 6, 3‬‬ ‫‪ 3, 0‬ومتنا طة متاما على ‪0, 3‬‬ ‫‪ )3‬الدّالّة ‪ f‬متزايدة متاما على ‪3, 5‬‬
‫‪ )4‬جدول تغيّرات الدّالّة ‪ f‬هو‬

‫‪1‬‬

‫‪0.5‬‬
‫‪x‬‬ ‫‪2‬‬ ‫‪x‬و‬ ‫‪2 ،x‬‬ ‫) ‪ f (x‬هي فواصل نقط تقاطع بيان الدّالّة ‪ f‬مع حمور الفواصل‪ ،‬أ ‪5‬‬ ‫‪ )5‬حلول املعادلة ‪0‬‬
‫‪.x‬‬ ‫‪6, 5‬‬ ‫‪2,2‬‬ ‫) ‪ f (x‬هي فواصل نقط بيان الدّالّة ‪ f‬الوا عة فوق حمور الفواصل‪ ،‬أ‬ ‫حلول املرتاجحة ‪0‬‬ ‫‪0.5‬‬

‫أستاذ المادّة‪ :‬عماد عقوني‬ ‫صفحة ‪ 1‬من ‪2‬‬


‫‪.x‬‬ ‫‪5, 2‬‬ ‫‪2, 5‬‬ ‫) ‪ f (x‬هي فواصل نقط بيان الدّالّة ‪ f‬الوا عة حتت حمور الفواصل‪ ،‬أ‬ ‫حلول املرتاجحة ‪0‬‬ ‫‪0.5‬‬

‫‪ )6‬إشارة الدّالّة ‪ f‬موصّحة يف اجلدول‬

‫‪0.5‬‬

‫متناظرة بالنّسبة إىل الطفر‪ ،‬وبيان الدّالّة ‪ f‬متناظر بالنّسبة إىل حمور التّراتيب على هذه اجملموعة‬ ‫‪ )7‬اجملموعة ‪3, 3‬‬
‫‪0.5‬‬
‫فهي دالّة زوجيّة على اجملموعة ‪. 3, 3‬‬

‫‪4‬‬
‫‪0.5‬‬
‫‪ x‬هي املسافة ب النّقطة ‪ M‬والنّقطة ‪. A‬‬ ‫‪ )1‬الكتابة ‪2‬‬
‫‪0.5‬‬
‫‪ x‬هي املسافة ب النّقطة ‪ M‬والنّقطة ‪. B‬‬ ‫الكتابة ‪5‬‬
‫‪0.5‬‬
‫‪ x‬هي املسافة ب النّقطة ‪ M‬والنّقطة ‪.C‬‬ ‫الكتابة ‪1‬‬
‫‪0.5‬‬
‫الكتابة ‪ 3 x‬هي املسافة ب النّقطة ‪ M‬والنّقطة ‪. D‬‬
‫‪ AM‬أ أنّ الـــنــّقـــطـــة ‪ M‬مـــنـــتطــــــــط الـــقـــطـــعـــة ‪ ، AC‬أ أ ّن‬ ‫‪CM‬‬ ‫‪ x‬تـــكـــافـــئ‬ ‫‪2‬‬ ‫‪x‬‬ ‫‪ )2‬لــــديـــنـــا ‪1‬‬

‫‪0.5‬‬ ‫‪xA‬‬ ‫‪xC‬‬ ‫‪1‬‬


‫‪.x‬‬
‫‪2‬‬ ‫‪2‬‬
‫‪ BM‬أ أنّ الـــنــّقـــطـــة ‪ M‬مـــنـــتطــــــــط الـــقـــطـــعـــة ‪ ، BD‬أ أنّ‬ ‫‪DM‬‬ ‫‪ x‬تـــكـــافـــئ‬ ‫‪5‬‬ ‫‪3‬‬ ‫لـــديـــنـــا ‪x‬‬
‫‪0.5‬‬ ‫‪xB‬‬ ‫‪xD‬‬
‫‪.x‬‬ ‫‪1‬‬
‫‪2‬‬
‫‪ DM‬أ أنّ النّقطــة ‪ M‬أ رب إىل النّقطــة ‪ D‬منهــا إىل النّقطــة ‪ ، A‬أ أ ّن‬ ‫‪ 3‬تكــافئ ‪AM‬‬ ‫‪x‬‬ ‫‪x‬‬ ‫‪ )3‬لــدينــا ‪2‬‬

‫‪1‬‬ ‫‪1‬‬
‫‪.x‬‬ ‫‪,‬‬
‫‪2‬‬

‫‪0.5‬‬ ‫‪0.5‬‬
‫‪2‬‬
‫‪ 259,21 a‬ومـــنـ ـ‬ ‫‪ 16,1‬ومـــنـ ـ ‪292, 41‬‬ ‫‪a‬‬ ‫‪ )1‬لــــيـــكـــن ‪ a‬نطــــــــط ـــطـــر الـــطـــبـــق‪ ،‬إذن لـــديـــنـــا ‪17,1‬‬
‫‪1 2‬‬ ‫‪0.5‬‬
‫‪. 101,74‬‬ ‫‪a 114,77‬‬ ‫‪ ، 813,92‬وعلي حطـــــر مســـــاحة وج طعة واحدة من الطبق هو‬ ‫‪a2‬‬ ‫‪918,17‬‬
‫‪8‬‬
‫‪0.5‬‬
‫‪0.5‬‬ ‫‪0.5‬‬ ‫(الوحدة هي السنتيمرت مربّع)‬
‫‪2‬‬ ‫‪2‬‬
‫‪ 1139, 49‬ومن حطـــر‬ ‫‪h a‬‬ ‫‪ 813,92‬ومن ‪1469, 07‬‬ ‫‪a‬‬ ‫‪ )2‬لدينا حطـــر مســـاحة وج الطبق هو ‪918,17‬‬
‫‪1‬‬
‫‪( . 142, 44‬الوحدة هي السنتيمرت مكعب)‪.‬‬ ‫‪h a2‬‬ ‫حمجد طعة واحدة من الطبق هو ‪183, 63‬‬
‫‪8‬‬
‫‪0.5‬‬ ‫‪0.5‬‬

‫أستاذ المادّة‪ :‬عماد عقوني‬ ‫صفحة ‪ 2‬من ‪2‬‬


‫السّنة الدّراسيّة‪2016/2015 :‬‬ ‫ثانويّة شريّط لزهر – احلمّامات‬

‫إقرتاح الفرض الثاني للثالثي األول يف مادّة الرياضيات‬


‫املستوى‪ :‬أوىل جذع مشرتك علوم وتكنولوجيا‬

‫‪12‬‬
‫نعترب الددّالدة ‪ f‬املعرّفددة متمليلددلددا ال يددان ) ‪ (C f‬املوضددددّ‬
‫الرّسم املُلحق‪ .‬مقراءة ميانيّة‪:‬‬
‫‪ )1‬عيّن جمموعة تعريف الدّالة ‪. f‬‬
‫‪ )2‬عيّن صور األعداد‪ 1 ، 0 :‬و ‪ 2‬مواسطة الدّالة ‪. f‬‬
‫‪17‬‬
‫وفق الددّالدة ‪( f‬إن‬ ‫و‬ ‫‪ )3‬عيّن سدددددوامق األعددداد‪2 ، 4 :‬‬
‫‪4‬‬
‫وُجدت)‪.‬‬
‫‪ )4‬عيّن اجتاه تغيّر الدّالة ‪ ، f‬وشكل جدول تغيّراتلا‪.‬‬
‫) ‪ f (x‬و‬ ‫‪ )5‬حددل ميددانيدّاد املعددادلددة ‪ f (x ) 0‬واملرتاجحتني ‪0‬‬
‫‪. f (x ) 0‬‬
‫‪ )6‬عيّن إشارة الدّالة ‪ f‬على جمموعة تعريفلا‪.‬‬

‫‪8‬‬
‫الشّكل املرفق يُملّل نصف دائرة (نصف قطرها ‪ ) 4‬حميط‬
‫مبللث ‪ ABC‬قائم ‪ A‬حيث ‪. 0 x 8‬‬
‫‪ )1‬أحسب قيمة ‪ y‬مداللة ‪. x‬‬
‫‪ )2‬حتقق أنّ ‪ S‬مساحة املللث ‪ ABC‬تُعطى مالعالقة‬
‫‪1‬‬
‫‪.S‬‬ ‫‪x 64‬‬ ‫‪x2‬‬
‫‪2‬‬
‫‪1‬‬
‫) ‪. f (x‬‬ ‫‪x 64‬‬ ‫‪ )3‬نعترب الدّالة ‪ f‬املعرّفة على ‪ 0, 8‬مد ‪x 2‬‬
‫‪2‬‬
‫أ) أدرس اجتاه تغيّر الدّالة ‪ f‬على اجملالني ‪ 0, 4 2‬و‬

‫‪. 4 2, 8‬‬
‫ب) شكل جدول تغيّرات الدّالة ‪ f‬على جمموعة تعريفلا‪.‬‬
‫ج) استنتج أكرب مساحة للمللث ‪ ABC‬ومن أجل أي قيمة لد ‪ x‬ت لغلا؟‬

‫أستاذ المادّة‪ :‬عماد عقوني‬


‫التصحيح النموذجي‬
‫‪12‬‬
‫‪. 2,2‬‬ ‫‪ )1‬جمموعة تعريف الدّالة ‪ f‬ه‬
‫)‪. f (0‬‬ ‫‪ )2‬صورة العدد ‪ 0‬مالدّالة ‪ f‬ه ‪2‬‬
‫)‪. f (1‬‬ ‫صورة العدد ‪ 1‬مالدّالة ‪ f‬ه ‪2‬‬
‫)‪. f (2‬‬ ‫صورة العدد ‪ 2‬مالدّالة ‪ f‬ه ‪2‬‬
‫‪ )3‬ليس للعدد ‪ 4‬سوامق وفق الدّالة ‪. f‬‬
‫‪.x‬‬ ‫‪2‬‬ ‫‪x‬و‬ ‫‪1 ،x‬‬ ‫‪1 ،x‬‬ ‫سوامق العدد ‪ 2‬وفق الدّالة ‪ f‬ه ‪2‬‬
‫‪3‬‬ ‫‪3‬‬ ‫‪17‬‬
‫‪.x‬‬ ‫‪x‬و‬ ‫وفق الدّالة ‪ f‬ه‬ ‫سوامق العدد‬
‫‪2‬‬ ‫‪2‬‬ ‫‪4‬‬
‫‪3‬‬ ‫‪3‬‬ ‫‪3‬‬ ‫‪3‬‬
‫‪ ، 2,‬وجدول تغيّراتلا هو‪:‬‬ ‫‪0,‬‬ ‫ومتناقصة متاماد على‬ ‫‪,0‬‬ ‫‪ )4‬الدّالة ‪ f‬متزايدة متاماد على ‪,2‬‬
‫‪2‬‬ ‫‪2‬‬ ‫‪2‬‬ ‫‪2‬‬

‫‪.x‬‬ ‫‪ x‬و ‪0,7‬‬ ‫) ‪ f (x‬ه فواصل نقط تقاطع املنحنى ) ‪ (C f‬مع حمور الفواصل أي ‪0,7‬‬ ‫‪ )5‬حلول املعادلة ‪0‬‬

‫‪.x‬‬ ‫‪0, 7; 0, 7‬‬ ‫) ‪ f (x‬ه فواصل نقط املنحنى ) ‪ (C f‬الواقعة فوق حمور الفواصل أي‬ ‫حلول املرتاجحة ‪0‬‬

‫) ‪ f (x‬هد د فدددواصددددددددل ندددقدددط املدددندددحدددندددى ) ‪ (C f‬الدددواقدددعدددة حتد د حمدددور الدددفدددواصددددددددل أي‬ ‫حدددلدددول املدددرتاجدددحدددة ‪0‬‬
‫‪.x‬‬ ‫‪2; 0, 7‬‬ ‫‪0, 7;2‬‬
‫‪ )6‬إشارة الدّالة ‪ f‬موضّحة اجلدول‬

‫‪8‬‬
‫‪.y‬‬ ‫‪64‬‬ ‫‪x2‬‬ ‫‪ x 2‬ومنه‬ ‫‪y2‬‬ ‫‪ A‬فإنّه حسب مربهنة فيلاغورس يكون ‪64‬‬ ‫‪ )1‬مبا أ ّن املللث ‪ ABC‬قائم‬
‫‪1‬‬ ‫‪1‬‬
‫‪.S‬‬ ‫‪xy‬‬ ‫‪x 8‬‬ ‫‪x2‬‬ ‫‪ )2‬لدينا مساحة املللث ‪ ABC‬ه‬
‫‪2‬‬ ‫‪2‬‬
‫‪1‬‬
‫) ‪. f (x‬‬ ‫‪x 64‬‬ ‫‪ )3‬لدينا ‪x 2‬‬
‫‪2‬‬
‫دراسة تغيّرات الدّالة ‪f‬‬ ‫أ)‬
‫‪.a‬‬ ‫‪ ‬على اجملال ‪ : 0, 4 2‬ليكن ‪ a‬و ‪ b‬عددين حقيقيّني من ‪ 0, 4 2‬حيث ‪b‬‬

‫أستاذ المادّة‪ :‬عماد عقوني‬


‫‪1‬‬ ‫‪1‬‬
‫‪a 64‬‬ ‫‪a2‬‬ ‫‪b 64‬‬ ‫‪b2‬‬
‫) ‪f (a‬‬ ‫)‪f (b‬‬ ‫‪2‬‬ ‫‪2‬‬ ‫‪a 64‬‬ ‫‪a 2 b 64‬‬ ‫‪b2‬‬ ‫‪(a‬‬ ‫‪b)(a 2‬‬ ‫‪b2‬‬ ‫)‪64‬‬
‫‪a‬‬ ‫‪b‬‬ ‫‪a‬‬ ‫‪b‬‬ ‫)‪2(a b‬‬ ‫‪2( 64‬‬ ‫‪a2‬‬ ‫‪b 64‬‬ ‫) ‪b2‬‬

‫‪ 0‬ومددنددده‬ ‫‪a2‬‬ ‫‪b2‬‬ ‫‪ 0‬ومددنددده ‪64‬‬ ‫‪b2‬‬ ‫‪ 0‬و ‪32‬‬ ‫‪a2‬‬ ‫مبدددا أنّ ‪ a‬و ‪ b‬عدددددان حددقدديددقدديددّان مددن ‪ 0, 4 2‬فدددإنّ ‪32‬‬

‫) ‪f (a‬‬ ‫)‪f (b‬‬


‫‪ ،‬إذن الدّالة ‪f‬‬ ‫‪ a‬ومنه ‪0‬‬ ‫‪b‬‬ ‫و‪0‬‬ ‫‪64‬‬ ‫‪a2‬‬ ‫‪b 64‬‬ ‫‪b2‬‬ ‫‪ ، 64‬ولدينا ‪0‬‬ ‫‪a2‬‬ ‫‪b2‬‬ ‫‪64‬‬ ‫‪0‬‬
‫‪a‬‬ ‫‪b‬‬
‫متزايدة على اجملال ‪. 0, 4 2‬‬

‫‪.a‬‬ ‫‪ ‬على اجملال ‪ : 4 2, 8‬ليكن ‪ a‬و ‪ b‬عددين حقيقيّني من ‪ 4 2, 8‬حيث ‪b‬‬

‫‪1‬‬ ‫‪1‬‬
‫‪a 64‬‬ ‫‪a2‬‬ ‫‪b 64‬‬ ‫‪b2‬‬
‫) ‪f (a‬‬ ‫)‪f (b‬‬ ‫‪2‬‬ ‫‪2‬‬ ‫‪a 64‬‬ ‫‪a 2 b 64‬‬ ‫‪b2‬‬ ‫‪(a‬‬ ‫‪b)(a 2‬‬ ‫‪b2‬‬ ‫)‪64‬‬
‫‪a‬‬ ‫‪b‬‬ ‫‪a‬‬ ‫‪b‬‬ ‫)‪2(a b‬‬ ‫‪2( 64‬‬ ‫‪a2‬‬ ‫‪b 64‬‬ ‫) ‪b2‬‬

‫‪ 64‬ومندده‬ ‫‪a2‬‬ ‫‪b2‬‬ ‫‪ 32‬ومندده ‪128‬‬ ‫‪b2‬‬ ‫‪ 32‬و ‪64‬‬ ‫‪a2‬‬ ‫مبددا أنّ ‪ a‬و ‪ b‬عددددان حقيقيدّان من ‪ 4 2, 8‬فددإنّ ‪64‬‬

‫) ‪f (a‬‬ ‫)‪f (b‬‬


‫‪ ،‬إذن ال ددّال دة ‪f‬‬ ‫‪ a‬ومندده ‪0‬‬ ‫‪b‬‬ ‫و ‪0‬‬ ‫‪64‬‬ ‫‪a2‬‬ ‫‪b 64‬‬ ‫‪b2‬‬ ‫‪ ، 0‬ولدددينددا ‪0‬‬ ‫‪a2‬‬ ‫‪b2‬‬ ‫‪64‬‬ ‫‪64‬‬
‫‪a‬‬ ‫‪b‬‬
‫متناقصة على اجملال ‪. 4 2, 8‬‬
‫ب) جدول تغيّرات الدّالة ‪ f‬هو‬

‫‪ x‬وعليه‬ ‫ج) القيمة احلدّيّة العظمى للدّالة ‪ f‬على اجملال ‪( 0, 8‬جمموعة تعريفلا) ه ‪ 16‬وت لغلا من أجل ‪4 2‬‬

‫‪.x‬‬ ‫أكرب مساحة للمللث ‪ ABC‬ه ‪ 16 u.c2‬وت لغلا من أجل ‪4 2‬‬

‫مالحظة‪ :‬الرتميز ‪ u.c 2‬يعين وحدة مرمّعة‪.‬‬

‫أستاذ المادّة‪ :‬عماد عقوني‬


‫السّنة الدّراسيّة‪2016/2015 :‬‬ ‫ثانويّة شريّط لزهر – احلمّامات‬

‫واجب منزلي رقم ‪ 01‬يف مادّة الرياضيات‬


‫املستوى‪ :‬أوىل جذع مشرتك علوم وتكنولوجيا‬

‫‪.b‬‬ ‫‪a‬و ‪2 7‬‬ ‫‪ )1‬قارن بني العددين ‪3 3‬‬


‫‪.(3 3‬‬ ‫‪ )2‬بسّط العدد ‪2 7 )2‬‬

‫‪.x‬‬ ‫‪55‬‬ ‫‪ )3‬استنتج كتابة مُبسّطة للعدد ‪ x‬حيث ‪12 21‬‬

‫‪. 2a 3b 5c‬‬ ‫‪ )1‬حدّد ثالث أعداد طبيعيّة ‪ b ، a‬و ‪ c‬حيث ‪486000‬‬

‫‪ A‬عدد طبيعي‪.‬‬ ‫‪7‬‬ ‫‪2 6‬‬ ‫‪7‬‬ ‫‪ )2‬أثبت أنّ العدد ‪2 6‬‬
‫‪2 103(3 10‬‬ ‫‪5 2‬‬
‫)‬
‫‪.B‬‬ ‫‪3‬‬
‫‪ )3‬أعطِ الكتابة العشريّة والعلميّة للعدد‬
‫‪1, 5 10‬‬
‫‪( . 333332‬يُمنع استعمال اآللة احلاسبة)‬ ‫‪444442‬‬ ‫‪ )4‬برهن أنّ ‪555552‬‬

‫أثبت أنّ العدد ‪ 2‬ليس عدداً ناطقاً‪.‬‬

‫أستاذ المادّة‪ :‬عماد عقوني‬

‫تاريخ التسليــم‪2015/09/28 :‬‬


‫تاريخ اإلستالم‪2015/10/01 :‬‬
‫السّنة الدّراسيّة‪2016/2015 :‬‬ ‫ثانويّة شريّط لزهر – احلمّامات‬

‫تصحيح الواجب املزنلل قم ‪ 01‬يف مادّة الرياضيات‬


‫املستوى‪ :‬أوىل جذع مشرتك علوم وتكنولوجيا‬

‫‪.a‬‬ ‫‪ a 2‬ومنه ‪b‬‬ ‫‪ b 2‬ومنه ‪b2‬‬ ‫‪ a 2‬و ‪28‬‬ ‫‪ )1‬لدينا ‪27‬‬


‫‪.(3 3‬‬ ‫‪2 7 )2‬‬ ‫‪27‬‬ ‫‪28‬‬ ‫‪12 21‬‬ ‫‪55‬‬ ‫‪ )2‬لدينا ‪12 21‬‬

‫‪.x‬‬ ‫‪55‬‬ ‫‪12 21‬‬ ‫‪2 7‬‬ ‫‪ a‬فإنّ ‪3 3‬‬ ‫‪ )3‬مبا أنّ ‪b‬‬

‫‪،a‬‬ ‫للد أنّ ‪4‬‬ ‫‪ 486000‬ومنلله‬ ‫‪24‬‬ ‫‪35‬‬ ‫‪ )1‬حتليللا الدللد ‪ 486000‬إىل جللداع عوامللا أوليلّة هو ‪53‬‬
‫‪.c‬‬ ‫‪b‬و ‪3‬‬ ‫‪5‬‬

‫‪ A‬إذن فهو عد طبيدي‪.‬‬ ‫‪7‬‬ ‫‪2 6‬‬ ‫‪7‬‬ ‫‪2 6‬‬ ‫‪(7‬‬ ‫‪2 6)(7‬‬ ‫)‪2 6‬‬ ‫‪ )2‬لدينا ‪5‬‬

‫‪2 103(3 10‬‬ ‫)‬


‫‪5 2‬‬
‫‪18 10‬‬ ‫‪7‬‬
‫‪4‬‬ ‫‪3‬‬
‫‪ ، B‬والكتابة الدشلللللريّة هي‬ ‫‪3‬‬ ‫‪3‬‬
‫‪12 10‬‬ ‫‪1,2 10‬‬ ‫‪ )3‬لدينا‬
‫‪1, 5 10‬‬ ‫‪1, 5 10‬‬
‫‪12‬‬
‫‪.B‬‬
‫‪104‬‬
‫‪2‬‬
‫‪ 55555‬ومنه‬ ‫‪52‬‬ ‫‪ 333332‬وكذلك ‪111112‬‬ ‫‪32‬‬ ‫‪ 33333‬ومنه ‪111112‬‬ ‫‪ )4‬لدينا ‪3 11111‬‬
‫‪333332‬‬ ‫‪444442‬‬ ‫‪111112 (32‬‬ ‫) ‪42‬‬ ‫‪111112‬‬ ‫‪25‬‬ ‫‪111112‬‬ ‫‪52‬‬ ‫‪(11111 5)2‬‬ ‫‪555552‬‬

‫نفرض أنّ الدد ‪ 2‬عد ناطق‪ ،‬إذن يُوجد عد ان صللل ل ي ان نسلللللب يّان ‪ p‬و ‪ q‬أوّليان فيما بينهما حبيث‬
‫‪p‬‬
‫‪ p‬ومنه‬ ‫‪ p 2‬ومنه الدد ‪ 2‬يقسللل ل ‪ p 2‬ومبا أنّ ‪ 2‬أوّلي فإنّ ‪ 2‬يقسللل ل ‪ p‬أي أنّ ‪2k‬‬ ‫ومنه ‪2q 2‬‬ ‫‪2‬‬
‫‪q‬‬
‫‪p‬‬
‫قابا لإلختزال (أي أنّ الدد ين ‪ p‬و ‪ q‬غري‬ ‫‪ q 2‬ومنه ‪ 2‬يقسللل ل ‪ ، q‬ومنه الكسلللللر‬ ‫‪ 4k 2‬ومنه ‪2k 2‬‬ ‫‪2q 2‬‬
‫‪q‬‬
‫أوّليان فيما بينهما)‪ ،‬وهذا تناقض مع الفرض‪ ،‬ومنه الدد ‪ 2‬ليس عد اً ناطقاً‪.‬‬

‫أستاذ المادّة‪ :‬عماد عقوني‬


2013 2012 2013 2012
1 5 1 1 5 1
10 10
A=− (7 + )(
48 7 − 48 ) 1 A=− (7 + )(
48 7 − 48 ) 1
9 2 − 8 9 2 − 8
C= B= C= B=
400  −4 400  −4
4 2 3 4 2 3
7 4  2  7 4  2 
D D =       2 D D =       2
 2   7   −49   2   7   −49 
B = −0.0000012 A = 108 1012 3 B = −0.0000012 A = 108 1012 3
A A
A B A B
B B
378 378
378 420 4 378 420 4
420 420
10 10
4−2 3 3 −1 -1 4−2 3 3 −1 -1
1+
a
a +1 a0 -2 1+
a
a +1 a0 -2
2 2
2 +1 2 +1
2+2 1+ 2+2 1+
2 2
2a − b 2a − b
0  b 1 1 a  3 A= A -3 0  b 1 1 a  3 A= A -3
a2 a2
10
‫ن‬3 -1

2 − 8
(7 + )( )
9
C= B= A=− 48 7 − 48
400  −4
2 ( − 4 )
3
(7) ( )
2
A=− −
2
C= B= 48
20  −4
C  ID B=2 A = −1
‫ن‬2 B A

D -2
4 2 3
7 4  2 
D =      
 2   7   −49 
 7 4 2 4 23 
D = − 4  2  6  •
2 7 7 
23 8
D=− 4 =−
7 2401
D •
‫ن‬2
A
A B -3
B
B = −1.2 10−6 A = 1.08 1014 B A
−110−6 11014 B A
A B
−1108 A  B 11014  −110−6 = −1108
A
B
A 110 14
−11020 = −11020
B −110−6
‫ن‬3 378
420 378 -4
420
420 = 22  3  5  7 378 = 2  33  7 •
378 23  73
3 9 2
= 2 = = •
420 2  3  5  7 2  5 10
10

‫ن‬3 4 − 2 3 ‫ و‬1− 3 -1

( ) ( )
2 2
3 −1 = 4 − 2 3 3 −1 = 4−2 3

a
‫ن‬4 1+ a +1 -2
2
2
a
( )  a
2
a +1 1+ a +1 1 +  •
2  2
2 +1
2+2 1+ a = 2 +1 a •
2
2a − b
0  b 1 1 a  3 A= A -3
a2
‫ن‬3
1  2a − b  6

1 1
 1 ‫و‬
9 a2

1 2a − b
 6
9 a2
‫السّنة الدّراسيّة‪9191/9112 :‬‬ ‫ثانويّة سعدي الصديق ‪ -‬تبسة ‪-‬‬

‫املدّة‪ :‬ساعتان‬ ‫املستوى‪ :‬أوىل جذع مشرتك علوم و تكنولوجيا‬


‫التّمرين األوّل‪ 80( :‬نقاط)‬
‫أجب بصحيح أو خاطئ مع التّعليل ‪:‬‬
‫‪ 4‬عدد صحيح ‪.‬‬ ‫‪7‬‬ ‫‪4‬‬ ‫‪ )1‬العدد ‪7‬‬
‫‪ )2‬الدّالة ‪ f‬املعرفة على ‪ 4; 4‬بــ ‪ f (x ) x 2 x :‬زوجيّة ‪.‬‬
‫)‪. d(x;1‬‬ ‫‪ x‬فإن ‪2 :‬‬ ‫‪ )3‬إذا كان ‪2; 4‬‬

‫‪.x‬‬ ‫‪x2‬‬ ‫‪ x‬فإن ‪x 3 x 4 :‬‬ ‫‪ )4‬إذا كان ‪24 13 2 :‬‬


‫‪ )5‬إذا كان ‪ x 1; 3‬فإن ‪. 5 (x 3)2 10 :‬‬
‫‪5 1‬‬ ‫‪5 1‬‬
‫‪. a 2018 b2019‬‬ ‫‪ b‬فإن ‪b :‬‬ ‫‪a‬و‬ ‫‪ )6‬إذا كان ‪:‬‬
‫‪2‬‬ ‫‪2‬‬
‫‪.‬‬ ‫‪ )7‬قيم العدد ‪ x‬اليت حتقق ‪ x 1 5‬هي‪5;5 :‬‬

‫‪.3 3‬‬ ‫‪ 3 3‬يساوي ‪2 7 :‬‬ ‫‪ )8‬العدد ‪2 7 :‬‬

‫التّمرين الثاني‪ 80( :‬نقاط)‬


‫‪y‬‬ ‫‪a‬‬ ‫‪ x‬و ‪b‬‬ ‫‪ab‬‬ ‫‪ . b‬نضع ‪1‬‬ ‫ليكن ‪ a‬و ‪ b‬عددان حقيقيان حبيث ‪ a 2 :‬و ‪2‬‬
‫‪ )1‬بيّن أن )‪. x 2 y 2 (a 1)(b 1‬‬
‫‪ )2‬قارن بني ‪ x 2‬و ‪ y 2‬و استنتج مقارنة بني ‪ x‬و ‪. y‬‬
‫‪. 3‬‬ ‫‪2‬‬ ‫‪ )3‬استنتج مقارنة بني العددين ‪ 6 :‬و ‪1‬‬
‫التّمرين الثالث‪ 80( :‬نقاط)‬
‫التمثيل البياني املقابل ‪ C f‬لدالة ‪. f‬بقراءة بيانية أجب على مايلي ‪:‬‬
‫‪ )1‬عيّن جمموعة تعريف الدّالة ‪. f‬‬
‫‪9 5‬‬
‫‪.‬‬ ‫‪ ،0،‬و‬ ‫‪2،‬‬ ‫‪ )2‬عيّن صور كل من األعداد ‪3 :‬‬
‫‪2 2‬‬
‫‪ )3‬عيّن السوابق املمكنة للعددين ‪ 2 :‬و ‪. 1‬‬
‫‪ )4‬عيّن اجتاه تغري الدّالة ‪. f‬‬
‫‪ )5‬عيّن القيم احلدية للدالة ‪. f‬‬
‫‪ )6‬شكل جدول تغريات الدالة ‪. f‬‬
‫‪ )7‬هل الدالة ‪ f‬زوجية على جمال جمموعة تعريفها ؟ ملاذا ؟‬
‫‪ ‬مالذي ميكن قوله حول شفعية الدّالة ‪ f‬يف اجملـال ‪1;1‬‬
‫و ملاذا ؟‬

‫‪1‬‬
‫تمرين إضافي‪ ( :‬خارج سلم التنقيط)‬
‫يف حمـــل للمـــاكويت ا‪،‬فيفـــة ‪ ،‬يصـــنع الطبـــا بيتـ ـ ا قطرهـــا ‪ r‬يـــرتاوني بـــني ‪ 32,2 cm‬و ‪ ، 34,2 cm‬يقـــوم‬
‫بتقسيمها إىل مثاني قطع ( نفرض أن القطع متساوية ) ‪.‬‬
‫‪.‬‬ ‫‪ )1‬إذا افرتضنا أن طبق البيت ا دائري الشكل ‪ ،‬أعط حصرا ملساحة وجه قطعة واحدة من الطبق ؟ ناخذ ‪3,14‬‬
‫‪ )2‬إذا افرتضنا أن طبق البيتـ ا املصـنوع أسـطواني الشـكل ارتفاعـه ‪ h‬يـرتاوني بـني ‪ 1,4 cm‬و ‪ 1,6 cm‬أعـط حصـرا‬
‫حلجم قطعة واحدة من الطّبق ‪.‬‬

‫‪9‬‬
‫انًسزوى‪ :‬األوني ثبَوً جرع يشزسك عهوو وركُونوجَب‬
‫انًدح ‪ :‬سبعزبٌ‬ ‫اخزجبز فٌ يبدح ‪ :‬انسٍبضَبد‬

‫التمرين األول( ‪ 50‬نقاط ) ‪:‬‬

‫التمرين الثاني(‪ 50‬نقاط )‪:‬‬


‫‪ A ‬و ‪B  1 2 2‬‬ ‫‪ .1‬اكزت انعددٍٍ ‪ A‬و ‪ B‬دوٌ زيز انقًَخ انًطهقخ حَث ‪2  1  2  2 2  (2  2) 2 :‬‬

‫ثى قبزٌ ثٍَ انعددٍٍ ‪ A‬و ‪.B‬‬


‫‪K  ;0‬‬ ‫و ‪1; ‬‬ ‫‪J   2; ‬‬ ‫‪َ .2‬عزجس يجًوعبد األعداد انحقَقَخ ‪ J ، I‬و ‪ K‬حَث‪، I  3;3 :‬‬
‫أ ‪ /‬عجس عٍ انًجبل ‪ I‬ثصَغخ حصس ‪.‬‬
‫‪I‬‬ ‫‪K ،I‬‬ ‫‪K ، I‬‬ ‫ة ‪ /‬عٍَ انًجبالد االرَخ‪J ، I J :‬‬
‫‪2‬‬ ‫‪1‬‬ ‫‪1‬‬ ‫‪3‬‬
‫‪‬‬ ‫‪،‬‬ ‫‪x  3‬‬ ‫جـ ‪ x /‬عدد حقَقٌ يٍ انًجبل ‪ I‬انسبثق ثٍَ أٌ ‪:‬‬
‫‪x 3 3‬‬ ‫‪2‬‬ ‫‪2‬‬
‫التمرين الثالث( ‪ 05‬نقاط )‪:‬‬
‫انًُحٌُ ‪ًٍC‬ثم اندانخ ‪ f‬انًسزقَى ‪ًٍ D‬ثم اندانخ ‪g‬‬
‫ثقساءح ثَبََخ أجت عٍ األسئهخ‪:‬‬
‫‪ .1‬أوجد يجًوعخ رعسٍف اندانخ ‪.f‬‬
‫‪ .2‬أوجد صوز األعداد ‪ 4 ، -‬و ‪ 8‬ثبندانخ ‪.f‬‬
‫‪ .3‬أوجد سبثقزٌ انعددٍٍ ‪ 2‬و ‪ -‬ثبندانخ ‪.f‬‬
‫‪ .4‬حم انًعبدنزٍَ ‪ f (x )  0‬و ‪f (x )  6‬‬
‫‪ .5‬حم انًزساجحخ ‪f (x )  2 :‬‬
‫‪ .6‬حم انًعبدنخ ) ‪ f (x )  g (x‬ثى ) ‪f (x )  g (x‬‬
‫‪ .7‬شكم جدول رغَساد اندانخ ‪. f‬‬
‫‪ .8‬حدد إشبزح ) ‪ f (x‬فٌ انًجبل ‪.  3;9 :‬‬
‫‪ .9‬إذا عهًذ أٌ اندانخ ‪ g‬دانخ رآنفَخ ‪ .‬أوجد دسزوز اندانخ ‪g‬‬

‫بالتىفيق‬ ‫صفحة ‪ 0‬من ‪0‬‬ ‫انتهى‬


‫االمتحان األول‬
‫السهة الدراسية ‪2403/2419 :‬هـ املوافقة لـ ‪ 1339 /1338‬م‬
‫‪1338‬‬ ‫‪03‬‬ ‫‪2419‬هـ‬ ‫‪31‬‬ ‫السهة األوىل ثانوي‬
‫املدة ‪:‬ساعتان‬ ‫الشعبة‪ :‬علوم وتكهولوجيا‬
‫مادة الرياضيات‬
‫التمرين األول ) ‪ 30‬نقاط )‪:‬‬
‫أذكر إٌ كبَت انجًم انتبنٍت صحٍحت أو خبطئت يع انتعهٍم‬
‫إنى ‪ -‬هو انعدد‬ ‫‪ )2‬يدور انعدد‬
‫‪-‬‬
‫×‬ ‫هو انعدد‬ ‫×‬ ‫×‬ ‫×‬ ‫‪ )1‬رتبت يقدار انعدد‬
‫‪393‬‬
‫‪ ,‬هو انكسر‬ ‫‪ )0‬انعدد انُبطق … …‬
‫‪99‬‬
‫‪231‬‬
‫هو عدد غٍر عشري‬ ‫‪ )4‬انكسر‬
‫‪4220‬‬
‫‪727‬‬
‫هو عدد عشري‬ ‫‪ )0‬انكسر‬
‫‪99‬‬

‫التمرين الثاني) ‪ 34‬نقاط )‬


‫يجًوعت األعداد انحقٍقٍت ( ببستعًبل انًسبفت) ‪:‬‬ ‫حم فً‬
‫‪5‬‬
‫‪;  x +2,5  ‬‬ ‫‪ x –  + x + 3‬‬ ‫;‬ ‫= ‪x+ ‬‬
‫‪2‬‬

‫نقاط )‪:‬‬ ‫التمرين الثالث)‬


‫بسط يب ٌهً بدوٌ استعًبل انحبسبت ‪A  4 1875  9 147  12 18 243‬‬
‫‪ 3   3 2  1‬‬
‫‪4‬‬
‫‪75‬‬ ‫‪300‬‬ ‫‪363‬‬
‫‪  3   3 ‬‬ ‫‪  ‬‬ ‫‪B‬‬ ‫‪‬‬ ‫‪‬‬
‫‪4‬‬ ‫‪5‬‬
‫‪C‬‬
‫‪ 3 ‬‬
‫‪6‬‬
‫‪98‬‬ ‫‪98‬‬ ‫‪338‬‬

‫التمرين الرابع) ‪ 30‬نقاط )‬


‫‪ ABC‬يثهث قبئى فً ‪C‬‬
‫'‪ A‬و '‪َ C‬قطتبٌ يٍ انًستقًٍٍٍ ) ‪ ( AB‬و ) ‪ (BC‬عهى انترتٍب بحٍث ) ‪ٌ ( AC‬وازي‬
‫) '‪ ( A'C‬أوجد حصر نـ ‪ BC‬بوضع ‪ AB = c‬و ‪ AC = b‬و ‪BC = a‬‬
‫عهًب أٌ ‪ 5,12  c  5,11‬و ‪4.01  b  3.99‬‬
‫بوضع '‪ A'C' = a‬و '‪ AA' = c‬وعهًب أٌ ‪ 2,84  a '  2,83 :‬فأوجد حصرا نـ '‪c‬‬
‫أستاذ املادة ‪:‬‬ ‫( يسبعدة ‪ًٌ :‬كُك استعًبل َظرٌت طبنس )‬

‫‪1 1‬‬
‫‪1211 1212‬‬ ‫‪1341 1341‬‬
‫انًسخىي‪ :‬األونً ثاَىٌ جرع يشخسك عهىو وحكُىنىجُا‪.‬‬
‫انًدة ‪ :‬ساعخاٌ‬ ‫اخخباز فٍ يادة ‪ :‬انسَاضُاث‬
‫التمرين األول ( ‪ 04‬نقط ) ‪:‬‬

‫‪2 5‬‬ ‫‪3  20 1  3 5‬‬


‫‪B‬‬ ‫‪,‬‬ ‫‪A‬‬ ‫‪‬‬ ‫َ‪ A‬و ‪ B‬عدداٌ حقُقُاٌ حُث ‪:‬‬
‫‪2 5‬‬ ‫‪1 5‬‬ ‫‪1 5‬‬

‫‪B  9  4 5‬‬ ‫‪A‬و‬


‫‪3‬‬
‫‪4‬‬
‫‪‬‬ ‫‪‬‬
‫‪ .1‬أثبج أٌ ‪1  3 5 :‬‬

‫‪ .2‬احسب ‪ A – B :‬ياذا حسخُخج ؟‬

‫التمرين الثاني (‪ 23‬نقط )‪:‬‬

‫نُكٍ ‪ a‬و ‪ b‬عددٍَ حقُقٍُُ بحُث ‪:‬‬


‫‪1‬‬ ‫‪1‬‬
‫‪ 0.75  a  0.80‬و ‪  b ‬‬
‫‪2‬‬ ‫‪4‬‬
‫‪ .1‬أوجد حصسا نـ ‪ 4b  5 :‬و ‪. 1  a‬‬
‫‪1‬‬ ‫‪1 a‬‬ ‫‪1‬‬
‫‪‬‬ ‫‪ .2‬بٍُ أٌ ‪ :‬‬
‫‪35 4b  5 16‬‬

‫التمرين الثالث ( ‪ 21‬نقط )‪:‬‬


‫نُكٍ ‪E = x² + 2 x :‬‬
‫‪ ‬احسب قًُت ‪ E‬يٍ أجم ‪.x =2,5 ×102‬‬
‫‪ ‬جد زحبت يقداز انقًُت انًحصّم عهُها‪.‬‬

‫التمرين الرابع ( ‪ 05‬نقط )‪:‬‬


‫‪ .1‬حهم انعددٍَ ‪ 45‬و ‪ 105‬إنً جداء عىايم أونُت‪.‬‬
‫‪ .2‬احسب انقاسى انًشخسك األكبس نهعددٍَ ‪ 45‬و ‪.105‬‬
‫‪45‬‬
‫‪ .3‬اخخزل‬
‫‪105‬‬
‫‪ .4‬اسخُخج انخحهُم إنً جداء عىايم أونُت نكم يٍ ‪ 105 × 45‬و ‪ 454‬و ‪.1053‬‬
‫‪ A .5‬عدد َاطق كخابخه انعشسَت اندوزَت هٍ ‪A = 0,321212121……….. :‬‬
‫جد انكخابت انكسسَت نهعدد ‪. A‬‬

‫‪1‬‬ ‫‪1‬‬
‫التمرين الخامس ( ‪ 20‬نقط )‪:‬‬
‫َ‪ a‬و ‪ b‬عدداٌ حقُقُاٌ يىجباٌ حًايا ‪َ :‬ضع ‪:‬‬
‫‪2‬‬ ‫‪a b‬‬
‫‪H ‬‬ ‫‪; G  ab‬‬ ‫;‬ ‫‪A‬‬
‫‪1 1‬‬ ‫‪2‬‬
‫‪‬‬
‫‪a b‬‬
‫‪‬‬ ‫‪‬‬
‫‪2‬‬
‫‪a b‬‬ ‫‪ .1‬أَشس‬

‫‪ .2‬أحسب ‪ A – G‬ثى اسخُخج أٌ ‪.A  G‬‬


‫‪ab‬‬
‫‪‬‬ ‫‪‬‬
‫‪2‬‬
‫‪ G  H ‬ثى اسخُخج أٌ ‪G  H :‬‬ ‫‪a b‬‬ ‫‪ .3‬أثبج أٌ ‪:‬‬
‫‪a b‬‬
‫‪ .4‬يًا سبق اسخُخج يقازَت بٍُ األعداد ‪ B ; A‬و ‪.G‬‬

‫‪1‬‬ ‫‪1‬‬
‫و‬ ‫‪/‬‬ ‫هـ‪//‬‬ ‫‪/‬‬ ‫انسُة اندراسَة ‪:‬‬ ‫االيتحاٌ انفصهٌ األول‬
‫انًسخىي‪ :‬األونً ثاَىٌ جرع يشخسك عهىو وحكُىنىجُا‪.‬‬
‫انًدة ‪ :‬ساعخاٌ‬ ‫اخخباز فٍ يادة ‪ :‬انسَاضُاث‬
‫َقط ) ‪:‬‬ ‫انتًزٍٍ األول (‬
‫‪x y‬‬ ‫‪1 x 2‬‬
‫‪B ‬‬ ‫‪A‬و‬ ‫‪ 1  x  4‬و ‪َ 3  y  8‬ضع ‪:‬‬ ‫‪ x‬و ‪ y‬عدداٌ حقُقُاٌ حُث ‪:‬‬
‫‪1  x .y‬‬ ‫‪2‬‬
‫‪13‬‬ ‫‪7‬‬
‫‪ ، y ‬ثى أوجد حصسا نهعدد ‪.A‬‬ ‫‪x ‬و‬ ‫‪ .1‬احسب قًُت ‪ B‬يٍ أجم‬
‫‪3‬‬ ‫‪5‬‬
‫‪َ .2‬فسض أٌ ‪ x  3  5‬و ‪. y  3  5‬‬
‫أ ‪ /‬احسب انًجًىع ‪ x 2  y 2 :‬ثى انجداء ‪. x .y‬‬
‫ب ‪ /‬استنتج قيمة مبسطة للمجموع ‪ x  y‬ثى ححقق أٌ ‪3B  10 :‬‬
‫انتًزٍٍ انثاٌَ ( ‪َ 50‬قط )‪:‬‬

‫√‬ ‫√ √‬ ‫√‬
‫√(‬ ‫)‬ ‫√(‬ ‫)‬ ‫√(‬ ‫)‬
‫]‬ ‫]‬ ‫|‬ ‫|‬ ‫| |‬
‫‪x, y‬‬

‫َقط )‪:‬‬ ‫انتًزٍٍ انثانث (‬


‫بانشكم‪. f (x )  x 2  2x  3 :‬‬ ‫انجزء األول ‪ f :‬دانت يعسفت عهً‬
‫‪ .1‬احسب صىز األعداد ‪ - ، - ، 3‬و ‪ 4‬باندانت ‪. f‬‬
‫‪ .2‬احسب سىابق انعدد ‪ -‬ثى اسخُخج يًا سبق سىابق انعدد ‪ 5‬باندانت ‪. f‬‬
‫‪ .3‬ححقق أَه يٍ أجم كم عدد حقُقٍ ‪ x‬فإٌ ‪f (x )  (x  1)2  4 :‬‬
‫‪ .4‬أثبج أٌ اندانت ‪f‬يخزاَدة حًايا عهً انًجال ‪ 1; ‬ويخُاقصت حًايا عهً انًجال ‪;1‬‬
‫انجزء انثاٌَ‪ :‬يٍ أجم يعسفت َقط حقاطع ) ‪ (C f‬فإَُا َحم انًعادنت ‪ f (x )  0‬نكٍ سىف َكىٌ انحم‬
‫بـ ‪ g (x )  x 2 :‬ونُكٍ ) ‪ (C g‬حًثُهها انبُاٍَ فٍ يعهى يخعايد‬ ‫بُاَُا‪ .‬نخكٍ اندانت ‪ g‬انًعسفت عهً‬
‫ويخجاَس ) ‪ (O ; i , j‬انشكم يوجود فٌ انوثَقة انًزفقة ‪.‬‬
‫‪ .1‬حدد يٍ انبُاٌ شفعُت اندانت ‪ g‬يع انخبسَس‪.‬‬
‫‪ .2‬نخكٍ اندانت ‪ h‬اندانت انخآنفُت ذاث يعايم انخىجُه ‪ 2‬حًثُهها انبُاٍَ ) ‪َ (C h‬شًم انُقطت )‪A (1;1‬‬
‫أكخب عبازة اندانت ‪. h‬‬
‫‪ .3‬ححقق أٌ يٍ أٌ ) ‪. g (x )  h (x )  f (x‬‬
‫‪ .4‬أَشئ ) ‪ (C h‬فٍ انىثُقت انًسفقت‪ .‬ثى حم بُاَُا انًعادنت‪ . g (x )  h (x ) :‬ياذا حسخُخج بانُسبت نـ ) ‪(C f‬‬
‫‪ .5‬حدد يٍ انبُاٌ انىضع انُسبٍ نكم يٍ ) ‪ (C h‬و ) ‪ (C g‬ياذا حسخُخج‬
‫بانُسبت نهىضع انُسبٍ نـ ) ‪ (C f‬بانُسبت نًحىز انفىاصم‪.‬‬
‫‪.‬‬
‫صفحة ‪ 1‬يٍ ‪2‬‬
‫تزجع هذِ انوثَقة يع ورقة اإلجابة ‪:‬‬

‫االسى انكايم ‪.....................................................................................:‬‬


‫انًستوى ‪ :‬األوني ثاَوً جذع يشتزك عهوو وتكُونوجَا ‪............................‬‬

‫صفحة ‪ 2‬يٍ ‪2‬‬


‫م‬ ‫‪/‬‬ ‫هـ‪//‬‬ ‫‪/‬‬ ‫السىة الدراسية ‪:‬‬ ‫االمتحان الفصلي األول‬
‫األربعاء ‪ 00‬صفر ‪ 11//1435‬ديسمبر ‪2013‬‬ ‫انًسرىي‪ :‬األونً ثاَىٌ خرع يشرسك عهىو وذكُىنىخُا‪.‬‬
‫انًدج ‪ :‬ساعراٌ‬ ‫اخرثاز فٍ يادج ‪ :‬انسَاضُاخ‬
‫التمريه األول ( ‪ 05‬وقط ) ‪:‬‬
‫‪ )1‬نُكٍ ‪ a‬و ‪ b‬عددٍَ حقُقٍُُ غُس يعدويٍُ‪.‬‬
‫‪3 1‬‬
‫‪(12a b ) .( 3ab )3‬‬
‫‪2‬‬
‫‪A  2‬‬ ‫تسط انعدد انحقُقٍ ‪ A‬حُث ‪:‬‬
‫‪0, 25.a‬‬
‫‪ )2‬نُكٍ انعدد انحقُقٍ ‪ B‬حُث ‪B  2  3 48  2 27  147 :‬‬
‫أ ‪ /‬تٍُ أٌ ‪. B  2  3 :‬‬
‫ب ‪ /‬احسة اندداء‪ . A  B :‬ياذا ًَكٍ انقىل عٍ انعددٍَ ‪ A‬و‪. B‬‬
‫‪1 1‬‬
‫‪‬‬
‫ج ‪ /‬اسرُرح قًُح ‪A B :‬‬
‫‪ )3‬نُكٍ انعدد انحقُقٍ ‪ C‬حُث‪C  5  2  2  3  2  3   :‬‬
‫أكرة انعدد انحقُقٍ ‪ C‬تدوٌ زيز انقًُح انًطهقح ثى تسطه‪.‬‬
‫التمريه الثاوي ( ‪ 05‬وقط )‪:‬‬
‫‪ a 3 3‬و ‪b 2 7‬‬ ‫‪ )1‬أ ‪ a /‬و ‪ b‬عدداٌ حقُقُاٌ حُث‪:‬‬
‫‪1‬‬
‫ب ‪ /‬ثى اسرُرح يقازَح تٍُ انعددٍَ ‪ a‬و ‪.b‬‬ ‫‪a b ‬‬ ‫تٍُ أٌ‪:‬‬
‫‪3 32 7‬‬
‫‪‬‬ ‫‪‬‬
‫‪2‬‬
‫ب ‪ /‬اسرُرح كراتح يثسطح نهعدد ‪ x‬حُث‪x  55  12 21 :‬‬ ‫‪. 3 32 7‬‬ ‫‪ )2‬أ ‪ /‬تسط انعدد‬
‫‪ )3‬إذا عهًد أٌ ‪ 2,6  7  2,7 :‬و ‪ ، 1,7  3  1,8‬اعط حصسا نهعدد ‪x‬‬
‫التمريه الثالث (‪ 03‬وقط )‪:‬‬
‫)‪ (‬يسرقُى يزود تانًعهى ) ‪(O ; i‬‬
‫‪ )1‬عهى انُقطرٍُ ‪ A‬و ‪ B‬ذاخ انفاصهرٍُ ‪ 3‬و ‪ 5‬عهً انرسذُة وانُقطح ‪ G‬يُرصف انقطعح‬
‫‪َ M ،  AB ‬قطح يرحسكح يٍ انًعهى انساتق فاصهرها ‪.x‬‬
‫‪ )2‬عٍُ فٍ كم حانح يٍ انحاالخ انرانُح انعدد انحقُقٍ ‪: x‬‬
‫ب ‪ ، x 3  x 5  6 /‬ج ‪x 3  x 5 /‬‬ ‫أ ‪، x 3  4 /‬‬
‫التمريه الرابع( ‪ 00‬وقط)‬
‫‪ f‬دانح يعسفح عهً تًا َهٍ‪f (x )  2x ²  7x :‬‬
‫) ‪ (C f‬ذًثُهها انثُاٍَ فٍ يعهى يرعايد ) ‪ (O ; i , j‬الصفحة‬
‫الجزء األول‪:‬‬
‫تقساءج تُاَُح أوخد يا َهٍ‪:‬‬
‫صىزج كم يٍ األعداد ‪0، (3,5) ، 2‬‬ ‫‪.1‬‬
‫هم نهعدد ‪ 0‬سىاتق ؟ حددها اذا كاَد االخاتح تُعى‬ ‫‪.2‬‬
‫حم تُاَُا انًعادنح ‪ . 2x ²  7x  22‬ثى انًرساخحح ‪f (x )  0 :‬‬ ‫‪.3‬‬
‫شكم خدول ذغُساخ اندانح ‪.f‬‬ ‫‪.4‬‬
‫اقلب الصفة‬ ‫صفحة ‪ 1‬مه ‪2‬‬
‫الجزء الثاوي‪:‬‬
‫تًا َهٍ‪g (x )  3x  4 :‬‬ ‫َعرثس اندانح ‪ g‬انعسفح عهً‬
‫‪ .1‬ياهٍ صىزج انعدد )‪ (3‬تاندانح ‪g‬‬
‫‪ .2‬حدد اٌ أيكٍ ساتقح او سىاتق انعدد ‪ 5‬تاندانح ‪g‬‬
‫‪ .3‬تٍُ اٌ اندانح ‪ g‬يرُاقصح ذًايا عهً ‪.‬‬

‫اوتهى‬ ‫صفحة ‪ 2‬مه ‪2‬‬


‫التصحيح الىمىذجي‬
05 ‫ التمريه الثاوي‬05 ‫التمريه االول‬
01 / ‫) أ‬1 01 1 2 3
12 a b  3 3a 3b 3
A  2
a b  3 3  2 7 a
:‫تانضسب فٍ انًسافق َدد‬ 4

a b 
 3 32 7 3 32 7   1 2  31 33
A  2  (4  3) a b 4 3
3 32 7 A  2 3 )1
1

3 32 7
01 /‫) ب‬1 01 B  2  3 48  2 27  147
a  b : ٌ‫ فإ‬a  b  0 : ٌ‫تًا أ‬
B  2  12 3  6 3  7 3
B  2 3 / ‫) أ‬2
/ ‫) أ‬2 01 / ‫) ب‬2
3 32 7 
2
 27  28  12 21 
A B  2 3 2 3  
 55  12 21 1
01 :x ‫ اسرُراج قًُح يثسطح نهعدد‬/ ‫) ب‬2 / ‫) ب‬2
3  1 1
2
x  55  12 21  32 7 B 
A
‫ أو‬A  B :ٌ‫ فإ‬A  B  1:ٌ‫إذا كا‬
 3 3 2 7  2 7 3 3 .‫أحدهًا يقهىب االخس‬

01 1,7  3  1,8 ‫ و‬2,6  7  2,7 ) 3 :ٌ‫ تاسرعًال انُرُدح انساتقح فإ‬/ ‫) ج‬2
: x ‫إعطاء حصس نهعدد‬
1 1
  B A  2 3  2 3
A B
   
5, 2  2 7  5, 4
 2 3
5,1  3 3  5, 4
)3
5, 4  3 3  5,1
C  5  2  2  3  2  3  
0, 2  2 7  3 3  0, 4
C  2  5  2  3  2  3  
0  x  0, 4 : ٌ‫ فإ‬x  0 ٌ‫تًا أ‬   1

03 :‫التمريه الثالث‬
01 x 3  x 5  6 )1
d (x ; 3)  d (x ;5)  6
S  / ‫) ب‬2
01 x 3  x 5 x 3  4
d (x ; 3)  d (x ;5) d (x ; 3)  4
S  1;  / ‫) ج‬2 S  1; 7
/ ‫) أ‬2
‫‪00‬‬ ‫التمريه الرابع‬
‫‪01‬‬ ‫‪ . 2‬نهعدد ‪ 0‬ساتقراٌ وهًا ‪ 0 :‬و ‪3,5‬‬ ‫الجزء األول‪:‬‬
‫‪.1‬‬
‫‪f (2)  22‬‬
‫‪f (3,5)  f (0)  0‬‬
‫‪01‬‬ ‫‪ .4 01‬خدول انرغُساخ‪:‬‬ ‫‪.3‬‬
‫المعادلة ‪ 2x ²  7x  22‬حالٌ هًا‪:‬‬
‫‪S  2; 5,5‬‬
‫‪01‬‬ ‫‪ .3‬اندانح ‪ g‬يرُاقصح ذًايا عهً ألَه يٍ‬ ‫‪. 3‬حم انًرساخحح ‪f (x )  0 :‬‬
‫اخم كم عددٍَ حقُقٍُ ‪ a‬و‪ b‬حُث‪:‬‬ ‫‪S  3,5;0‬‬
‫‪a b‬‬
‫‪3a  3b‬‬ ‫الجزء الثاوي‬
‫‪.1‬‬
‫‪3a  4  3b  4‬‬ ‫‪g (3)  13‬‬
‫) ‪g (a )  g (b‬‬ ‫‪.2‬‬
‫نى َحافظ عهً انرسذُة فاندانح ‪ g‬يرُاقصح‬ ‫‪1‬‬
‫‪‬‬
‫عهً ‪.‬‬ ‫ساتقح انعدد ‪ 5‬هٍ ‪3‬‬
‫السىة الدراسية ‪ 1436/1435 :‬هـ‪ 2015/2014 //‬م‬ ‫االمتحان الفصلي األول‬
‫انًسرىي‪ :‬األونً ثاَىٌ خزع يشرشك عهىو وذكُىنىخُا‪.‬‬
‫انًذج ‪ :‬ساعراٌ‬ ‫اخرثاس فٍ يادج ‪ :‬انشَاضُاخ‬
‫التمريه األول ( ‪ 04‬وقط ) ‪:‬‬
‫نكم سؤال إخاتح واحذج صحُحح يٍ تٍُ اإلخاتاخ انرانُح فأخرش انصحُحح يُها يع انرعهُم‪:‬‬
‫‪ /1‬إرا كاٌ ‪ 2  x  2‬فئٌ‪:‬‬
‫ج‪d (x ;0)  2 /‬‬ ‫ب‪d (x ;0)  4 /‬‬ ‫أ‪x  2  1 /‬‬
‫سطح نهعذد ‪ a‬هٍ‪:‬‬ ‫‪ a  (33.34 ).(53 )2 .55 /2‬انقًُح انًث ّ‬
‫ج‪a  31.5 /‬‬ ‫ب‪a  32.5 /‬‬ ‫أ‪a  32.5 /‬‬
‫‪ ‬فئٌ ‪:‬‬
‫‪2; 1‬‬
‫عذدا يٍ انًدال‬ ‫‪x‬‬ ‫‪ /3‬إرا كاٌ‬
‫‪1 1 ‬‬
‫‪ ;1‬‬
‫‪x  1; 2‬‬ ‫‪x  2 ‬‬ ‫‪x 2  1; 4‬‬
‫ج‪/‬‬ ‫ب‪/‬‬ ‫أ‪/‬‬
‫‪ x‬فئٌ ‪:‬‬ ‫‪ /4‬إرا كاٌ ‪1‬‬
‫‪1‬‬
‫‪x‬‬
‫‪. x‬‬ ‫ج‪x /‬‬ ‫ب‪x /‬‬ ‫‪x2‬‬ ‫أ‪x /‬‬
‫التمريه الثاوي ( ‪ 03‬وقط )‪:‬‬

‫التمريه الثالث (‪ 06‬وقط )‪:‬‬


‫‪ -1‬ذشخى انعالقاخ اِذُح فٍ عثاسج يسافح ‪ ،‬ثى يث ّم انىضعُح عهً يسرقُى عذدٌ ثى اسرخهص انحم ‪:‬‬
‫ج‪x 2 x 1 /‬‬ ‫ب ‪3x 3 6 /‬‬ ‫أ ‪x5  6 /‬‬
‫‪4x  3 y‬‬
‫‪ , y  1 ; 2‬عٍُ حصشا نـ ‪:‬‬ ‫‪ x  3 ; 5‬و‬ ‫‪ -2‬تفشض‬
‫‪ 3x  4 y‬‬
‫التمريه الرابع( ‪ 00‬وقط)‬
‫الجزء األول‪:‬‬
‫نركٍ انذانح انعذدَح ‪ f‬انًعشفح تـ‪. f (x ) x ² 4x 3 :‬‬
‫‪ .1‬أوخذ أكثش يدًىعح نهرعشَف نهذانح ‪f‬‬
‫‪ .2‬احسة صىس األعذاد ‪ 2 ، 1 ، 3‬تانذانح ‪.f‬‬
‫‪ .3‬أحسة انسىاتق انًًكُح نهعذد ‪ 3‬تانذانح ‪.f‬‬

‫اقلب الصفة‬ ‫صفحة ‪ 1‬مه ‪2‬‬


‫الجزء الثاوي‪:‬‬
‫نركٍ انذانح ‪ g‬انًعشفح ترًثُهها انثُاٍَ ( انشكم انًقاتم)‬
‫تقشاءج تُاَُح أوخذ يا َهٍ‪:‬‬
‫‪y‬‬
‫‪ .1‬يدًىعح ذعشَف انذانح ‪g‬‬
‫‪1‬‬
‫‪ .2‬صىسج كم يٍ األعذاد ‪0، (1) ، 2‬‬
‫‪ .3‬هم نهعذد ‪ 0‬سىاتق ؟ حذدها ارا كاَد االخاتح تُعى‬
‫‪ .4‬اسرُرح اذداِ ذغُّش انذانح ‪g‬عهً انًدال ‪.  0; 2 ‬‬
‫‪-3‬‬ ‫‪-2‬‬ ‫‪-1‬‬ ‫‪0‬‬ ‫‪1‬‬ ‫‪2‬‬ ‫‪x‬‬

‫‪-1‬‬

‫‪-2‬‬

‫‪-3‬‬

‫‪-4‬‬

‫‪-5‬‬

‫‪-6‬‬

‫‪-7‬‬

‫‪-8‬‬

‫اوتهى‬ ‫صفحة ‪ 2‬مه ‪2‬‬


‫السىة الدراسية ‪ 1437/1436 :‬هـ‪ 2016/2015 //‬م‬ ‫االمتحان الفصلي األول‬
‫األزد ‪ 11‬صفس ‪َ 22 // 1431‬ىفًثس ‪2015‬‬ ‫انًستىي‪ :‬األونً ثاَىٌ خرع يشتسك عهىو وتكُىنىخُا‪.‬‬
‫انًدج ‪ :‬ساعتاٌ‬ ‫اختثاز فٍ يادج ‪ :‬انسَاضُاخ‬
‫التمريه األول ( ‪ 04‬وقط ) ‪:‬‬
‫فٍ االقتسازاخ انثالثح األونً أخة إيا صسُر أو خاطئ يع انتعهُم وأيا فٍ االقتساذ انساتع فاختس اإلخاتح‬
‫انصسُسح يع انتعهُم دائًا‪:‬‬
‫‪1‬‬ ‫‪1‬‬ ‫‪1‬‬
‫هٍ‬ ‫‪‬‬ ‫‪‬‬ ‫‪ .1‬أصغس يدًىعح َُتًٍ إنُها انعدد‬
‫‪2 1‬‬ ‫‪32‬‬ ‫‪2 3‬‬
‫‪38‬‬
‫هٍ‬ ‫‪A  0, 038‬‬ ‫‪ .2‬انكتاتح انكسسَح نهعدد‬
‫‪99‬‬
‫‪ .3‬زتثح يقداز انعدد ‪ 12000  0,005 103‬هٍ ‪5 103‬‬
‫‪ a .4‬و ‪ b‬عدداٌ زقُقُاٌ زُث ‪ a  b  0 :‬إذٌ‪:‬‬
‫ج ‪ a0 /‬و ‪b 0‬‬ ‫ب ‪a b  0 /‬‬ ‫أ ‪a b  0 /‬‬
‫)‪ :‬نتكٍ انعثازج ‪ E‬كًا َهٍ‪E  1053  3 325  2 52 :‬‬ ‫التمريه الثاوي ( ‪ 04‬وقط‬
‫‪ / 1‬أكتة انعثازج ‪ E‬عهً شكم ‪ a b‬زُث ‪ a‬عدد صسُر و‪ b‬عدد صسُر أصغس يا ًَكٍ‬
‫‪‬‬ ‫‪‬‬
‫‪2‬‬
‫‪ 2  5‬ثى استُتح كتاتح يث ّ‬
‫سطح نهعدد ‪9  4 5‬‬ ‫‪ / 2‬أ ) ازسة‬
‫‪.‬‬ ‫ب)اكتة ‪ 9  4 5‬عهً شكم ) ‪ (x  y‬ثى استُتح كتاتح يث ّ‬
‫سطح نهعدد ‪9  4 5‬‬ ‫‪2‬‬

‫‪9  4 5 ‬‬ ‫‪94 5  94 5‬‬‫‪‬‬ ‫‪‬‬ ‫سط يا َهٍ‪9  4 5 :‬‬


‫ج)ت ّ‬
‫)‪ x:‬و ‪ y‬عدداٌ زقُقُاٌ زُث ‪5  y  9 ;  9  x  5‬‬ ‫التمريه الثالث (‪ 06‬وقط‬
‫‪K   6 ; 6‬‬ ‫‪; I  8 ; 8 ; J   10 ;  6‬‬ ‫زُث‪6 ; 10 :‬‬
‫‪ J ، I‬و ‪ K‬ثالثح يداالخ يٍ‬
‫‪ .1‬عٍُ انًداالخ انتانُح‪I K ; I K ; I J ; I J :‬‬
‫‪ .2‬عثس عٍ انسصس ‪ 9  x  5‬عهً شكم يدال ‪ ،‬ثى عهً شكم قًُح يطهقح‬
‫‪ .3‬زم انًعادنح ‪ x  7  2‬ثى استُتح زهىل انًتساخسح ‪x  7  2‬‬
‫‪1.7‬‬ ‫√‬ ‫و‬ ‫√‬ ‫‪ .4‬تفسض إٌ‬
‫√‬ ‫√‬ ‫√‬ ‫√‬ ‫عٍُ زصسا نهعددٍَ انسقُقٍُُ‬

‫تـ ‪f  x   x 2  2 x  3 :‬‬ ‫التمريه الرابع( ‪ 06‬وقط) َعتثس اندانح ‪ f‬انًعسفح عهً انًدال ‪ 6 ; 6‬‬
‫‪2‬‬ ‫‪,‬‬ ‫‪ .1‬ازسة صىز االعداد ‪1 , 1 ,  2‬‬
‫‪ .2‬عٍُ سىاتق انعدد )‪ (3‬إٌ أيكٍ‬
‫‪ .3‬تسقق اٌ ‪f  x    x  1  4‬‬
‫‪2‬‬

‫‪ .4‬تٍُ اٌ اندانح ‪ f‬يتزاَدج عهً انًدال ‪1 ; 6‬‬


‫‪ 6 ; 1‬‬
‫ويتُاقصح عهً انًدال‬
‫‪ .5‬شكم خدول تغُساخ اندانح ‪ f‬عهً انًدال‬
‫‪ 6 ; 6‬‬
‫اوتهى‬ ‫صفحة ‪ 1‬مه ‪1‬‬
‫السنة الدراسية ‪ 1438/1437 :‬هـ‪ 2017/2016 //‬م‬ ‫االمتحان الفصلي األول‬
‫االثنين ‪ 05‬ربيع األول ‪ 05 // 1438‬ديسمبر ‪2016‬‬ ‫المستوى‪ :‬األولى ثانوي جذع مشترك علوم وتكنولوجيا‬
‫المدة ‪ :‬ساعتان‬ ‫اختبار في مادة ‪ :‬الرياضيات‬
‫التمرين األول ( ‪ 04‬نقط ) ‪:‬‬
‫أذكر صحة أو خطأ العبارات التالية مع التعليل مع تصحيح الخطأ ما أمكنك‪:‬‬
‫‪( 3 3)( 3‬‬ ‫)‪3‬‬
‫هو عدد عشري‪.‬‬ ‫‪300‬‬
‫‪)1‬‬
‫‪.‬‬ ‫‪2; 2‬‬ ‫;‪2‬‬ ‫‪2; 2‬‬ ‫‪)2‬‬
‫‪2‬‬
‫‪.‬‬ ‫‪(3‬‬ ‫‪x‬‬ ‫)‪1‬‬ ‫‪16‬‬
‫‪ x )3‬عدد حقيقي حيث ‪ x 1‬يكافئ‬
‫)‪. f (a‬‬ ‫)‪f (b‬‬ ‫‪ )4‬إذا كانت ‪ f‬متناقصة تماما على مجال ‪ a; b‬فإن‬
‫)‪ f ( x‬يشمل النقطة )‪. A(1; 6‬‬ ‫‪3x 2‬‬ ‫‪5x 4‬‬ ‫‪ )5‬التمثيل البياني للدالة ‪ f‬المعرفة بـ‪:‬‬
‫التمرين الثاني ( ‪ 04‬نقط )‪:‬‬
‫‪y‬‬ ‫‪J‬‬ ‫‪ J‬بحيث‪ x I :‬و‬ ‫‪2;2‬‬ ‫‪I‬و‬ ‫نعتبر ‪ x‬و ‪ y‬عددين حقيقيين والمجالين ‪1;4‬‬
‫‪ .1‬عين ‪ I J‬و ‪. I J‬‬
‫‪x‬‬ ‫‪y‬‬ ‫‪8‬‬
‫‪xy‬‬
‫و‬ ‫‪xy‬‬
‫‪x‬‬
‫‪ .2‬اعط حصرا لما يلي‪، y 2 x :‬‬
‫‪1 1‬‬
‫‪m‬‬
‫‪2 3‬‬
‫‪ .3‬عين قيمة ‪ m‬بحيث ‪:‬‬
‫‪1‬‬
‫‪ .4‬نعتبر ‪ t‬من مجال مركزه ‪ c 2‬و نصف قطره ‪ r 2‬أوجد حصرا للعدد ‪. t‬‬

‫التمرين الثالث (‪ 03‬نقط )‪:‬‬


‫‪X‬‬ ‫‪3 2 2‬‬ ‫‪ X‬عدد حقيقي حيث‪3 2 2 :‬‬
‫‪ .1‬ادرس إشارة العدد ‪.X‬‬
‫‪ .2‬احسب ‪. X 2‬‬
‫‪ .3‬استنتج القيمة المضبوطة للعدد ‪. X‬‬
‫التمرين الرابع( ‪ 09‬نقط)‬
‫الجزء األول‪:‬‬
‫نعتبر الدالة العددية ‪ f‬المعرفة على ‪ 3;5‬الممثلة بيانيا بالمنحني ) ‪ ( (C f‬الوثيقة المرفقة )‬
‫بقراءة بيانية‪:‬أوجد‪:‬‬
‫‪ .1‬صور األعداد ‪ 0‬و ‪ 2‬بالدالة ‪.f‬‬
‫‪ .2‬السوابق الممكنة للعددين ‪ 0‬و ‪ 2‬بالدالة ‪.f‬‬
‫‪ .3‬حلول المعادلة ‪f ( x) 3‬‬

‫اقلب الصفحة‬ ‫صفحة ‪ 1‬من ‪3‬‬


‫‪ .4‬لتكن الدالة ‪ g‬المعرفة بـ‪. g ( x) 2 x 1 :‬‬
‫أ ‪ /‬ارسم التمثيل البياني ) ‪ (C g‬للدالة ‪ g‬في الوثيقة المرفقة‪.‬‬
‫ب ‪ /‬اوجد نقط تقاطع ) ‪ (C g‬و ) ‪. (C f‬‬
‫الجزء الثاني‪:‬‬
‫يعطى دستور الدالة ‪ f‬كما يلي‪f ( x) 4 ( x 1) :‬‬
‫‪2‬‬

‫‪ .1‬انشر وبسط عبارة )‪. f ( x‬‬


‫‪ .2‬حلل عبارة ) ‪ f ( x‬ثم بين أنه ‪f ( x) ( x 1)(3 x) :‬‬
‫‪. f‬‬ ‫‪ .3‬احسب )‪ f (2‬و ‪3‬‬
‫‪ .4‬استنتج مما سبق سوابق العدد ‪ 0‬بالدالة ‪. f‬‬

‫انتهى‬ ‫صفحة ‪ 2‬من ‪3‬‬


‫الوثيقة المرفقة‬
‫األولى جذع مشترك علوم وتكنولوجيا‬ ‫االسم الكامل‪:‬‬
‫ترجع مع وثيقة اإلجابة‬
‫السنة الدراسية ‪ 1439/1438 :‬هـ‪ 2018/2017 //‬م‬ ‫االمتحان الفصلي األول‬
‫األربعاء ‪ 17‬ربيع األول ‪ 06 // 1439‬ديسمبر ‪2017‬‬ ‫المستوى‪ :‬األولى ثانوي جذع مشترك علوم وتكنولوجيا‬
‫المدة ‪ :‬ساعتان‬ ‫اختبار في مادة ‪ :‬الرياضيات‬
‫التمرين األول ( ‪ 05‬نقط ) ‪:‬‬
‫‪22‬‬
‫‪.F‬‬ ‫‪18‬‬ ‫و ‪8‬‬
‫‪E‬‬ ‫‪2 27‬‬ ‫‪ ( I‬يعطى العددان ‪ E‬و ‪ F‬حيث‪3 75 3 48 :‬‬

‫‪ / 1‬أكتب ‪ E‬على الشكل‪ a 3 :‬و ‪ F‬على الشكل‪ b 2 :‬حيث ‪ a‬و‪ b‬عددان صحيحان‪.‬‬
‫‪1‬‬
‫‪.‬‬ ‫‪E‬‬ ‫‪F‬‬
‫‪ / 2‬بيّن أن ‪E F :‬‬
‫‪ / 3‬قارن بين ‪ E‬و ‪ ، F‬علّل‪.‬‬
‫‪، 1, 42‬‬ ‫‪ 1,74‬و ‪1, 41‬‬
‫‪2‬‬ ‫‪ / 4‬إذا علمت أن‪3 1,73 :‬‬
‫‪1 1‬‬
‫أوجد حصرا لكل من ‪ E+F ، E ، F:‬و ‪. E F‬‬
‫التمرين الثاني ( ‪ 04‬نقط )‪:‬‬
‫لكل سؤال من األسئلة التالية إجابة صحيحة واحدة فقط ‪ ،‬حددها مع التبرير( ال تقبل إجابة‬
‫بدون تبرير) ‪.‬‬
‫‪ ، a  0.02 ) 1‬الكتابة العلمية لــ ‪ a 5‬هي ‪3.2  1010 * 3.2  109 * 3.2 109 * :‬‬

‫* ‪. 3 107 * 3  10 7‬‬ ‫هي ‪3  10 6 * :‬‬ ‫‪ ) 2‬رتبة مقدار العدد‬


‫‪0.00278 104‬‬
‫‪ x ) 3‬عدد حقيقي ‪ 3x  2  0 ،‬يكافئ ‪:‬‬
‫‪‬‬ ‫‪2‬‬ ‫‪2‬‬ ‫‪‬‬
‫‪.‬‬ ‫‪x   ;  * x   ; 2 ‬‬ ‫*‬ ‫‪x   ;  ‬‬ ‫*‬
‫‪‬‬ ‫‪3‬‬ ‫‪‬‬ ‫‪3 ‬‬ ‫‪3‬‬ ‫‪‬‬
‫‪7‬‬
‫‪ ) 4‬المجال ‪  2;7 ‬نصف قطره‬
‫‪1‬‬ ‫‪9‬‬
‫* ‪. 2‬‬ ‫‪2‬‬
‫*‬ ‫‪2‬‬
‫‪*:‬‬

‫التمرين الثالث (‪ 03‬نقط )‪:‬‬


‫‪ I‬و ‪ J‬مجموعتان حيث‬
‫‪ I‬مجموعة األعداد الحقيقية ‪ x‬التي تحقق ‪x  2  4‬‬
‫‪ J‬مجموعة األعداد الحقيقية ‪ x‬التي تحقق ‪x  3  x  1‬‬
‫عين كال من ‪ I‬و ‪ J‬و ‪ I  J‬و ‪I  J‬‬

‫اقلب الصفحة‬ ‫صفحة ‪ 1‬من ‪2‬‬


‫التمرين الرابع( ‪ 08‬نقط)‬
‫لتكن ‪ f‬الدالة المعرفة بمنحناها البياني في‬
‫معلم ) ‪ (O ; i , j‬كما هو موضح في الشكل‬
‫المقابل ‪.‬‬
‫باستعمال المعلومات الواردة في الشكل‪،‬‬
‫أجب على األسئلة التالية‪:‬‬
‫‪ -1‬عيّن مجموعة تعريف الدالة ‪ f‬ولتكن ‪. D f‬‬
‫‪ -2‬ما هي صور )‪ 1،،0 (2‬و ‪ 5‬بالدالة ‪ f‬؟‬
‫‪ -3‬ما هي السوابق الممكنة لـ )‪ 3، ( 4‬و ‪5‬‬
‫بالدالة ‪ f‬؟‬
‫‪ -4‬ما هي القيمة الحدية الصغرى للدالة‬
‫‪ -5‬أعط جدول تغيرات ‪. f‬‬
‫برر‪.‬‬‫‪ -6‬ماذا تالحظ بالنسبة لشفعية الدالة ‪ّ f‬‬
‫ل في المجموعة ‪ D f‬المعادلتين ‪ f  x   0‬و ‪f  x   5‬‬ ‫‪ -7‬ح ّ‬
‫ح ّل المجموعة ‪ D f‬المتراجحتين ‪ f  x   0‬و ‪. f  x   3‬‬

‫انتهى‬ ‫صفحة ‪ 2‬من ‪2‬‬


‫صفحة فيلدز‬
‫يف الرياضيات‬
‫اةقات وألغاز‬
‫مت حتميل هذا امللف‬
‫سالت وأفالم‬
‫من برانمج‬
‫وفلسفة الرياضيات‬
‫اجلامع يف الرايضيات‬
‫وملفات تعليمية‬
‫اجلمهورية اجلسائرية الدميقراطية الشعبية‬
‫ثانوية اجلديدة غرب حاشي حببح‬ ‫مديرية الرتبية لوالية اجللفة‬ ‫وزارة الرتبية الوطنية‬
‫الفرض األول للفصل األول ‪‬نوفنرب ‪2212‬‬
‫املدة ‪ :‬ساعـ ــة‬ ‫مادة ‪ :‬الرياضيات‬ ‫املستوى ‪ 1 :‬ج م ع تك‪21‬‬

‫االسه‪.................:‬‬
‫‪‬التنرين األول‪ 20( :‬نقاط)‬
‫اللقب‪..................:‬‬
‫اكنل اجلدول املقابل ‪:‬‬

‫املشـ ــافة‬ ‫اجمل ـ ــال‬ ‫احلص ــر‬ ‫القينة املطلقة‬


‫‪x  5 1‬‬

‫‪ 5  x  2‬‬
‫‪x  1;5‬‬

‫‪d x;1  3‬‬


‫‪‬التنرين الثاني‪ 20( :‬نقاط)‬
‫‪ ‬املعادالت واملرتاجحات التالية ‪:‬‬ ‫املشتقيه مزود مبعله ‪ M ، OI ‬نقطة فاصلتها ‪ ، x‬حل يف‬
‫‪x2 2‬‬ ‫‪)3‬‬ ‫‪x3  x5‬‬ ‫‪)2‬‬ ‫‪x3 5‬‬ ‫‪)1‬‬

‫‪x3 5‬‬ ‫‪)1‬‬


‫‪0‬‬ ‫‪1‬‬
‫‪.......................................................................................................................................................‬‬
‫‪............................................................................................. .........................................................‬‬
‫‪.......................................................................................................................................................‬‬
‫‪........................................................................................................................................... ...........‬‬
‫‪x3  x5‬‬ ‫‪)2‬‬
‫‪0‬‬ ‫‪1‬‬
‫‪................................................................ .......................................................................................‬‬
‫‪.......................................................................................................................................................‬‬
‫‪...................................................... ........................................................ .........................................‬‬
‫‪.................................................................... ...................................................................................‬‬
‫‪x2 2‬‬ ‫‪)3‬‬
‫‪0‬‬ ‫‪1‬‬
‫‪.......................................................................................................................................................‬‬
‫‪................................................................................. ......................................................................‬‬
‫‪...................................................... .................................................................................................‬‬
‫‪................................................................... ........................................................... ........................‬‬

‫الصفحة ‪1/2‬‬
‫‪‬التنرين الثالث‪ 20( :‬نقاط)‬
‫‪ 1  y  7‬‬ ‫‪ x‬و ‪ y‬عددان حقيقان حيث ‪ 2  x  5 :‬و‬
‫‪2‬‬
‫‪ 2x 2 ‬‬ ‫‪، 5x  3 y ، x  y ، x  y‬‬ ‫اوجد حصر لكل من ‪:‬‬
‫‪y‬‬
‫اجلواب‪...........................................................................................................................................: 3‬‬
‫‪........................................................................................................................................ ...............‬‬
‫‪.......................................................................................................................................................‬‬
‫‪.......................................................................................................................................................‬‬
‫‪............................................................................. ..........................................................................‬‬
‫‪.......................................................................................................................................................‬‬
‫‪........................................................................................................................... ............................‬‬
‫‪.......................................................................................................................................................‬‬
‫‪.......................................................................................................................................................‬‬
‫‪.......................................................................................................................................................‬‬
‫‪.................................................. ....................................................................................................‬‬
‫‪.............................................................................................................. .........................................‬‬
‫‪.......................................................................................................................................................‬‬
‫‪.......................................................................................................................................................‬‬
‫‪................................................... ....................................................................................................‬‬
‫‪.......................................................................................................................................................‬‬
‫‪................................................................................................. ......................................................‬‬
‫‪.......................................................................................................................................................‬‬
‫‪............................................................................................................................................... ........‬‬
‫‪....................................... ................................................................................................................‬‬
‫‪.......................................................................................................................................................‬‬
‫‪.............................................................................................................................. .........................‬‬
‫‪.......................................................................................................................................................‬‬
‫‪.......................................................................................................................................................‬‬
‫‪................................................................... ....................................................................................‬‬
‫‪.......................................................................................................................................................‬‬
‫‪...................................................................................................... .................................................‬‬
‫‪......................................................................................................................................................‬‬
‫بالتوفيـق‬ ‫الصفحة ‪2/2‬‬
‫اجلمهورية اجلسائرية الدميقراطية الشعبية‬
‫ثانوية اجلديدة غرب حاشي حببح‬ ‫مديرية الرتبية لوالية اجللفة‬ ‫وزارة الرتبية الوطنية‬
‫تصحيح الفرض األول للفصل األول ‪‬نوفنرب ‪2102‬‬
‫املدة ‪ :‬ساعـ ــة‬ ‫مادة ‪ :‬الرياضيات‬ ‫املستوى ‪ 0 :‬ج م ع تك ‪14‬‬
‫‪‬التنرين األول‪ 16 ( :‬نقاط)‬
‫اكنل اجلدول املقابل ‪:‬‬
‫املسـ ــافة‬ ‫اجمل ـ ــال‬ ‫احلص ــر‬ ‫القينة املطلقة‬
‫‪d x;5  1‬‬ ‫‪x   6;4‬‬ ‫‪ 6  x  4‬‬ ‫‪x  5 1‬‬
‫‪‬‬ ‫‪7 3‬‬ ‫‪7 3‬‬
‫‪d  x;  ‬‬ ‫‪x   5;2‬‬ ‫‪ 5  x  2‬‬
‫‪x‬‬ ‫‪‬‬
‫‪0.5 12‬‬ ‫‪‬‬ ‫‪2 2‬‬ ‫‪2 2‬‬

‫‪d x;3  2‬‬ ‫‪x  1;5‬‬ ‫‪1 x  5‬‬ ‫‪x 3  2‬‬

‫‪d x;1  3‬‬ ‫‪x   2;4‬‬ ‫‪2 x4‬‬ ‫‪x 1  3‬‬

‫‪‬التنرين الثاني‪ 19 ( :‬نقاط)‬


‫‪ ‬املعادالت واملرتاجخات التالية ‪:‬‬ ‫املستقيه مزود مبعله ‪ M ، OI ‬نقطة فاصلتوا ‪ ، x‬حل يف‬
‫‪x2 2‬‬ ‫‪)3‬‬ ‫‪x3  x5‬‬ ‫‪)2‬‬ ‫‪x3 5‬‬ ‫‪)0‬‬
‫‪A‬‬
‫‪x3 5‬‬ ‫‪)0‬‬
‫‪3‬‬ ‫‪0‬‬ ‫‪1‬‬
‫‪x   3  5‬‬ ‫لدينا ‪ x  3  5 :‬تكافئ ‪:‬‬
‫‪03‬‬ ‫‪ M‬نقطة من املستقيه املدرج ذات الفاصلة ‪ x‬و لتكن النقطة ‪ A‬ذات الفاصلة ‪.  3‬‬
‫جند ‪ d x;3  5 :‬أي‪. MA  5 :‬‬
‫‪ MA  5‬هنا ‪S1   8;2 :‬‬ ‫توجد نقطتان فاصلتونا حتقق ‪:‬‬
‫‪B‬‬ ‫‪A‬‬
‫‪x3  x5‬‬ ‫‪)2‬‬
‫‪5‬‬ ‫‪0‬‬ ‫‪1‬‬
‫‪x   3  x   5‬‬ ‫لدينا ‪ x  3  x  5 :‬تكافئ ‪:‬‬
‫‪5‬‬ ‫‪ M‬نقطة من املستقيه املدرج ذات الفاصلة ‪ x‬و لتكن النقطة ‪ A‬ذات الفاصلة ‪  3‬و النقطة ‪ B‬ذات الفاصلة‬

‫‪03‬‬
‫جند ‪ d x;3  d x;5 :‬أي‪. MA  MB :‬‬
‫‪ M‬تقع منتصف القطعة ‪ AB ‬ومنى ‪S2   4 :‬‬ ‫اي أن النقطة‬
‫‪A‬‬
‫‪2‬‬ ‫‪x2 2‬‬ ‫‪)3‬‬
‫‪0‬‬ ‫‪1‬‬
‫‪x   2  2‬‬ ‫لدينا ‪ x  2  2 :‬تكافئ ‪:‬‬
‫‪ M‬نقطة من املستقيه املدرج ذات الفاصلة ‪ x‬و لتكن النقطة ‪ A‬ذات الفاصلة ‪.  2‬‬
‫‪03‬‬ ‫جند ‪ d x;2  2 :‬أي‪. MA  2 :‬‬
‫‪S3   ;4  0;‬‬ ‫ومنى جمنوعة احللول هي ‪:‬‬
‫الصفخة ‪1/2‬‬
‫‪‬التنرين الثالث‪ 15 ( :‬نقاط)‬
‫‪ 1  y  7‬‬ ‫‪ x‬و ‪ y‬عددان حقيقان حيث ‪ 2  x  5 :‬و‬
‫‪2‬‬
‫‪ 2x 2 ‬‬ ‫‪، 5x  3 y ، x  y ، x  y‬‬ ‫اوجد حصر لكل من ‪:‬‬
‫‪y‬‬
‫اجلواب‪:4‬‬
‫‪x y‬‬ ‫‪ -‬إجياد حصر للعدد ‪:‬‬
‫‪01‬‬ ‫‪5  x  y  4‬‬ ‫لدينا ‪ 2  x  5 :‬و ‪  7  y  1‬يعين ‪:‬‬
‫‪x y‬‬ ‫‪ -‬إجياد حصر للعدد ‪:‬‬
‫‪1 y  7‬‬ ‫لدينا‪ 2  x  5 :‬و ‪  1  y  7‬اي ‪ 2  x  5 :‬و‬
‫‪01‬‬
‫‪3  x  y  12‬‬ ‫ومنى‪:‬‬
‫‪5x  3 y‬‬ ‫‪ -‬إجياد حصر للعدد ‪:‬‬
‫‪ 10  5x  25‬و ‪3  3 y  21‬‬ ‫لدينا‪ 2  x  5 :‬و ‪  1  y  7‬اي‪:‬‬
‫‪01‬‬
‫‪13  5x  3 y  46‬‬ ‫ومنى‪:‬‬
‫‪3‬‬
‫‪ 2 x2 ‬‬ ‫‪ -‬إجياد حصر للعدد ‪:‬‬
‫‪y‬‬
‫‪1‬‬ ‫‪1‬‬
‫‪02‬‬ ‫‪1 ‬‬ ‫‪‬‬ ‫لدينا‪ 2  x  5 :‬و ‪  1  y  7‬اي ‪ 4  x2  25 :‬و‬
‫‪y‬‬ ‫‪7‬‬
‫‪3 3‬‬
‫‪3‬‬ ‫‪‬‬ ‫اي‪  8  2x2  50 :‬و‬
‫‪y 7‬‬
‫‪3‬‬ ‫‪347‬‬
‫‪ 5  2 x 2 ‬‬ ‫‪‬‬ ‫ومنى‪:‬‬
‫‪y‬‬ ‫‪7‬‬

‫بالتوفيـق‬ ‫الصفخة ‪2/2‬‬


‫اجلنوورية اجلسائرية الدميقراطية الشعبية‬
‫ثانوية اجملاهد شونان حمند‬ ‫مديرية الرتبية لوالية اجللفة‬ ‫وزارة الرتبية الوطنية‬
‫الفرض الثاني للفصل األول ‪‬نوفنرب ‪9102‬‬
‫‪10‬‬ ‫‪0‬‬
‫اللقـ ــب ‪.......................:‬‬
‫اإلش ــه ‪.......................:‬‬
‫‪‬التنرٍن األًل‪ 14 ( :‬نقاط)‬
‫دالة معرفة بتنثَلوا البَانُ على اجملال ‪(  4;3‬الحظ الصكل )‬ ‫‪g‬‬

‫اخرت اجلٌاب املناشب يف كل مما ٍلُ ‪:‬‬


‫‪:‬‬ ‫‪g‬‬ ‫‪- 0‬الدالة‬
‫متناقصة متاما على اجملال ‪ 4;1‬‬ ‫متساٍدة متاما على اجملال ‪ 4;1‬‬
‫هُ ‪:‬‬ ‫‪g‬‬ ‫‪- 9‬صٌرة العدد ‪ 0‬بالدالة‬
‫‪1‬‬ ‫‪3‬‬ ‫‪1‬‬
‫تقبل قَنة حدٍة حملَة صغرى هُ‪:‬‬ ‫‪g‬‬ ‫‪- 3‬الدالة‬
‫‪3‬‬
‫‪1‬‬ ‫‪‬‬ ‫‪2‬‬
‫‪2‬‬
‫‪- 4‬للعدد ‪: 3‬‬
‫ال ٌٍجد لى شابقة‬ ‫شابقتان‬ ‫شابقة‬
‫‪‬التنرٍن الثانُ‪ 14 ( :‬نقاط)‬
‫املعرفة على اجملال ‪ 5;5‬‬ ‫‪f‬‬ ‫الصكل املقابل ميثل جسءا من املنحنى املنثل للدالة‬
‫أكنل الرشه بفرض ‪:‬‬

‫‪‬التنرٍن الثالث‪ 09 ( :‬نقطة)‬


‫نعترب الدالة العددٍة ‪ f‬املعرّفة على ‪ ‬بـ ‪ً ، f  x    x2  2 x  3 :‬لَكن ‪ C ‬متثَلوا البَانُ يف معله متعامد ًمتجانس‪.‬‬
‫‪ .0‬حتـقـّق بأنى من أجل كل ‪ x‬من ‪f  x     x  1  2 : ّ‬‬
‫‪2‬‬

‫على كل من اجملالني ‪ 1; ً  ;1‬‬ ‫‪ .9‬ادرس اجتاي تغَّر الدالة ‪f‬‬
‫‪ .3‬شكل جدًل تغريات الدالة ‪. f‬‬
‫‪ .4‬حدد القَنة احلدٍة للدالة ‪. f‬‬
‫‪ .5‬هل الدالة ‪ f‬زًجَة أً فردٍة ؟‬
‫الصفحة ‪1/9‬‬
‫اجلموورية اجلسائرية الدميقراطية الشعبية‬
‫ثانوية اجملاهد شونان حممد‬ ‫مديرية الرتبية لوالية اجللفة‬ ‫وزارة الرتبية الوطنية‬
‫تصحيح الفرض الثاني للفصل األول ‪‬نوفمرب ‪1023‬‬
‫املدٗ ‪ :‬ساعـ ــ٘‬ ‫مادٗ ‪ :‬الرٓاضٔات‬ ‫املشتْ‪ 2 : ٚ‬ج و ع تك ‪02‬‬
‫اللقــــب ‪:‬ثانوية اجلديدة غرب‬
‫االســــم ‪ 2 :‬ج ع تك ‪02‬‬
‫‪‬التنرًٓ األّل‪ 04 ( :‬ىقاط)‬
‫(الحظ الشكل )‬ ‫دال٘ معرف٘ بتنثٔلَا البٔاىٕ عل‪ ٙ‬اجملال ‪ 4;3‬‬ ‫‪g‬‬

‫اخرت اجلْاب املياسب يف كل مما ٓلٕ ‪:‬‬


‫‪:‬‬ ‫‪g‬‬ ‫‪- 2‬الدال٘‬
‫‪02‬‬ ‫متياقص٘ متاما عل‪ ٙ‬اجملال ‪ 4;1‬‬
‫ٍٕ ‪:‬‬ ‫‪g‬‬ ‫‪- 1‬صْرٗ العدد ‪ 0‬بالدال٘‬
‫‪02‬‬ ‫‪1‬‬
‫تقبل قٔن٘ حدٓ٘ حملٔ٘ صغر‪:ٍٕ ٚ‬‬ ‫‪g‬‬ ‫‪- 3‬الدال٘‬
‫‪3‬‬
‫‪02‬‬ ‫‪‬‬
‫‪2‬‬
‫‪- 4‬للعدد ‪: 3‬‬
‫‪02‬‬ ‫سابقتاٌ‬
‫‪‬التنرًٓ الثاىٕ‪ 04 ( :‬ىقاط)‬
‫املعرف٘ عل‪ ٙ‬اجملال ‪ 5;5‬‬ ‫‪f‬‬ ‫الشكل املقابل ميثل جز‪ٛ‬ا مً امليخي‪ ٙ‬املنثل للدال٘‬
‫أكنل الرسه بفرض ‪:‬‬

‫‪01‬‬

‫‪01‬‬

‫الصفحة ‪1/1‬‬
‫اجلْاب الثالث‬
‫‪f x   x  1  2‬‬ ‫‪ .2‬التخـقـّق بأىُ مً أجل كل ‪ x‬مً ‪: ّ‬‬
‫‪2‬‬
‫‪01‬‬
‫لدٓيا ‪ :‬مً أجل كل عدد حقٔقٕ ‪x  12  2  x2  2x  1  2  x2  2x  1  f x : x‬‬
‫ّميُ‪ :‬مً أجل كل ‪ x‬مً ‪f x   x  1  2 : ّ‬‬
‫‪2‬‬

‫‪ .1‬ادرس اجتاِ تغّٔر الدال٘ ‪ f‬عل‪ ٙ‬كل مً اجملالني ‪ 1; ّ  ;1‬‬
‫‪-‬دراس٘ إجتاِ تغري الدال٘ ‪ f‬يف اجملال ‪:  ;1‬‬
‫بفرض ‪ x2 ، x1‬مً اجملال ‪  ;1‬حٔث‪x1  x2 :‬‬
‫‪x1  1  x2  1‬‬ ‫بإضاف٘ العدد ‪ 1‬إىل طريف املتبآي٘ جند‪:‬‬
‫أٖ ‪x1  12  x2  12 :‬‬
‫بإظاف٘ العدد ‪  2‬إىل طريف املتبآي٘ جند‪x1  12  2  x2  12  2 :‬‬
‫أٖ ‪f x1   f x2  :‬‬
‫‪03‬‬ ‫ّميُ‪ :‬الدال٘ ‪ f‬متياقص٘ متاما عل‪ ٙ‬اجملال ‪ ;1‬‬
‫‪-‬دراس٘ إجتاِ تغري الدال٘ ‪ f‬يف اجملال ‪:  1;‬‬
‫بفرض ‪ x2 ، x1‬مً اجملال ‪  1;‬حٔث‪x1  x2 :‬‬
‫‪x1  1  x2  1‬‬ ‫بإظاف٘ العدد ‪ 1‬اىل طريف املتبآي٘ جند‪:‬‬
‫أٖ ‪x1  12  x2  12 :‬‬
‫بإضاف٘ العدد ‪  2‬اىل طريف املتبآي٘ جند‪x1  12  2  x2  12  2 :‬‬
‫أٖ ‪f x1   f x2  :‬‬
‫‪03‬‬ ‫ّميُ‪ :‬الدال٘ ‪ f‬متزآدٗ متاما عل‪ ٙ‬اجملال ‪ 1;‬‬
‫‪ .3‬تشكٔل جدّل تغريات الدال٘ ‪. f‬‬
‫‪x‬‬ ‫‪‬‬ ‫‪1‬‬ ‫‪‬‬

‫‪01‬‬
‫‪f x ‬‬
‫‪2‬‬

‫‪ .4‬حتدد القٔن٘ احلدٓ٘ للدال٘ ‪. f‬‬


‫‪01‬‬ ‫‪2‬‬ ‫مً جدّل تغريات الدال٘ ‪ f‬ىالحظ أٌ الدال٘ ‪ f‬تقبل قٔن٘ حدٓ٘ حملٔ٘ ( صغر‪ )ٚ‬قٔنتَا ‪:‬‬
22/10/2005:
: 2 1 1:

:
:

(−10)9 × 63 0.28 27 + 12 − 75 2 2+3 3


25 × 2 × 3
4 11 0.8 2 8 + 18 − 50 2 6+9 3

. 105 45 (1
45
. 105 (2

1053 454 105 ×45 (3

:
0.2121212121……. : A
.A

: -
C = 0.000637 × 10 −2 B = 0.000524 A = 2315698745

10-5 B 10-7 C -
A×B -
30 1: -2 1 1: - 22/10/2005 : -

06 - - - 01
06 . – – PGCD- 02
03 03
05 – – 04

:( 06 )
:

2 2 + 3 3 (2 2+3 3)(2 6-9 3) (4 12 – 18 6 + 6 18 -81) (8 3 -18 6 + 18 2 -81)


= = =- ∈R
2 6 + 9 3 (2 6+9 3)(2 6-9 3) (24-243) 219

27 + 12 − 75 ( 33 + 22 .3 – 3 .52) (3 3 + 2 3 – 5 3 )
(1.5) = )= =0 ∈ N
2 8 + 18 − 50 2 8 + 18 - 50 2 8 + 18 - 50

0.28 28 22 .7 7
(1.5) = = 4 = 2 ∈D
0.8 80 2 .5 2 5
(1.5)

(− 10)9 × 6 3 =-
(2 .5)9 .( 2 .3)3
=-
29.59 .23.33
=- 212-11 59-8 33-1 = -2 . 5. 32 =-90 ∈ Z (1.5)
2 4 11
25 4 × 211 × 3 (5 ) .2 .3 58 .211.3

:( 06 )
(02) 105 =3. 5 .7 45 =32 .5: (1
pgcd (104, 45) = 3 .5 =15 :
45 45 :15 3 45
. = = : (02) (2
105 105 :15 7 105
105 3 45 4 105 ×45 (3
1053 =(3.5.7)3 =33.5 3 .73 , 454 =(32.5)4 =38 .54 , 105 .45 =(3.5.7).(32 .5) = 33.52 .7 ( 02)

( 03 )
.A
100 A =21.2121212121.....
21
(03) A = : 99A=21 : 100A-A =21 100A-A = 99A
99
:( 05 )
9 -4 -6
A= 2.31569745 . 10 B =5.24 .10 C = 6.37 .10 (1.5) : -
( 0.5) 0.00052 10-5 B (0.5) 0.0000064 10 -7 C -
A×B -
(0.5) 5.10-4 B (0.5) 2.109 A
(01) 10.105 A .B
: 1 1:

:
: .1
75+ 49 4×75×13
62×8 - 288
2 26×25×53

. 2 -1

( 2-1)2
. -2
10-2-2

:
B=14,31598 A=0,019235 : B A
. 10-4 B A .1
. B A .2
.B A .3

-1
197 -2
.660660 -3

1 2006/10/28:
: 1:

06 - - - 01
04 . - 02
06 – – - 03
04 . – - 04

:( 06 )
:
4×75×13 4×52×13 1 1
5 3= 5 3 = 4 = ∈D 02
26×2 ×5 2×13×2 ×5 2 ×5 80
75+ 49 5 5+7
= ∈R 02
2 2
62×8 - 288 = 0 ∈N 02

:( 04 )
02 1.41 2 -

( 2-1)2
02 -
10-2-2
( 2 – 1 ) x2 / ( 1 0 yx 2 - - 2 )
:( 06)
01 0,0192 10-4 A .1
01 14,316 10-4 B
01 A=0,019235 = 1,9235 ×10-2 A .2
01 B=14,31598=1,431598 × 10 B
01 2×10-2 A .3
01 10 B

:( 04 )
1 .1
1.5 197 .2
1.5 660660 = 22× 3× 5× 7 × 112× 13 .3

1 2006/10/28:
‫نومفرب ‪1027‬‬ ‫املس توى‪ :‬ا ألوىل اثنوي (جذع مشرتك علوم) ‪TCST‬‬
‫فرض يف مادة الرايضيات للفصل ا ألول‬
‫‪8:‬‬
‫‪α = √6 − 2√5‬‬ ‫‪α‬‬
‫‪α‬‬ ‫‪(√5 − 1)2‬‬ ‫‪1‬‬

‫‪β = √4 − √10 + 2√5 + √4 + √10 + 2√5‬‬ ‫‪β2‬‬


‫‪2‬‬
‫‪β‬‬
‫‪β‬‬
‫‪β‬و‪α‬‬

‫‪4:‬‬
‫‪h‬‬ ‫‪r‬‬ ‫‪1.9l‬‬
‫‪3.14< 𝜋 < 3.15 ; 9 < ℎ < 9.1 9 < ℎ < 9.1‬‬
‫‪cm‬‬

‫‪8:‬‬

‫‪|𝒙−. . | ≤..‬‬ ‫‪d(.. ;..) ≤..‬‬ ‫] ‪x∈ [. . ; . .‬‬ ‫‪−2 ≤ 𝑥 ≤ 4‬‬


‫[‪x∈ ]−1; 6‬‬
‫‪d(x ;3/2) ≤ 7/2‬‬
‫‪|𝟐𝒙 + 𝟔| ≤ 4‬‬
‫‪IR‬‬

‫|𝟔 ‪|𝒙 + 𝟐| < |𝒙 −‬‬


‫𝟒 ≥ |𝟐 ‪|𝒙 +‬‬
‫الصفحة ‪1/1‬‬

‫‪Web site : www.ets-salim.com /Fax023.94.83.37‬‬ ‫‪- Tel : 0560.94.88.02/05.60.91.22.41/05.60.94.88.05 :‬‬


:
1.1 (√5 − 1)2 =6 − 2√5 1
1.11 α √5 − 1 : α
2
‫ن‬1.1.......................................................................................... β = 6 − 2√5 2
1.1 β √5 + 1 : β
1.1 𝑎<𝑏 : 3

:
3.14< 𝜋 < 3.15 9 < ℎ < 9.1 8< 𝑟 < 8.1
x
𝜋 𝑟2ℎ
64 < 𝑟 2 < 65.61 8< 𝑟 < 8.1

𝑐𝑚3 1808.64 < 𝜋𝑟 2 ℎ < 1880.70𝑐𝑚3

1.8𝑙 < 𝜋𝑟 2 ℎ < 1.88𝑙


:

|𝒙 − 𝟏| ≤ 3 d(x ;1) ≤ 3 x∈ [−2; 4] −2 ≤ 𝑥 ≤ 4


|𝒙 − 𝟓/𝟐| ≤ 7/2 d(x ;5/2) < 7/2 x∈ ]−1; 6[ −1 < 𝑥 < 6
|𝒙 − 𝟑/𝟐| ≤ 7/2 d(x ;3/2) ≤ 7/2 x∈ [−2; 5] −2 ≤ 𝑥 ≤ 5
|𝟐𝒙 + 𝟔| ≤ 4 d(x ;-3) ≤ 2 x∈ [−5; −1] −5 ≤ 𝑥 ≤ −1
|𝒙 + 𝟐| < |𝒙 − 𝟔| IR

1.1
1.1 s = ]−∞; 2[
|𝒙 + 𝟐| ≥ 𝟒 IR
1.1
1.1…………………………………………………………………………. s = ]−∞; 6] ∪ [2 , +∞[

Web site : www.ets-salim.com /Fax023.94.83.37 - Tel : 0560.94.88.02/05.60.91.22.41/05.60.94.88.05 :


éJ


 J£ñË@ éJ 
 K QJË@

.
 Pð
èP@
é‚ 
 K éK BñË éJ K QË@
.

. éK
QK

 
2017/2018: éJ
ƒ@P YË@ éJ‚Ë@ Yªƒ  ñ
®m ×: éK
 ñKA

K
Aƒ 
1:
èYÖ Ï@ ù
ÒÊ« 1: øñJ‚Ö
 Ï@
b š¤± ¨®l˜ š¤± |rf˜ c
.PAJ.J«B
 @ á
ªK. Y g ñ K éK . Ag. B @ ú¯ é¯YË@  
 ð Õæ¢ J JË@


: é¢kCÓ
: 1 áK
QÖß
TbFAnm˜ AžA˜ ¨ ? T›®ˆ ‰R a 6= b : y  Ay`ybV   dˆ b ¤ a
N Z D Q R
3.10−3
π2
15

2
a2 − b2
a−b
: 2 áK
QÖß
1 1
7.(−9)3 .(12)2 .7 −
A= B= 2 3 ¨þþl§A› Xs
72 .25 .3−2 1
1 − ( )2
2
: 3 áK
QÖß

a+b= 5 ¤ ab = 1 ¤ a > b y A›Am  Abw›  Ayqyq√  dˆ b ¤ a
√ √ √
a− b √¤ a√ + b s /1
b ¤ a ¤ b ¤ a : tntF /2
: 4 áK
QÖß
B = 2700 ¤ A = 6300 y  Ayqyq   dˆ B ¤ A
Ty˜¤ ™› wˆ º d Y˜ B ¤ A ™l/1
P P CM (A; B) ¤ P GCD(A; B) : s /2
A
rsk˜ šzt /3
√ B
√ √ √
Ay`ybV dˆ A.B.n  wk§√Yt n ¨`ybW˜ √d`˜ Tmy’ rŒ} Ÿyˆ /4
A+ B ¤ A.B þþ˜ rO d 1.7 < 3 < 1.8 ¤ 2.6 < 7 < 2.7   mlˆ Ð /5
1
‡J
 ¯ñ   P : èXAÖ
 Ï @ XA Jƒ
 @ f

JËAK. YJ
Ëð ú
¯P
‫عرض حال للفرض األول‬
‫حل التمرين األول‬
‫𝑁‬ ‫𝑍‬ ‫𝐷‬ ‫𝑄‬ ‫𝑅‬
‫‪3. 10−3‬‬ ‫∗‬ ‫∗‬ ‫∗‬
‫‪π2‬‬ ‫∗‬ ‫∗‬
‫‪15‬‬ ‫∗‬ ‫∗‬ ‫∗‬
‫‪−‬‬
‫‪2‬‬
‫‪𝑎2 − b2‬‬ ‫∗‬ ‫∗‬ ‫∗‬ ‫∗‬ ‫∗‬
‫‪a−b‬‬
‫حل التمرين الثاني‬
‫‪7. (−9)3 . 122 . 7 −72 . (32 )3 . (3. 22 )2 −72 . 38 . 24‬‬ ‫‪310‬‬
‫=𝐴‬ ‫=‬ ‫‪= 2 5 −2 = −‬‬
‫‪72 . 25 . 3−2‬‬ ‫‪72 . 25 . 3−2‬‬ ‫‪7 .2 .3‬‬ ‫‪2‬‬
‫‪1 1‬‬ ‫‪1‬‬ ‫‪1‬‬
‫‪−‬‬ ‫‪1 4 2‬‬
‫= ‪𝐵= 2 32= 6 = 6 = .‬‬
‫‪1‬‬ ‫‪1‬‬ ‫‪3 6 3 9‬‬
‫‪1‬‬ ‫‪−‬‬
‫)‪1 − (2‬‬ ‫‪4‬‬ ‫‪4‬‬
‫حل التمرين الثالث‬
‫‪2‬‬
‫‪(√𝑎 + √b) = a + b + 2√a. b‬‬
‫‪= √5 + 2√1 = √5 + 1‬‬
‫‪2‬‬
‫‪(√𝑎 + √b) = √5 + 1‬‬
‫‪√𝑎 + √b = √√5 + 1‬‬
‫بنفس الطريقة نجد ‪:‬‬
‫‪√𝑎 − √b = √√5 − 1‬‬
‫‪1‬‬
‫‪√√5+1+√√5−1‬‬
‫= 𝑎√‬ ‫‪2‬‬
‫بحل جملة معادلتين نحصل على ‪:‬‬
‫‪√√5 + 1 − √√5 − 1‬‬
‫= 𝑏√‬
‫‪2‬‬
‫‪2‬‬
‫‪√√5+1+√√5−1‬‬ ‫‪2√5+4‬‬ ‫‪√5‬‬
‫(=𝑎‬ ‫‪2‬‬
‫= )‬ ‫‪4‬‬
‫=‬ ‫‪2‬‬
‫‪+1‬‬ ‫ومنه ‪:‬‬
‫‪2‬‬
‫‪√√5+1−√√5−1‬‬ ‫‪2√5−4‬‬ ‫‪√5‬‬
‫(=𝑏‬ ‫= )‬ ‫=‬ ‫‪−1‬‬ ‫ومنه ‪:‬‬
‫‪2‬‬ ‫‪4‬‬ ‫‪2‬‬
‫اصغر قيمة لــــ 𝑛 هي ‪ 𝑛 = 5 .7 :‬اي ‪𝑛 = 35‬‬
‫حل تمرين‪04‬‬
‫‪𝐴 = 6300 = 22 . 32 . 52 . 7‬‬ ‫التحليل الى جداء عوامل أولية ‪:‬‬ ‫‪/1‬‬
‫‪𝐵 = 2700 = 22 . 33 . 52‬‬
‫‪𝑃𝐺𝐶𝐷(𝐴; 𝐵) = 22 . 32 . 52 = 900‬‬ ‫‪/2‬‬
‫‪𝑃𝐺𝐶𝐷(𝐴; 𝐵) = 52 . 22 . 33 . 7 = 18900‬‬
‫𝐴‬ ‫‪52 .22 .32 .7‬‬ ‫‪7‬‬
‫=‬ ‫‪3‬‬ ‫=‬ ‫‪ /3‬اختزال الكسر ‪:‬‬
‫‪B‬‬ ‫‪5 .22.32‬‬ ‫‪5‬‬
‫‪: /4‬‬
‫‪√𝐴. 𝐵. 𝑛 = √22 . 32 . 52 . 7. 22 . 33 . 52 . n‬‬
‫‪= √24 . 35 . 54 . 7. n‬‬
‫‪2‬‬
‫‪= 22 52 √35 . 7. n‬‬
‫اصغر قيمة لــــ 𝑛 هي ‪ 𝑛 = 3 .7 :‬اي ‪𝑛 = 21‬‬
‫‪ /5‬الحصر ‪:‬‬
‫‪2‬‬ ‫‪2‬‬ ‫‪2‬‬
‫‪√𝐴 = √2‬‬ ‫‪. 3 . 5 . 7 = 2.3.5. √7 = 30√7‬‬
‫‪2 3 2‬‬
‫‪√𝐵 = √2 . 3 . 5 = 2.5. √3 = 25√3‬‬
‫‪√𝐴 + √𝐵 = 30√7 + 25√3‬‬
‫‪√𝐴. 𝐵 = 750√7. √3‬‬
‫لدينا ‪ 2,6 < √7 < 2,7 :‬و ‪1,7 < √3 < 1,8‬‬
‫‪(1,7). (2,6). 750 < √𝐴. 𝐵 < (1,8). (2,7). 750‬‬
‫‪3315 < √𝐴. 𝐵 < 3645‬‬
‫)‪(30). (2,6) + (25). (1,7) < √𝐴 + √𝐵 < (30). (2,7) + (25). (1,8‬‬
‫‪120,5 < √𝐴 + √𝐵 < 126‬‬
‫‪3‬‬
 º  Šǀſ¦°ËŶƫ¦ ŠƴËƀƫ¦ º ŶǀƸƄƫ¦ ŠƿƺƳŚŧ

©ŚǀƋŚƿźƫ¦
Ë ¨Ë®ŚƯ ƾƟ ¾Âǃ¦ ƪƈƠƬƫ ¾Âǃ¦ µźƠƫ¦

߱ࠧƎƥ ࣍ᄕߢड़ ƆƮ࡫ஷளƄ ߢࡉࢬইऐইࡓࣺ࠭ ƥ Ƣ৬৚ᄕ Ơഗ೻४ञ Ɩࡪᄛ ᐆᏸƥƀ ࣍ࡓƮ१ऐƄ Ʀইࡖ१ஷளƄ

Ƒ‫  ޚ߼ݒ‬ ¾Âǃ¦
Ë ƲƿźưËŤƫ¦ƫ¦

am
୴ୢࠠ ߢञ ዑወከƀ ∈
/ ƥƀ ∈ ಑౵ఞ఑࿆ྵƄ ơߢड߿ࡖड़ߢࠧߢࠧ ‚
  √ √
7+ 3+ 1...N, −45...R
− 45....R
45
a2 −b2
a+b
...N a > b ࠆࡉࢨ ྇ཫࡉ߿ࡉࡐॴ
ࡐॴ ಑౵ƌ࡫ᄕ b ƥ a
√ √
B = [( 32 )2 + 4−1 ]−2 , A = − 3+ √ 363
2 3
 B ƥ A ಑౵ƌ࡫߿ऐ
಑౵ƌ࡫߿ऐƄ ॱ೘
ॱ೘ು ƒ
0.0025×10−6 ×3×108
b= 0.15×10−4
, a = 0.045 × 10−2 ࠆࡉࢨ
ࠆࡉࢨ ƣߢࠠ ၙᅈ߸ ƣ
ƣƄƌ࡫ᄕ b, a
‫ݸ‬খฅ฀߿ऐƄ
߿ऐƄ ኖቴ४ऐƄ
ቴ४ऐƄ ୮ୢᄕ b ƥ a णञ ኖቄ ࠑࡖࣨƀ „
‫ݸ‬a × b ƎƄ࡫ࢌञ ࣍௴ຮƎ ྇ཫᄕ …

Ƒ‫  ޚ߼ݒ‬ ƾƳŚŨƫ¦ ƲƿźưËŤƫ¦




‫ ݸ‬12
12, 56 5656... ƌ࡫߿ᓪᓤ ࣍ࠠ ၙᅅࣺᝢऐƄ ࣍ࠧߢཏ༧ऐƄ ࡫ᄛ ‚
‫ݸ‬ᐌᏸƥƀ ƌ࡫ᄕ 103 ƌ࡫߿ऐƄ ऊ࣎ ƒ
D = 540,
540, C = 1125 ࠆࡉࢨ ƣߢࡉ߿ࡉࡐॴ ƣƄƌ࡫ᄕ D ƥ C णࣺࡉऐ
‫ࡉ࣍ݸ‬ऐƥƀ ऊञƄই߸ žƄ࡫ᄛ ᐆᏸƂ D ƥ C ಑౵ƌ࡫߿ऐƄ ᔲᔎᄗ „
࣍ࡉऐƥƀ ऊञƄই߸ žƄ࡫ᄛ ᐆᏸƂ ऊࡉखᄨࡖऐƄ ơߢड߿ࡖड़ߢࠧ …
‫ݸ‬D ƥ C ಑౵ƌ࡫߿ᓪᓤ ഗ೵ࣨூ஺Ƅ Ơഗ೻४ஷளƄ ᆵᄺߢࢌऐƄ ྇ཫᄕ ƀ
‫ݸ‬D ƥ C ಑౵ƌ࡫߿ᓪᓤ ऽߺॉூ஺Ƅ Ơഗ೻४ஷளƄ ࡵ߸ߢौஷளƄ ྇ཫᄕ ƅ
‫ݸ‬ơƄക೻ᄖ෴ᓰᓤ ऊࠧߢࡿ ഗ೮ᄐ ኖቴय़ ୮ୢᄕ 540
1125
ၙᅅࣺᝢऐƄ ࠑࡖࣨƀ Ɖ
√ √
‫ ݸ‬D ƥ C ᝢऐ ƆၙᅗࡖෝොෘƄ ࣍ࠧߢཏ༧ऐƄ ྇ཫᄕ ƌ

‫ ॴࡐࡉ߿ࡉߢݸ‬Ƅƌ࡫ᄕ C × D × n ƣইࣺࠠ တ࿹ࢨ n ුఏࡉࡐॷऐƄ ƌ࡫߿ᓪᓤ ࣍डࡉࡿ ऽߺॉƀ ྇ཫᄕ ᝢ࣎
Ƒ‫  ޚ߼ݒ‬ ŦƫŚŨƫ¦ ƲƿźưËŤƫ¦
‫ߢݸ‬ञߢൿ൰ ࠑࢬইञ ਎৶ࡉࢌࢨ ƌ࡫ᄕ x
√ √ √ √
‫ݸ‬ x + 1 − x ƌ࡫߿ऐƄ ই࣎ x + 1 + x ƌ࡫߿ऐƄ ƅ৬৚ࢌञ ƣƀ ྇ཫࠧ ‚
‫ݸ‬S= 1√
1+ 2
+ √ 1√
2+ 3
+ √ 1√
3+ 4
+ ... + √ 1√
99+ 100
Ɩইड෢ොෘƄ ࣍डࡉࡿ ࢛ࡖࡓᆛड़Ƅ ƒ

मࡉࢀইࡖऐߢࠧ
æúńļĈįà çŃľķáî
Ʃƹǃř ƪƈƠƬƫ Ʃƹǃř ƉźƠƫř ŮǀŰƈţ
šŚǀƋŚƿźƫř şŵŚưƫ
ᦅᦃᦄᦌᦅᦃᦄᦋ

s
 D ƥ C ಑౵ƌ࡫߿ऐƄ ऊࡉख࿏Ⴏ 
 Ŝ IJħý ĶŃþĴê

m
C = 1125 = 32 × 53 

D = 540 = 33 × 22 × 5  

xa
√ √
7+ 3 + 1 ∈ N, −45
− 45 ∈
/R
P GCD(C, D) = 32 × 5 = 45 
ƀ
a2 − b2
P P CM (C, D) = 33 × 53 × 22 = 13500 ƅ@
 =a−b∈N
a+b
ၙᅅࣺᝢऐƄ ơƄക೻ᄖƄ 
Ɖ  B ƥ A ྇ཫ
྇ཫ࠭Ǝߢࡐ߿ऐƄ ॱࡉ१ᆕᅲ 


m
540 33 × 22 × 5 12 √ √
= =
1125 2
3 ×5 3 25 − 3 + 11 3 10
1
A= √ = =5
2 3 2

√  √
D = 33 × 22 × 5 = 6 15
ॱࡉ१ᆕࡖऐƄ 
ƌ co 3
B = [( )2 + 4−
2
−1 −2
] = 3 2
1
[( 2 ) + 4−1 ]2
1 1 4
√  √ = 9 1 2 = 25 =
C = 32 × 53 = 15 5 (4 + 4) 4
25

 n ࣍डࡉࡿ ྇ཫࡉ߿࠭ 
ᝢ࣎ ࣍ࡉฅ฀߿ऐƄ ࣍ࠧߢཏ༧ऐƄ 

io

√  √
C ×D×n= 35 × 22 × 54 × n = 32 ×2×5
×52 × 33n
n a = 4, 5 × 10−4
‫ݸ‬n = 3 ࣉࡓञƥ b = 5 × 104
at

 a × b ƎƄ࡫ࢌஷளƄ ࣍௴ຮƎ 

Ş IJħý ĶŃþĴê
uc

a × b = 2 × 101
྇ཫࡉᆕᅲ 
྇ཫ 
√ √ ŝ IJħý ĶŃþĴê
1 1 x+1− x
√ √ =√ √ ×√ √
ed

x+1+ x x+1+ x x+1− x ‫ ݸ‬ই࣎ ƎƥጭጘƄ ƢߢࡿƎƀ ƌ࡫ᄕ


√ √
x+1− x
=

x+1−x

K ƌ࡫߿ᓪᓤ ࣍ࠠ ၙᅅࣺᝢऐƄ ࣍ࠧߢཏ༧ऐƄ 

= x+1− x
y-

K = 12, 565656...
2
Ɩইड෢ොෘƄ ƅߢ१ࢨ 
 10 .K = 1256.565656..
100K = 1244 + 12.565656..
ŤƄƯ ƕŸū  Šƴſ

(100K − K) = 1244
n

1 1 1
S= √ +√ √ + ... + √ √ 1244
1+ 2 2+ 3 99 + 100 K=
√ √ √ √ √ √ √ 99
Śǀūƺƫƺƴƨţƹ ƭƺƬƗ ƥźŤƄƯ

= 2 − 1 + 3 − 2 + 4 − 3 + .. + 100 − 99

= −1 + 100  103 ƌ࡫߿ऐƄ ࣍ࡉऐƥƀ ƎߢࡐࡖࢧƄ 


= −1 + 10 11 7 5 3 2 ୮ୢᄕ ࣍ड१ࢌऐƄ ऊࡐࢌࠠ ऊ࣎


=9 9 14 20 34 51 ࣍ड१ࢌऐƄ ऊॉߢᄗ
4 5 3 1 1 ࣍ड१ࢌऐƄ ᐌᏥߢࠧ
ƣƍƂ ࣉࡉखᄕ Ƣই१ࢌஷளƄ णञ ऊࡿƀ  ࣍ड१ࢌऐƄ ऊॉߢᄗ ƣƀ ॰ࢨ෵෰ࠪ
‫ݸ‬ᐌᏸƥƀ ƌ࡫ᄕ 

 णञ ࣍ᄨࢍॉ
‫ﺍﳉﻤﻬﻮﺭﻳﺔ ﺍﳉﺰﺍﺋﺮﻳﺔ ﺍﻟﺪﳝﻘﺮﺍﻃﻴﺔ ﺍﻟﺸﻌﺒﻴﺔ‬
‫ﺍﻟﺸﻌﺒﺔ ‪:‬ﺃﻭﱃ ﻋﻠﻮﻡ ﻭﺗﻜﻨﻮﻟﻮﺟﻴﺎ‬ ‫ﺛﺎﻧﻮﻳﺔ ﺳﺎﺟﻲ ﺍﳌﺨﺘﺎﺭ ﺍﻟﺴﻤﺎﺭ‬

‫ﺳﺎ‬ ‫‪02‬‬ ‫اﻟﻤــﺪة ‪:‬‬ ‫—‬ ‫اﻟﻔﺮض اﻟﻤﺤﺮوس اﻷول ﻓﻲ ﻣﺎدة ‪ :‬ﺍﻟﺮﻳﺎﺿﻴﺎﺕ‬

‫ﻧﻘﺎط(‬ ‫)‪06‬‬ ‫اﻟﺘﻤﺮﻳﻦ اﻷول‪:‬‬


‫‪x‬‬ ‫‪x−1‬‬
‫=‪.B‬‬ ‫=‪A‬و‬ ‫‪ x‬عدد حقيقي موجب تماما نضع ‪:‬‬
‫‪x+1‬‬ ‫‪x‬‬
‫‪ (1‬احسب الفرق ‪. A − B‬‬

‫‪ (2‬استنتج اشارة الفرق بين ‪ ، A − B‬ثم قارن بين ‪ A‬و ‪. B‬‬


‫√‬ ‫√‬
‫‪2017‬‬ ‫‪2018‬‬
‫‪.‬‬ ‫و‬ ‫‪ (3‬استنتج مقارنة بين العددين‬
‫‪2018‬‬ ‫‪2019‬‬

‫ﻧﻘﺎط(‬ ‫)‪06‬‬ ‫اﻟﺘﻤﺮﻳﻦ اﻟﺜﺎﻧﻲ‪:‬‬

‫‪ (1‬حل ّل كل من العددين ‪ 1782‬و ‪ 999‬الى جداء عوامل أولية ‪.‬‬

‫‪ (2‬استنتج )‪. PGCD (999, 1782‬‬

‫‪ (3‬نضع ‪. a = 1.783782783... = 1.783 :‬‬

‫أ( ما طبيعة العدد ‪. a‬‬


‫‪1782‬‬
‫=‪.a‬‬ ‫بـ( بيّن أن‬
‫‪999‬‬
‫جـ( استنتج الشكل الغير قابل للإختزال للعدد ‪. a‬‬

‫ﻧﻘﺎط(‬ ‫)‪12‬‬ ‫اﻟﺘﻤﺮﻳﻦ اﻟﺜﺎﻟﺚ‪:‬‬


‫‪ x‬و ‪ y‬عددان حقيقيان حيث ‪ 6 ≺ x ≺ 10 :‬و ‪. −10 ≺ y ≺ −9‬‬

‫‪ I‬و ‪ J‬مجموعتان من ‪ R‬حيث ‪ I = [−2 ; 1] ∪ [2; 4] :‬و [∞‪. J = [0 ; 1 [∪ [5; +‬‬

‫‪ (1‬عيّن المجالات التالية ‪ I ∩ J :‬و ‪. I ∪ J‬‬

‫‪ (2‬اعط حصرا لكل من ‪ x − y‬و ‪. xy‬‬

‫‪ (3‬بيّن ان ‪. | x − 8 | ≺ 2 :‬‬

‫‪ (4‬عب ّر عن الحصر ‪ −10 ≺ y ≺ −9‬على شكل مجال ثم على شكل قيمة مطلقة ‪.‬‬

‫‪ (5‬حل المعادلة ‪ | y − 8 | = 2‬ثم استنتج حلول المتراجحة ‪. | y − 8 | ≺ 2‬‬

‫ﲤﻨﻴﺎﺗﻲ ﻟﻜﻢ ﺑﺎﻟﺘﻮﻓﻴﻖ‬


‫انتهى‬ ‫صفحة ‪ 1‬من ‪3‬‬ ‫مكتوب بـ‪Arab XELATEX :‬‬
‫اﻟﺘﺼﺤﻴﺢ اﻟﻤﻔﺼﻞ ﻟﻠﻔﺮض اﻟﻤﺤﺮوس اﻷول ﻟﻠﻔﺼﻞ اﻷول‬

‫ﺣﻞ اﻟﺘﻤﺮﻳﻦ اﻷول‬


‫‪ (1‬حساب الفرق ‪. A − B‬‬
‫‪x‬‬ ‫‪x−1‬‬
‫= ‪ B‬ومنه ‪:‬‬ ‫=‪A‬و‬ ‫لدينا‪:‬‬
‫‪x+1‬‬ ‫‪x‬‬
‫‪x−1‬‬ ‫‪x‬‬ ‫‪( x − 1)( x + 1) − x2‬‬ ‫‪x2 − 1 − x2‬‬ ‫‪−1‬‬
‫= ‪ A − B‬وبالتالي ‪:‬‬ ‫‪−‬‬ ‫=‬ ‫=‬ ‫=‬
‫‪x‬‬ ‫‪x+1‬‬ ‫)‪x ( x + 1‬‬ ‫)‪x ( x + 1‬‬ ‫)‪x ( x + 1‬‬
‫‪−1‬‬
‫= ‪A−B‬‬
‫)‪x ( x + 1‬‬

‫‪ (2‬استنتاج اشارة الفرق بين ‪A − B‬‬


‫‪−1‬‬
‫وبالتايلي ‪A − B ≺ 0‬‬ ‫بمأن ‪ x ≻ 0‬فإن ‪ x ( x + 1) ≻ 0‬وعليه ‪≺ 0‬‬
‫)‪x ( x + 1‬‬
‫‪ -‬المقارنة بين ‪ A‬و ‪. B‬‬
‫بمأن ‪ A − B ≺ 0‬فإن ‪ A ≺ B‬وعليه ‪:‬‬

‫‪x−1‬‬ ‫‪x‬‬
‫≺‬ ‫)‪(1‬‬
‫‪x‬‬ ‫‪x+1‬‬
‫√‬ ‫√‬
‫‪2017‬‬ ‫‪2018‬‬
‫‪.‬‬ ‫و‬ ‫‪ (3‬استنتاج مقارنة بين العددين‬
‫‪2018‬‬ ‫‪2019‬‬
‫‪2018‬‬ ‫‪2017‬‬ ‫‪2017‬‬ ‫‪2018‬‬
‫فإن ‪:‬‬ ‫و‪≻0‬‬ ‫وبمأن ‪≻ 0‬‬ ‫≺‬ ‫بأخذ ‪ x = 2018‬في المتراجحة )‪ (1‬نجد ‪:‬‬
‫‪2019‬‬ ‫‪2018‬‬ ‫‪2018‬‬ ‫‪2019‬‬ ‫√‬ ‫√‬
‫‪2017‬‬ ‫‪2018‬‬
‫≺‬
‫‪2018‬‬ ‫‪2019‬‬

‫ﺣﻞ اﻟﺘﻤﺮﻳﻦ اﻟﺜﺎﻧﻲ‬

‫‪ (1‬تحليل العددين ‪ 1788‬و ‪ 999‬الى جداء عوامل أولية ‪.‬‬


‫لدينا ‪999 = 33 × 37 :‬‬
‫و ‪1782 = 2 × 34 × 11‬‬

‫‪ (2‬استنتاج )‪. PGCD (999, 1782‬‬


‫‪PGCD (999, 1782) = 33 = 27‬‬

‫أ( طبيعة العدد ‪. a‬‬ ‫‪(3‬‬


‫بمأن للعدد ‪ a‬دور فهو عدد عشري‬

‫‪1782‬‬
‫=‪.a‬‬ ‫بـ( تبيان أن‬
‫‪999‬‬
‫لدينا ‪a = 1.783782783... = 1.783 :‬‬
‫ومنه ‪ a = 1 + 0.783782783... :‬بأخذ ‪ x = 0.783782783...‬نجد ‪1000x = 783.782783...‬‬
‫‪783‬‬
‫=‪x‬‬ ‫اي ‪ 1000x = 783 + 0.782783...‬اي ‪ 1000x = 783 + x‬وبالتالي ‪ 999x = 783‬ومنه‬
‫‪999‬‬

‫تصحيح الفرض‬ ‫صفحة ‪ 2‬من ‪3‬‬ ‫مكتوب بـ‪Arab XELATEX :‬‬


‫‪783‬‬
‫‪a = 1+‬‬ ‫لدينا ‪ a = 1 + 0.783782783... :‬و ‪ x = 0.783782783...‬ومنه ‪ a = 1 + x‬اي‬
‫‪999‬‬
‫‪1782‬‬
‫=‪a‬‬ ‫واخيرا نجد ‪:‬‬
‫‪999‬‬
‫جـ( استنتاج الشكل الغير قابل للإختزال للعدد ‪. a‬‬
‫‪1782‬‬
‫‪66‬‬
‫=‪a‬‬ ‫بمأن ‪ PGCD (999, 1782) = 27‬فإن ‪ a = 27‬ومنه ‪:‬‬
‫‪37‬‬ ‫‪999‬‬
‫‪27‬‬

‫ﺣﻞ اﻟﺘﻤﺮﻳﻦ اﻟﺜﺎﻟﺚ‬


‫‪ x‬و ‪ y‬عددان حقيقيان حيث ‪ 6 ≺ x ≺ 10 :‬و ‪. −10 ≺ y ≺ −9‬‬

‫‪ I‬و ‪ J‬مجموعتان من ‪ R‬حيث ‪ I = [−2 ; 1] ∪ [2; 4] :‬و [∞‪. J = [0 ; 1 [∪ [5; +‬‬

‫‪ (1‬تعيّن المجالات ‪ I ∩ J :‬و ‪. I ∪ J‬‬


‫لدينا ‪ I = [−2 ; 1] ∪ [2; 4] :‬و [∞‪ J = [0 ; 1 [∪ [5; +‬ومنه نجد ‪:‬‬
‫[‪ I ∩ J = [0, 1‬و [∞‪I ∪ J = [−2 ; 1] ∪ [2 ; 4] ∪ [5; +‬‬

‫‪ (2‬اعطاء حصرا لكل من ‪ x − y‬و ‪. xy‬‬

‫‪ -‬حصر ‪: x − y‬‬
‫لدينا ‪:‬‬

‫‪6 ≺ x ≺ 10‬‬ ‫)‪(2‬‬


‫‪−10 ≺ y ≺ −9‬‬ ‫)‪(3‬‬

‫بضرب المتراجحة )‪ (3‬في العدد )‪ (−1‬نجد ‪:‬‬

‫‪9 ≺ −y ≺ 10‬‬ ‫)‪(4‬‬

‫بالجمع بين المتراجحتين )‪ (2‬و )‪ (4‬نجد ‪15 ≺ x − y ≺ 20 :‬‬


‫‪ -‬حصر ‪: xy‬‬
‫بالضرب بين المتراجحتين )‪ (2‬و )‪ (4‬نجد ‪54 ≺ − xy ≺ 100 :‬‬
‫بضرب هذه المتراجحة الأخيرة في العدد )‪ (−1‬نجد ‪−100 ≺ xy ≺ −54 :‬‬

‫‪ (3‬تبيان أن ‪. | x − 8 | ≺ 2 :‬‬
‫لدينا ‪ 6 ≺ x ≺ 10 :‬بطرح العدد )‪ (8‬من هذه المتراجحة نجد ‪ −2 ≺ x − 8 ≺ 2 :‬ومنه ‪. | x − 8 | ≺ 2‬‬

‫‪ (4‬التعبيرعن الحصر ‪ −10 ≺ y ≺ −9‬على شكل مجال ثم على شكل قيمة مطلقة ‪.‬‬

‫‪ 7‬على شكل مجال ‪:‬‬


‫‪ −10 ≺ y ≺ −9‬معناه ‪y ∈ [−10 ; −9] :‬‬
‫‪ 7‬على شكل قيمة مطلقة ‪:‬‬
‫ ‬ ‫ ‬
‫ ‬ ‫ ‬
‫‪ x + 19 ≺ 1‬‬ ‫‪ −10 ≺ y ≺ −9‬معناه ‪:‬‬
‫ ‬ ‫ ‪2‬‬ ‫‪2‬‬

‫تصحيح الفرض‬ ‫صفحة ‪ 2‬من ‪3‬‬ ‫مكتوب بـ‪Arab XELATEX :‬‬


‫‪ (5‬حل المعادلة ‪ | y − 8 | = 2‬ثم استنتاج حلول المتراجحة ‪. | y − 8 | ≺ 2‬‬

‫‪ 7‬حل المعادلة ‪: | y − 8 | = 2‬‬


‫{‬ ‫{‬ ‫ولتكن ‪ S‬مجموعة حلولها ‪:‬‬
‫‪y = 10‬‬ ‫‪y−8 = 2‬‬
‫ومنه ‪S = {6, 10} :‬‬ ‫تكافئ‬ ‫‪ | y − 8 | = 2‬تكافئ‬
‫‪y=6‬‬ ‫‪y − 8 = −2‬‬
‫‪ 7‬استنتاج حلول المتراجحة ‪: | y − 8 | ≺ 2‬‬
‫لتكن ‪ D‬مجموعة حلول هذه المتراجحة ‪:‬‬
‫‪ | y − 8 | ≺ 2‬تكافئ ‪ −2 ≺ y − 8 ≺ 2‬تكافئ ‪ 6 ≺ y ≺ 10‬تكافئ [‪ y ∈ ]6; 10‬ومنه ‪. D = ]6; 10[ :‬‬

‫ﻣﻊ ﺧﺎﻟﺺ ﲤﻨﻴﺎﺗﻨﺎ ﻟﻜﻢ ﺑﺎﻟﺘﻮﻓﻴﻖ ﻭ ﺍﻟﻨﺠﺎﺡ‬

‫تصحيح الفرض‬ ‫صفحة ‪ 3‬من ‪3‬‬ ‫مكتوب بـ‪Arab XELATEX :‬‬


‫السنة الدراسية ‪2020 − 2019 :‬‬ ‫القسم‪ 01 :‬ج م ع ‪2‬‬ ‫ثانو ية ‪ :‬أحمد رضا حوحو‬

‫ﺍﻟﻔﺮﺽ ﺍﻟﻤﺤﺮﻭﺱ ﺍﻷﻭﻝ ﻟﻠﺜﻼﺛﻲ ﺍﻷﻭﻝ ﻓﻲ ﻣﺎﺩﺓ ﺍﻟﺮﻳﺎﺿﻴﺎﺕ‬

‫اليوم ‪2019/10/22 :‬‬ ‫الأستاذة‪ :‬مرواني‪.‬ن‬ ‫المدة‪ 01 :‬ساعة —‬

‫التمرين الأول‪:‬‬
‫‪x−1‬‬ ‫‪x‬‬
‫=‪B‬‬ ‫و‬ ‫=‪A‬‬ ‫‪ x‬عدد حقيقي موجب تماما حيث ‪:‬‬
‫‪x‬‬ ‫‪x+1‬‬

‫⃝ أحسب الفرق ‪.A − B‬‬


‫‪1‬‬

‫‪.B‬‬ ‫و‬ ‫⃝ استنتج إشارة الفرق ‪ A − B‬ثم قارن ‪A‬‬


‫‪2‬‬
‫√‬ ‫√‬
‫‪2018‬‬ ‫‪2019‬‬
‫و‬ ‫⃝ إستنتج مقارنة بين العددين‬
‫‪3‬‬
‫‪2019‬‬ ‫‪2020‬‬

‫التمرين الثاني‪:‬‬

‫ثم أحسب‬ ‫‪7n+2 − 7n‬‬ ‫و‬ ‫⃝ أوجد تحليلا إلى جداء عوامل أولية لكل من ‪5n+2 − 5n‬‬
‫‪1‬‬

‫) ‪.PGCD(5n+2 − 5n , 7n+2 − 7n‬‬

‫⃝ ‪ y ،x‬و ‪ z‬أعداد طبيعية حيث ‪x + y = 2z :‬‬


‫‪2‬‬

‫)ا( أثبت أن ‪.2x × 2y = 4z‬‬


‫)ب( أثبت أن ‪.22x × 22y+2 × 22z = (23z+1 )2‬‬
‫√‬ ‫√‬ ‫√‬
‫√‬ ‫‪a+c‬‬ ‫‪a−c‬‬
‫= ‪ a + b‬أثبت أن‪.b = a − c :‬‬
‫‪2‬‬ ‫‪2‬‬
‫‪+‬‬ ‫⃝ ‪ b ،a‬و ‪ c‬ثلاث أعداد طبيعية‪ ،‬علما أن‬
‫‪3‬‬
‫‪2‬‬ ‫‪2‬‬

‫التمرين الثالث‪:‬‬
‫‪D‬‬

‫)‪ (C‬دائرة مركزها ‪ O‬و نصف قطرها ‪ r‬كما هو موضح في الشكل‪.‬‬


‫‪A‬‬ ‫‪C‬‬ ‫علما أن ‪ 3.14 ⩽ π ⩽ 3.15‬و ‪.1.1 ⩽ r ⩽ 1.21‬‬
‫‪O‬‬ ‫⋆ عين حصرا لمساحة الجزء الملون بالأسود‪.‬‬

‫‪B‬‬

‫ومن طلب العلا سهر الليالي‬ ‫بقدر الـكـد تكتسب المعـالي‬


‫‪2x × 2y = 2x+y = 22z = 4z‬‬ ‫تصحيح الفرض الأول في مادة الر ياضيات سنة أولى ج م ع ت‬

‫إثبات أن ‪:22x × 22y+2 × 22z = (23z+1 )2‬‬ ‫‪3‬‬


‫ـ )‪(1‬‬
‫‪22x × 22y+2 × 22z = 22x+2y+2+2z‬‬
‫‪= 26z+2 = 22(3z+1) = (23z+1 )2‬‬ ‫حساب الفرق ‪A − B‬‬ ‫‪1‬‬
‫‪x‬‬ ‫‪x−1‬‬
‫إثبات أن ‪b = a2 − c2 :‬‬ ‫‪4‬‬ ‫=‪A−B‬‬ ‫‪−‬‬
‫‪x+1‬‬ ‫‪x‬‬
‫√‬ ‫√‬ ‫√‬
‫√‬ ‫‪a+c‬‬ ‫‪a−c‬‬ ‫)‪x2 − (x2 − 1‬‬
‫=‪a+ b‬‬ ‫‪+‬‬ ‫لدينا ‪:‬‬ ‫=‬
‫‪2‬‬ ‫‪2‬‬ ‫)‪x(x + 1‬‬
‫بتربيع الطرفين نجد‬ ‫=‬
‫‪1‬‬
‫)‪x(x + 1‬‬
‫√‬ ‫‪2‬‬ ‫√‬ ‫‪2‬‬ ‫√‬
‫√‬ ‫‪a+c‬‬ ‫‪a−c‬‬ ‫)‪(a + c)(a − c‬‬
‫=‪a+ b‬‬ ‫‪+‬‬ ‫‪−2‬‬ ‫إشارة ‪A − B‬‬ ‫‪2‬‬
‫‪2‬‬ ‫‪2‬‬ ‫‪4‬‬
‫√‬
‫=‬
‫‪a+c‬‬
‫‪+‬‬
‫‪a−c‬‬
‫‪−2‬‬
‫)‪(a − c)(a + c‬‬ ‫لدينا ‪ x‬عدد موجب تماما أي ‪ x ≻ 0‬و عليه ‪ x+1 ≻ 0‬بما أن جميع‬
‫‪2‬‬ ‫‪2‬‬ ‫‪2‬‬ ‫‪1‬‬
‫√‬ ‫‪2 √ 2‬‬ ‫الأطراف موجبة فإن ‪ x(x + 1) ≻ 0‬و منه ‪≻ 0‬‬
‫‬
‫= ‪a + b‬‬‫‪a−‬‬ ‫‪a − c2‬‬ ‫)‪x(x + 1‬‬
‫‪2‬‬
‫‬ ‫إذا ‪ A − B ≻ 0‬و منه ‪A ≻ B‬‬
‫√ ‪√ 2‬‬ ‫‪2‬‬
‫‪b = a2 − c2‬‬ ‫بالتربيع مرة أخرى نجد‬ ‫√‬ ‫√‬
‫‪2018‬‬ ‫‪2019‬‬
‫و‬ ‫مقارنة العددين‬ ‫‪3‬‬
‫‪b = a2 − c2‬‬ ‫و منه‬ ‫‪2019‬‬ ‫‪2020‬‬
‫من السؤال )‪ (1‬من أجل ‪ x = 2019‬نجد‬
‫‪x − 1 2018‬‬ ‫‪x‬‬ ‫‪2019‬‬
‫=‪B‬‬ ‫=‬ ‫=‪ A‬و‬ ‫=‬
‫‪x‬‬ ‫‪2019‬‬ ‫‪x + 1 2020‬‬
‫ـ )‪(3‬‬ ‫‪2019‬‬ ‫‪2018‬‬
‫بالمرور إلى الجذر نجد‬ ‫≻‬ ‫حيث ‪ A ≻ B‬و عليه‬
‫‪2020‬‬ ‫√ ‪2019‬‬ ‫√‬
‫نسمي ‪ S‬مساحة الجزء الملون بالأسود ‪ S1 ،‬مساحة الدائرة و ‪ S2‬مساحة‬ ‫‪2019‬‬ ‫‪2018‬‬
‫≻‬
‫المربع‬ ‫‪2020‬‬ ‫‪2019‬‬

‫‪S = S1 − S2‬‬ ‫و عليه‬


‫‪S1 = πr2‬‬ ‫إ يجاد حصر لـ ‪S1‬‬
‫حصر ‪r2‬‬ ‫ـ )‪(2‬‬
‫لدينا ‪ 1.1 ≤ r ≤ 1.21‬بالمرور إلى التربيع نجد ‪1.21 ≤ r ≤ 1.4641‬‬
‫‪2‬‬
‫تحليل العددين إلى جداء عوامل أولية ‪:‬‬ ‫‪1‬‬
‫و لدينا ‪3.14 ≤ π ≤ 3.15‬‬
‫≤ ‪1.21 × 3.14 ≤ πr2‬‬ ‫بما أن جميع الأطراف موجبة نجد‬
‫‪1.4641 × 3.15‬‬
‫‪5n + 2 − 5n = 5n × 52 − 5n‬‬
‫)‪3.90 ≤ S1 ≤ 4.61 · · · · (1‬‬ ‫و منه‬
‫)‪= 5n (52 − 1‬‬
‫‪S2 = AD‬‬ ‫‪2‬‬
‫إ يجاد حصر لـ ‪S2‬‬
‫)‪= 5n (5 − 1)(5 + 1‬‬
‫حساب ‪AD2‬‬
‫‪AD2 = 2r2‬‬ ‫بما أن ‪ AOD‬مثلث قائم حسب فيثاغورس نجد‬ ‫)‪= 5n (4 × 6‬‬
‫لدينا من قبل ‪ 1.21 ≤ r2 ≤ 1.4641‬و عليه‬ ‫‪= 5n × 23 × 3‬‬
‫‪2.42 ≤ 2r2 ≤ 2.93‬‬
‫‪2.42 ≤ AD2 ≤ 2.93‬‬ ‫أي‬
‫)‪2.42 ≤ S2 ≤ 2.93 · · · · (2‬‬ ‫ومنه‬
‫‪7n + 2 − 7n = 7n × 72 − 7n‬‬
‫) ‪S = S1 + (−S2‬‬ ‫‪ S = S1 − S2‬أي‬ ‫نعلم أن‬
‫)‪= 7n (72 − 1‬‬
‫بضرب طرفي )‪ (2‬في العدد )‪ (−1‬نجد‬
‫)‪−2.93 ≤ −S2 ≤ −2.42 · ·(3‬‬ ‫)‪= 7n (7 − 1)(7 + 1‬‬

‫بجمع )‪ (1‬و )‪ (3‬طرف لطرف نجد‬ ‫)‪= 7n (6 × 8‬‬


‫‪−2.93 + 3.90 ≤ S1 + (−S2 ) ≤ −2.42 + 4.61‬‬ ‫‪= 7n × 24 × 3‬‬
‫‪0.97 ≤ S ≤ 2.19‬‬ ‫و عليه‬
‫و منه ‪PGCD(5n +2−5n ; 7n +2−7n ) = 3×23 = 24‬‬

‫إثبات أن ‪2x × 2y = 4z‬‬ ‫‪2‬‬

‫‪1‬‬
‫السّنة الدّراسيّة‪8302/8302 :‬‬ ‫ثانويّة سعدي الصديق ‪ -‬تبسة ‪-‬‬

‫رقم الواجب‪10 :‬‬ ‫املستوى‪ :‬أوىل جذع مشرتك علوم و تكنولوجيا‬

‫التّمرين األوّل‪:‬‬

‫*‬ ‫‪n‬‬
‫‪.n‬‬ ‫ليس عدداً ناطقاً حيث‬ ‫‪ )1‬أثبت أنّ العدد‬
‫‪n‬‬ ‫‪2‬‬
‫‪a 2 b2‬‬
‫‪ 2‬ليس عدداً طبيعيّاً‪.‬‬ ‫‪ . a‬أثبت أنّ‬ ‫‪ )2‬ليكن ‪ a‬و ‪ b‬عددين طبيعيّني غري معدومني حيث ‪b‬‬
‫‪a b2‬‬
‫‪ 3n 3‬عدداً طبيعيّاً زوجيّاً‪.‬‬ ‫‪ )3‬أثبت أنّه من أجل كل عدد طبيعي ‪ n‬يكون العدد ‪n‬‬
‫‪679‬‬
‫عشري (دون استعمال اآللة احلاسبة)‪.‬‬ ‫‪ )4‬أثبت أنّ العدد‬
‫‪800‬‬
‫التّمرين الّثاني‪:‬‬
‫‪ )1‬أُكتُب كُالًّ من العددين ‪ x 7,23‬و ‪ y 5,243‬على شكل كسر‪.‬‬
‫‪.z‬‬ ‫‪ )2‬استنتج الكسر الذي كتابته العشريّة ‪ w 59,66‬و الكسر الذي كتابته العشريّة ‪45,20‬‬

‫التّمرين الّثالث‪:‬‬
‫‪.x‬‬ ‫‪2016‬‬ ‫‪ . xy‬أثبت أنّ ‪y‬‬ ‫‪ x‬و ‪2016‬‬ ‫‪ )1‬ليكن ‪ x‬و ‪ y‬عددين حقيقيّني موجبني متاماً حيث ‪y‬‬
‫‪1‬‬ ‫‪1‬‬ ‫‪1‬‬ ‫‪1‬‬
‫‪.‬‬ ‫‪.....‬‬ ‫‪ )2‬أُحسُب اجملموع‪:‬‬
‫‪1‬‬ ‫‪2‬‬ ‫‪2‬‬ ‫‪3‬‬ ‫‪3‬‬ ‫‪4‬‬ ‫‪2015‬‬ ‫‪2016‬‬
‫التّمرين ال ّرابع‪:‬‬
‫‪ n‬عدداً طبيعيّاً‪ .‬نضع )‪. p n(n 3‬‬ ‫ليكن‬
‫أُكتُب )‪ (n 1)(n 2‬بداللة ‪. p‬‬ ‫‪)1‬‬
‫أُكتُب )‪ n(n 1)(n 2)(n 3‬بداللة ‪. p‬‬ ‫‪)2‬‬
‫استنتج أنّ ‪ n(n 1)(n 2)(n 3) 1‬مربّع تام‪.‬‬ ‫‪)3‬‬
‫تطبيق عددي‪ :‬ما هو العدد الذي مُربّعه ‪. 2014 2015 2016 2017 1‬‬ ‫‪)4‬‬

‫تاريخ اإلستالم‪ 32 :‬أكتوبر ‪8302‬‬ ‫تاريخ التّسليم‪ 30 :‬أكتوبر ‪8302‬‬


‫التّمرين األوّل‪:‬‬
‫‪n‬‬ ‫)‪n(n 2‬‬
‫‪ n‬طبيعيي موجيب متاميًا‬ ‫‪ .‬العيدد ‪2‬‬ ‫‪ )1‬من أجل كل عدد طبيعيي غيري معيدوم ‪ n‬ليدينا‬
‫‪n‬‬ ‫‪2‬‬ ‫‪n 2‬‬
‫‪ n(n‬عيدداً‬ ‫‪ n(n‬وعلييه يكيون العيدد )‪2‬‬ ‫)‪2‬‬ ‫‪(n‬‬ ‫‪1)2‬‬ ‫‪ n(n‬ليس مُربّعاً تامّياً ننّ ‪1‬‬ ‫لكن العدد )‪2‬‬
‫‪n‬‬
‫غري ناطق‪.‬‬ ‫أصمًّا ومنه يكون العدد‬
‫‪n‬‬ ‫‪2‬‬
‫‪a2‬‬ ‫‪b2‬‬
‫‪ a‬أي أنّييه يُوجييد عييدد طبيعييي غييري معييدوم ‪k‬‬ ‫‪2‬‬ ‫‪2‬‬
‫‪ a‬يقسييم ‪b‬‬ ‫‪2‬‬ ‫‪2‬‬
‫‪ 2‬ومنييه ‪b‬‬ ‫نضييرّ أنّ‬ ‫‪ )2‬بييالُل‬
‫‪a‬‬ ‫‪b2‬‬
‫‪k 1 2‬‬
‫‪ b 2‬نُميّز احلاالت اآلتية‪:‬‬ ‫‪ (k 1)a 2 (k‬و منه ‪a‬‬ ‫‪ a 2‬أي ‪1)b 2‬‬ ‫‪b2‬‬ ‫‪k(a 2‬‬ ‫حبيث ) ‪b 2‬‬
‫‪k 1‬‬
‫‪ b‬و هيذا تنياقم ميع كيون العيدد ‪ b‬غيري معيدوم و منيه يكيون العيدد‬ ‫‪ b 2‬و منه ‪0‬‬ ‫‪ k‬فإنّ ‪0‬‬ ‫أ) إذا كان ‪1‬‬
‫‪a 2 b2‬‬
‫‪ 2‬غري طبيعي‪.‬‬
‫‪a b2‬‬
‫‪k‬‬ ‫‪1‬‬ ‫‪k‬‬ ‫‪1‬‬ ‫‪k‬‬ ‫‪1 2‬‬
‫عييييدد أص يييم ف يييإنّ ‪b‬‬ ‫‪ b‬و مبي ييا أنّ‬ ‫‪a‬‬ ‫‪ b 2‬تعييييّ أنّ‬ ‫‪ k‬فييييإنّ ‪a‬‬ ‫ب) إذا كييييان ‪1‬‬
‫‪k‬‬ ‫‪1‬‬ ‫‪k‬‬ ‫‪1‬‬ ‫‪k‬‬ ‫‪1‬‬
‫‪a2‬‬ ‫‪b2‬‬
‫‪ 2‬غري طبيعي‪.‬‬ ‫كذلك و هذا تناقم مع كون العدد ‪ b‬طبيعي و منه يكون العدد‬
‫‪a‬‬ ‫‪b2‬‬
‫‪ )3‬نُميّز حالتني للعدد الطّبيعي ‪: n‬‬
‫أ) إذا كان ‪ n‬فرديّاً أي ‪ n 2k 1‬حيث ‪ k‬عدد طبيعي‪.‬‬
‫‪ 3n 3‬ومنيييييه يكي يييون العيييييدد‬ ‫‪n‬‬ ‫ليييييدينا )‪3(2k 1)3 (2k 1) 2(12k 3 18k 2 10k 2‬‬
‫‪ 3n 3 n‬زوجيّاً‪.‬‬
‫ب) إذا كان ‪ n‬زوجيّاً أي ‪ n 2k‬حيث ‪ k‬عدد طبيعي‪.‬‬
‫‪3n 3‬‬ ‫‪ 3n 3‬ومني ييه يكي ييون العييييدد ‪n‬‬ ‫‪n‬‬ ‫لييييدينا ) ‪3(2k )3 (2k ) 24k 2 2k 2(12k 2 k‬‬
‫زوجيّاً‪.‬‬
‫‪679‬‬ ‫‪679‬‬ ‫‪679‬‬
‫عشري‪.‬‬ ‫ومنه العدد‬ ‫‪ )4‬لدينا‬
‫‪800‬‬ ‫‪800 25 52‬‬

‫التّمرين الثاني‪:‬‬
‫‪ 100a‬و منيييييييييه‬ ‫‪23‬‬ ‫‪0,23‬‬ ‫‪23‬‬ ‫‪ a‬ومنيي ي ي ييه ‪a‬‬ ‫‪ )1‬لييييي يييدينا ‪ . x 7,23 7 0,23‬نضيييييييييع ‪0,23‬‬
‫‪716‬‬ ‫‪23‬‬
‫‪.x‬‬ ‫‪7‬‬ ‫‪a‬‬ ‫‪ a‬و منه نستنتج أنّ‬ ‫‪ 99a 23‬و منه‬
‫‪99‬‬ ‫‪99‬‬
‫‪43‬‬
‫‪ a‬ومنيييه‬ ‫‪ .Y‬ب يينضس الطريق يية جن ييد أنّ‬ ‫‪10y‬‬ ‫‪52, 43‬‬ ‫‪52‬‬ ‫‪ y‬و منيييه ‪0, 43‬‬ ‫ليييدينا ‪5,243‬‬
‫‪99‬‬
‫‪5191‬‬ ‫‪5191‬‬
‫‪.y‬‬ ‫‪ Y‬و منه‬ ‫‪10y‬‬ ‫‪52‬‬ ‫‪a‬‬ ‫نستنتج أنّ‬
‫‪990‬‬ ‫‪99‬‬
‫‪0‬‬
‫‪4475‬‬ ‫‪5907‬‬
‫‪.z‬‬ ‫‪45,20‬‬ ‫‪10y‬‬ ‫‪x‬‬ ‫‪ w‬و لدينا‬ ‫‪59, 66‬‬ ‫‪x‬‬ ‫‪10y‬‬ ‫‪ )2‬لدينا‬
‫‪99‬‬ ‫‪99‬‬
‫التّمرين الثالث‪:‬‬
‫‪ x 2‬و منييييه‬ ‫‪ xy‬و ك يييذلك ‪xy‬‬ ‫‪ x‬ومني ييه ‪y 2‬‬ ‫‪ )1‬ليييييكن ‪ x‬و ‪ y‬عييييددين حقيقيي ييّني مييييوجبني متاميي ياً حيييييث ‪y‬‬
‫‪.x‬‬ ‫‪2016‬‬ ‫‪ x 2‬و منه ‪y‬‬ ‫‪xy‬‬ ‫‪ x 2‬و منه ‪y 2‬‬ ‫‪xy‬‬ ‫‪y2‬‬
‫‪1‬‬ ‫‪1‬‬ ‫‪1‬‬ ‫‪1‬‬
‫‪ S‬و منييييييييييييييييييييييييييييه‬ ‫‪.....‬‬ ‫‪ )2‬نضييييييييييييييييييييييييييييع‬
‫‪1‬‬ ‫‪2‬‬ ‫‪2‬‬ ‫‪3‬‬ ‫‪3‬‬ ‫‪4‬‬ ‫‪2015‬‬ ‫‪2016‬‬
‫‪1‬‬ ‫‪2‬‬ ‫‪2‬‬ ‫‪3‬‬ ‫‪2015‬‬ ‫‪2016‬‬
‫‪ S‬و منييييييييه‬ ‫‪.....‬‬
‫‪1‬‬ ‫‪2 1‬‬ ‫‪2‬‬ ‫‪2‬‬ ‫‪3‬‬ ‫‪2‬‬ ‫‪3‬‬ ‫‪2015‬‬ ‫‪2016‬‬ ‫‪2015‬‬ ‫‪2016‬‬

‫‪1‬‬ ‫‪2‬‬ ‫‪2‬‬ ‫‪3‬‬ ‫‪2015‬‬ ‫‪2016‬‬


‫‪.S‬‬ ‫‪2016‬‬ ‫‪ S‬و منه ‪1‬‬ ‫‪.....‬‬
‫‪1‬‬ ‫‪1‬‬ ‫‪1‬‬

‫التّمرين الرّابع‪:‬‬
‫‪. (n‬‬ ‫‪1)(n‬‬ ‫)‪2‬‬ ‫‪n2‬‬ ‫‪3n‬‬ ‫‪2‬‬ ‫‪p‬‬ ‫‪ )1‬لدينا ‪2‬‬
‫‪. n(n‬‬ ‫‪1)(n‬‬ ‫‪2)(n‬‬ ‫)‪3‬‬ ‫‪p(p‬‬ ‫‪ )2‬لدينا )‪2‬‬
‫‪ n(n‬و منييييييييييييييييييييييييييييييييييييييييييييييه‬ ‫‪1)(n‬‬ ‫‪2)(n‬‬ ‫)‪3‬‬ ‫‪1‬‬ ‫‪p(p‬‬ ‫‪ )3‬لييييييييييييييييييييييييييييييييييييييييييييييدينا ‪2) 1 (p 1)2‬‬
‫‪ n(n 1)(n 2)(n 3) 1‬مُربّع تام‪.‬‬
‫‪ (p‬إذن جي ي يييذر هيييي ييو ‪ p‬حيييييييييث‬ ‫‪ )4‬ليييي ييدينا ‪ 2014 2015 2016 2017 1‬يُكتيييييييب عليييييييى الشّ ي ي ييكل ‪1)2‬‬
‫‪. p 2014 2017 4 062238‬‬

‫‪8‬‬
2013 2012 2013 2012
1 5 1 1 5 1
10 10
A=− (7 + )(
48 7 − 48 ) 1 A=− (7 + )(
48 7 − 48 ) 1
9 2 − 8 9 2 − 8
C= B= C= B=
400  −4 400  −4
4 2 3 4 2 3
7 4  2  7 4  2 
D D =       2 D D =       2
 2   7   −49   2   7   −49 
B = −0.0000012 A = 108 1012 3 B = −0.0000012 A = 108 1012 3
A A
A B A B
B B
378 378
378 420 4 378 420 4
420 420
10 10
4−2 3 3 −1 -1 4−2 3 3 −1 -1
1+
a
a +1 a0 -2 1+
a
a +1 a0 -2
2 2
2 +1 2 +1
2+2 1+ 2+2 1+
2 2
2a − b 2a − b
0  b 1 1 a  3 A= A -3 0  b 1 1 a  3 A= A -3
a2 a2
10
‫ن‬3 -1

2 − 8
(7 + )( )
9
C= B= A=− 48 7 − 48
400  −4
2 ( − 4 )
3
(7) ( )
2
A=− −
2
C= B= 48
20  −4
C  ID B=2 A = −1
‫ن‬2 B A

D -2
4 2 3
7 4  2 
D =      
 2   7   −49 
 7 4 2 4 23 
D = − 4  2  6  •
2 7 7 
23 8
D=− 4 =−
7 2401
D •
‫ن‬2
A
A B -3
B
B = −1.2 10−6 A = 1.08 1014 B A
−110−6 11014 B A
A B
−1108 A  B 11014  −110−6 = −1108
A
B
A 110 14
−11020 = −11020
B −110−6
‫ن‬3 378
420 378 -4
420
420 = 22  3  5  7 378 = 2  33  7 •
378 23  73
3 9 2
= 2 = = •
420 2  3  5  7 2  5 10
10

‫ن‬3 4 − 2 3 ‫ و‬1− 3 -1

( ) ( )
2 2
3 −1 = 4 − 2 3 3 −1 = 4−2 3

a
‫ن‬4 1+ a +1 -2
2
2
a
( )  a
2
a +1 1+ a +1 1 +  •
2  2
2 +1
2+2 1+ a = 2 +1 a •
2
2a − b
0  b 1 1 a  3 A= A -3
a2
‫ن‬3
1  2a − b  6

1 1
 1 ‫و‬
9 a2

1 2a − b
 6
9 a2
‫ ا‬‫اط‬‫ ا‬‫ا‬‫ ا‬‫ر‬‫ا‬
2019 / 2018 :‫را‬‫ ا‬‫ا‬ ‫ي‬  ‫م‬
2018 / 10 / 24 :  ‫ ع‬ ‫ ج‬1: ‫ى‬‫ا‬
‫عم‬ 01 ‫اﻟﻔﺮض رﻗﻢ‬
(‫ ن‬03) : ‫ول‬‫ ا‬‫ا‬
. ‫وم‬ ‫د‬ n . B  378  n ‫ و‬A  3m 3 3m : B ‫و‬A ‫د‬ ‫ا‬
0.5 . ‫د‬ B ‫ن‬ n ‫د‬ ‫ا‬ -1

0.5 A  3 m 22  7 ‫أن‬ -2


. n 168 ‫ و‬m 3 ‫ا‬ -3
0.75 PPCM (A; B ) ‫ و ا‬PGCD(A; B ) ‫ا‬ ‫أو‬ ‫ا‬ ‫اء‬ ‫ إ‬B‫د‬ ‫ا‬ -‫أ‬
2 189  3 2
0.75 ‫ا‬ C ‫د‬ ‫ا‬ ‫إ‬ ‫ا‬ . C  :‫د‬ ‫ ا‬-‫ب‬
252 756
B
0.5 ‫إ‬ ‫ا‬ . ‫لا‬ ‫ ا‬-‫ج‬
A

(‫ ن‬05) : ‫م‬‫ ا‬‫ا‬


: D ‫ و‬C , B , A ‫اد‬ ‫ا‬
2
1 1  8 
12  75 (4)
4 9
10 10 
8 4 a ²  b² 
 

.C  ( 3  2)2  4  2 3 , D  , B a b  , A
1 1 (a  b)²
84  411
 
4
 252 186 104
a b
01 A ‫د‬ ‫ا‬ -1

03 ‫د‬ C ‫د‬ ‫ و ا‬D  16 ; B  1 ‫د‬ ‫أن ا‬ -2


0.5 E  D  103 E‫د‬ ‫ار ا‬ ‫ر‬ -3
0.5 2  3  2  3  6 :‫أن‬ -4

(‫ ن‬02) : ‫ ا‬‫ا‬


‫ن‬ ‫ن‬ ‫دان‬ y ‫ و‬x
1 1
01 2 2
 ‫أن‬ -1
x y 2x .y
x y 1  1 1
    ‫أن‬ ‫ ا‬-2
01 x 2  y2 2  x y 

 1/1 ‫ص‬


‫ ا‬‫اط‬‫ ا‬‫ا‬‫ ا‬‫ر‬‫ا‬
2019 / 2018 :‫را‬‫ ا‬‫ا‬ ‫ي‬  ‫م‬
2018 / 10 / 24 :  ‫ ع‬ ‫ ج‬1: ‫ى‬‫ا‬
‫عم‬ 01 ‫اﻟﻔﺮض رﻗﻢ‬
(‫ ن‬03) : ‫ول‬‫ ا‬‫ا‬
. ‫وم‬ ‫د‬ n . B  378  n ‫ و‬A  3m 3 3m : B ‫و‬A ‫د‬ ‫ا‬
0.5 . ‫د‬ B ‫ن‬ n ‫د‬ ‫ا‬ -1

0.5 A  3 m 22  7 ‫أن‬ -2


. n 168 ‫ و‬m 3 ‫ا‬ -3
0.75 PPCM (A; B ) ‫ و ا‬PGCD(A; B ) ‫ا‬ ‫أو‬ ‫ا‬ ‫اء‬ ‫ إ‬B‫د‬ ‫ا‬ -‫أ‬
2 189  3 2
0.75 ‫ا‬ C ‫د‬ ‫ا‬ ‫إ‬ ‫ا‬ . C  :‫د‬ ‫ ا‬-‫ب‬
252 756
B
0.5 ‫إ‬ ‫ا‬ . ‫لا‬ ‫ ا‬-‫ج‬
A

(‫ ن‬05) : ‫م‬‫ ا‬‫ا‬


: D ‫ و‬C , B , A ‫اد‬ ‫ا‬
2
1 1  8 
12  75 (4)
4 9
10 10 
8 4 a ²  b² 
 

.C  ( 3  2)2  4  2 3 , D  , B a b  , A
1 1 (a  b)²
84  411
 
4
 252 186 104
a b
01 A ‫د‬ ‫ا‬ -1

03 ‫د‬ C ‫د‬ ‫ و ا‬D  16 ; B  1 ‫د‬ ‫أن ا‬ -2


0.5 E  D  103 E‫د‬ ‫ار ا‬ ‫ر‬ -3
0.5 2  3  2  3  6 :‫أن‬ -4

(‫ ن‬02) : ‫ ا‬‫ا‬


‫ن‬ ‫ن‬ ‫دان‬ y ‫ و‬x
1 1
01 2 2
 ‫أن‬ -1
x y 2x .y
x y 1  1 1
    ‫أن‬ ‫ ا‬-2
01 x 2  y2 2  x y 

 1/1 ‫ص‬


‫ ا‬‫اط‬‫ ا‬‫ا‬‫ ا‬‫ر‬‫ا‬
-‫ي –ب ب ع‬  ‫ر‬‫ ا‬‫م‬ ‫ط‬‫ ا‬‫وزارة ا‬
2019 – 2018 :‫را‬‫ ا‬‫ا‬ 01 ‫ض ر‬‫ ا‬

01: ‫ت‬‫د ا‬ ‫ت‬‫ ر‬: ‫دة‬‫ا‬ 06 ‫ ع‬ ‫ ج‬01 : ‫ى‬‫ا‬


‫ا‬ ‫ا‬ ‫ا‬ ‫ا‬

‫د‬ C ‫ و ا د‬D  16 ; B  1 ‫ ن أن ا د‬-2 (‫ ن‬03) : ‫ول‬‫ ا‬‫ا‬


. B 378  n ‫ و‬A  3m 3 3m
01
1 1 b a . ‫د‬ B ‫ن‬ n ‫د‬ ‫ ا‬-1

a ²  b ² (a  b)(a  b)
Ba b   ab 
1 1 (a  b)² a  b (a  b)² n ‫د‬ ‫ا‬ ‫و‬B  2  33  7  n
 0.5
a b ab n  2  3  7  42 . ‫د‬ B ‫ن‬
b  a a  b b  a a  b a  b
      1 A  3 m 22  7 ‫ ن أن‬-2
a  b a  b a  b a b a b
A  3m 3 3m  3m  33  3m
810  410 230  220 0.5
D 4  12
01 8  411 2  222  
 3m  33  1  3m  28  3m  22  7


28 222  212  24  16
 . n 168 ‫ و‬m 3 ‫ا‬ -3
22
2 2 12
‫أو‬ ‫ا‬ ‫اء‬ ‫ إ‬B‫د‬ ‫ا‬ -‫أ‬

 
2

01 C  ( 3 2)2  4 2 3 2 3  1 3 0.25 B  2  33  7  168  2  33  7  23  3  7


 24  34  72  22  32  7
2 3 1 3  3
E  D  103 E ‫ار ا د‬ ‫ر‬-3
A  3 322  7
0.25 PGCD(A; B )  22  32  7  252  B ‫ب‬
‫ار‬ ‫ر‬ ‫ و‬E  D  103  16  103  1.6  104
0.5 PPCM (A; B )  3 322  7  756  A ‫ج‬ ‫ا‬
2  104 E‫ا د‬
0.25
2  3  2  3  6 :‫ن أن‬ -4 2 189  3 2
. C   :‫د‬ ‫ ا‬-‫ب‬
2 252 756
 
 2  3  2  3   2  3  2  3  2  6 2 189  3 2
  C   2
0.5 2  3  7 2  33  7
22

3 2 189  3 2 33  7 1
0.5 2 3  2 3  6 ‫و‬  2 3
 2 3
 2 3
 2
2 3 7 2 3 7 2 3 7 2
(‫ ن‬02) : ‫ ا‬‫ا‬ ‫ا‬ C ‫ا د‬ ‫إ‬ ‫ا‬
‫ن‬ ‫ن‬ ‫دان‬ y ‫ و‬x 0.25 ‫ال و‬ ‫ب‬ C D
1 1 ‫ا‬
 ‫أن‬ -1 2
2
x y 2
2x .y 2 2
0.25  B   2  3  7   1 .  B ‫ال ا‬ ‫ ا‬-‫ج‬
x  y 
2
‫و‬ x 2  y 2  2x .y  0 ‫و‬ 0 : A 3 2
3 2  7 3 A
01 1 1 2 B
2 2
 ‫و‬x  y 2  2x .y 0.25   ‫إ‬ ‫ا‬
x y 2x .y A
x y 1  1 1  (‫ ن‬05) : ‫م‬‫ ا‬‫ا‬
    ‫أن‬ ‫ ا‬-2
x 2  y2 2  x y  A ‫ا د‬ -1
2
x y x y 1 1  8 
12  75 (4) 2 3 35  (2 )
4 9 16 4
‫و‬2 2
 ‫و‬   
2 2 2 9

x y 2x .y 2
x y 2
2x .y A 
25  18 10 5  23  25
4 8 6 4
2 6 4 2 2

x y 1  1 1  x y 1 y x 
01    
 ‫ و‬2     232 316 34 58 (218) 214 312 58
x 2  y2 2  x y  x y 2
2  x.y x.y  01 
516 26 312 24 54
 10 12 12
2 3 5
 24 54 10000
‫ ا‬‫اط‬‫ ا‬‫ا‬‫ ا‬‫ر‬‫ا‬
-‫ي –ب ب ع‬  ‫ر‬‫ ا‬‫م‬ ‫ط‬‫ ا‬‫وزارة ا‬
2019 – 2018 :‫را‬‫ ا‬‫ا‬ 01 ‫ض ر‬‫ ا‬

01: ‫ت‬‫د ا‬ ‫ت‬‫ ر‬: ‫دة‬‫ا‬ 06 ‫ ع‬ ‫ ج‬01 : ‫ى‬‫ا‬


‫ا‬ ‫ا‬ ‫ا‬ ‫ا‬

‫د‬ C ‫ و ا د‬D  16 ; B  1 ‫ ن أن ا د‬-2 (‫ ن‬03) : ‫ول‬‫ ا‬‫ا‬


. B 378  n ‫ و‬A  3m 3 3m
01
1 1 b a . ‫د‬ B ‫ن‬ n ‫د‬ ‫ ا‬-1

a ²  b ² (a  b)(a  b)
Ba b   ab 
1 1 (a  b)² a  b (a  b)² n ‫د‬ ‫ا‬ ‫و‬B  2  33  7  n
 0.5
a b ab n  2  3  7  42 . ‫د‬ B ‫ن‬
b  a a  b b  a a  b a  b
      1 A  3 m 22  7 ‫ ن أن‬-2
a  b a  b a  b a b a b
A  3m 3 3m  3m  33  3m
810  410 230  220 0.5
D 4  12
01 8  411 2  222  
 3m  33  1  3m  28  3m  22  7



28 222  212
 24  16
 . n 168 ‫ و‬m 3 ‫ا‬ -3
22
2 2 12
‫أو‬ ‫ا‬ ‫اء‬ ‫ إ‬B‫د‬ ‫ا‬ -‫أ‬

 
2

01 C  ( 3 2)2  4 2 3 2 3  1 3 0.25 B  2  33  7  168  2  33  7  23  3  7


 24  34  72  22  32  7
2 3 1 3  3
E  D  103 E ‫ار ا د‬ ‫ر‬-3
A  3 322  7
0.25 PGCD(A; B )  22  32  7  252  B ‫ب‬
‫ار‬ ‫ر‬ ‫ و‬E  D  103  16  103  1.6  104
0.5 PPCM (A; B )  3 322  7  756  A ‫ج‬ ‫ا‬
2  104 E‫ا د‬
0.25
2  3  2  3  6 :‫ن أن‬ -4 2 189  3 2
. C   :‫د‬ ‫ ا‬-‫ب‬
2 252 756
 
 2  3  2  3   2  3  2  3  2  6 2 189  3 2
  C   2
0.5 2  3  7 2  33  7
22

3 2 189  3 2 33  7 1
0.5 2 3  2 3  6 ‫و‬  2 3
 2 3
 2 3
 2
2 3 7 2 3 7 2 3 7 2
(‫ ن‬02) : ‫ ا‬‫ا‬ ‫ا‬ C ‫ا د‬ ‫إ‬ ‫ا‬
‫ن‬ ‫ن‬ ‫دان‬ y ‫ و‬x 0.25 ‫ال و‬ ‫ب‬ C D
1 1 ‫ا‬
 ‫أن‬ -1 2
2
x y 2
2x .y 2 2
0.25  B   2  3  7   1 .  B ‫ال ا‬ ‫ ا‬-‫ج‬
x  y 
2
‫و‬ x 2  y 2  2x .y  0 ‫و‬ 0 : A 3 2
3 2  7 3 A
01 1 1 2 B
2 2
 ‫و‬x  y 2  2x .y 0.25   ‫إ‬ ‫ا‬
x y 2x .y A
x y 1  1 1  (‫ ن‬05) : ‫م‬‫ ا‬‫ا‬
    ‫أن‬ ‫ ا‬-2
x 2  y2 2  x y  A ‫ا د‬ -1
2
x y x y 1 1  8 
12  75 (4) 2 3 35  (2 )
4 9 16 4
‫و‬2 2
 ‫و‬   
2 2 2 9

x y 2x .y 2
x y 2
2x .y A 
25  18 10 5  23  25
4 8 6 4
2 6 4 2 2

x y 1  1 1  x y 1 y x 
01    
 ‫ و‬2     232 316 34 58 (218) 214 312 58
x 2  y2 2  x y  x y 2
2  x.y x.y  01 
516 26 312 24 54
 10 12 12
2 3 5
 24 54 10000
‫اﻟﺠﻤﮭﻮرﯾﺔ اﻟﺠﺰاﺋﺮﯾﺔ اﻟﺪﯾﻤﻘﺮاﻃﯿﺔ اﻟﺸﻌﺒﯿﺔ‬
‫ﻣﺪﯾﺮﯾﺔ اﻟﺘﺮﺑﯿﺔ ﻟﻮﻻﯾﺔ ﺑـــــــــــﺎﺗﻨﺔ‬ ‫وزارة اﻟﺘﺮﺑﯿﺔ اﻟﻮﻃﻨﯿﺔ‬
‫اﻟﻤﺴﺘﻮى ‪:‬اﻷوﻟﻰ ﺟﺬع ﻣﺸﺘﺮك ﻋﻠﻮم ‪ .‬اﻟﺴﻨﺔ اﻟﺪراﺳﯿﺔ‪2019 /2018:‬‬
‫اﻟﻮﻇﯿﻔﺔ اﻟﻤﻨﺰﻟﯿﺔ‬
‫رﻗﻢ)‪(1)(1‬‬ ‫اﻟﺘﻤﺮﯾﻦ اﻷول‪:‬‬
‫ﺑﺴّﻂ اﻷﻋﺪاد اﻟﺘﺎﻟﯿﺔ ‪،‬ﺛﻢ أذﻛﺮ أﺻﻐﺮ ﻣﺠﻤﻮﻋﺔ ﺗﻨﺘﻤﻲ إﻟﯿﮭﺎ‪:‬‬

‫‪2  4‬‬
‫‪‬‬ ‫‪‬‬
‫‪4‬‬
‫‪38‬‬ ‫‪،‬‬ ‫‪52 6  52 6‬‬ ‫‪،‬‬ ‫‪،‬‬ ‫‪11‬‬
‫‪3  6‬‬
‫‪1‬‬
‫‪1‬‬ ‫‪110  1‬‬
‫‪2‬‬
‫‪8‬‬
‫‪2‬‬ ‫‪1‬‬ ‫‪7‬‬ ‫‪12  27‬‬
‫‪.‬‬ ‫‪،‬‬ ‫‪1‬‬ ‫‪،‬‬ ‫‪6‬‬ ‫‪‬‬ ‫‪،‬‬
‫‪1‬‬
‫‪1‬‬
‫‪2‬‬
‫‪1‬‬ ‫‪2‬‬ ‫‪300‬‬ ‫‪108‬‬
‫‪1‬‬
‫‪1‬‬ ‫‪1 2‬‬
‫‪3‬‬
‫اﻟﺘﻤﺮﯾﻦ اﻟﺜﺎﻧﻲ‪:‬‬
‫‪(1‬ﺣﻠّﻞ ﻛﻼ ﻣﻦ اﻟﻌﺪدﯾﻦ ‪ 1386‬و‪ 999‬إﻟﻰ ﺟﺪاء ﻋﻮاﻣﻞ أوﻟﯿﺔ‪.‬‬
‫‪(2‬أﺣﺴﺐ اﻟﻘﺎﺳﻢ اﻟﻤﺸﺘﺮك اﻷﻛﺒﺮ ﻟﻠﻌﺪدﯾﻦ ‪ 1386‬و‪.999‬‬
‫‪(3‬ﻧﻀﻊ‪a  1,387387387... :‬‬
‫أ‪ -‬ﻣﺎھﻲ ﻃﺒﯿﻌﺔ اﻟﻌﺪد ‪ a‬؟‬
‫‪1386‬‬
‫‪a‬‬ ‫ﺑـ ‪ -‬ﺑﯿّﻦ أن ‪:‬‬
‫‪999‬‬
‫ﺟـ ‪ -‬أﻛﺘﺐ اﻟﻌﺪد ‪ a‬ﻋﻠﻰ ﺷﻜﻞ ﻛﺴﺮ ﻏﯿﺮ ﻗﺎﺑﻞ ﻟﻼﺧﺘﺰال‪.‬‬

‫اﻟﺘﻤﺮﯾﻦ اﻟﺜﺎﻟﺚ‪:‬‬
‫‪ a‬ﻋﺪد ﺣﻘﯿﻘﻲ ﻣﻮﺟﺐ ﺗﻤﺎﻣﺎ ‪.‬‬

‫‪(1‬ﺑﯿّﻦ أن ﻣﻘﻠﻮب اﻟﻌﺪد ‪ a  1  a‬ھﻮ اﻟﻌﺪد ‪a  1  a‬‬


‫‪1‬‬ ‫‪1‬‬ ‫‪1‬‬ ‫‪1‬‬
‫‪S ‬‬ ‫‪‬‬ ‫‪‬‬ ‫‪ ..... ‬‬ ‫‪ (2‬إﺳﺘﻨﺘﺞ ﻗﯿﻤﺔ اﻟﻤﺠﻤﻮع ‪:‬‬
‫‪1 2‬‬ ‫‪2 3‬‬ ‫‪3 4‬‬ ‫‪99  100‬‬

‫اﻟﺘﻤﺮﯾﻦ اﻟﺮاﺑﻊ‪:‬‬
‫‪1‬‬
‫‪x‬‬ ‫‪ x‬ﻋﺪد ﺣﻘﯿﻘﻲ ﻣﻮﺟﺐ ﻏﯿﺮ ﻣﻌﺪوم ﺑﺤﯿﺚ ‪ 1 :‬‬
‫‪x‬‬
‫‪1‬‬
‫‪(1‬ﺑﯿّﻨﺄن ‪. x   5 :‬‬
‫‪x‬‬
‫‪1 5‬‬
‫‪. x ‬‬ ‫‪(2‬إﺳﺘﻨﺘﺞ أن ‪:‬‬
‫‪2‬‬

‫‪#‬أﻧﺖ ﻻ ﺗﻔﺸﻞ أﺑﺪا إﻻّ ﺣﯿﻦ ﺗﺘﻮﻗﻒ ﻋﻦ اﻟﻤﺤﺎوﻟﺔ ‪..‬‬


‫ﺣﻞ_ﻣﻘﺘﺮح‬
:‫اﻟﺘﻤﺮﯾﻦ اﻷول‬
: ‫ ﺗﺒﺴﯿﻂ اﻷﻋﺪاد‬
2  4 2   2  2
   
2
 
3  3   11 
4 2 2
4 2 2 2
  38   34   32  9 ، 11  11   1
3  6 3   2  3  

5  2 6   5  2 6    
2
52 6  52 6  52  2 6  25  24  1  1

110  1  110  1  110    10 2 10   2 10


2 2
8 8 8
2
1  1108 1 8 8
8

108 108 108 108


7 12  27 7 2 33 3 7 5 3 7 1
6   6   6   6   6 4  4  2
2 300 2 10 3 2 10 3 2 2

1
1
1
1
 1
1
 1
1 1  2 
 11 2  2

2
1
2

1 1  2  1 2 
1 2  1 2   
1 2
1  2   1  2 
1 3 3 3
1
2 2 3 4 12 6
2
   2    
1 1 3 7 2 7 14 7
1 1 1
1 4 4 4
1
3 3
: ‫ ﺗﺼﻨﯿﻒ اﻷﻋﺪاد إﻟﻰ أﺻﻐﺮ ﻣﺠﻤﻮﻋﺔ ﺗﻨﺘﻤﻲ إﻟﯿﮭﺎ‬

 
2  4
 
4
38   ، 5 2 6  5 2 6  ،  ، 11  
3  6
1
1 1  108   1
2
7 12  27 2  1
6   ، ، 8
D ،1  
2 300 1 10 1
1 2
1
1 1 2
3

:‫اﻟﺘﻤﺮﯾﻦ اﻟﺜﺎﻧﻲ‬
. 1386  2  32  7 11 ، 999  33  37 : ‫(اﻟﺘﺤﻠﯿﻞ‬1
. PGCD  999;1386   32  9 (2
a  1,387387387..... (3
. ‫ ﻧﺎﻃﻖ‬: a ‫ ﻃﺒﯿﻌﺔ اﻟﻌﺪد‬-‫أ‬
1386
a : ‫ ﺗﺒﯿﺎن أن‬- ‫ﺑـ‬
999
a  1  0,387387387..... :‫ ھﻲ‬a  1,387387387..... ‫▪ اﻟﻜﺘﺎﺑﺔ اﻟﻜﺴﺮﯾﺔ ﻟﻠﻌﺪد‬
a  1  x :‫ وﻣﻨﮫ‬x  0,387387387..... :‫ﻧﻀﻊ‬
100  x  100  0,387387387..... :‫ ﻓﺈن‬x  0,387387387..... :‫إﻧﻄﻼﻗﺎ ﻣﻦ‬
387 1386 387
a  1  :‫ إذن‬x  :‫ وﻣﻨﮫ‬1000x  387  x
999 999 999
1386  9 154
a  :‫ ﻋﻠﻰ ﺷﻜﻞ ﻛﺴﺮ ﻏﯿﺮ ﻗﺎﺑﻞ ﻟﻼﺧﺘﺰال‬a ‫ ﻛﺘﺎﺑﺔ اﻟﻌﺪد‬- ‫ﺟـ‬
999  9 111
‫اﻟﺘﻤﺮﯾﻨﺎﻟﺜﺎﻟﺚ‪:‬‬
‫‪:‬‬
‫‪ a‬ﻋﺪد ﺣﻘﯿﻘﻲ ﻣﻮﺟﺐ ﺗﻤﺎﻣﺎ ‪.‬‬

‫‪1‬‬
‫‪‬‬
‫‪1‬‬ ‫‪‬‬ ‫‪a 1  a‬‬ ‫‪‬‬ ‫‪‬‬
‫‪1‬‬ ‫‪‬‬ ‫‪  1  a  1  a  ‬‬
‫‪a 1  a‬‬
‫‪a 1  a‬‬
‫‪a 1  a‬‬ ‫‪‬‬ ‫‪a 1  a‬‬ ‫‪‬‬ ‫‪a 1 ‬‬ ‫‪a   a  1   a ‬‬
‫‪2‬‬
‫‪a 1 a‬‬
‫‪2‬‬ ‫‪(1‬‬

‫‪ (2‬اﺳﺘﻨﺘﺎج ﻗﯿﻤﺔ اﻟﻤﺠﻤﻮع ‪:‬‬


‫‪1‬‬ ‫‪1‬‬ ‫‪1‬‬ ‫‪1‬‬
‫‪S ‬‬ ‫‪‬‬ ‫‪‬‬ ‫‪ ..... ‬‬
‫‪1 2‬‬ ‫‪2 3‬‬ ‫‪3 4‬‬ ‫‪99  100‬‬
‫‪‬‬ ‫‪2  1  3  2  4  3  .......  100  99‬‬
‫‪ 100  1‬‬
‫‪9‬‬
‫اﻟﺘﻤﺮﯾﻨﺎﻟﺮاﺑﻊ‪:‬‬
‫‪1‬‬ ‫‪: x‬‬
‫ﻋﺪد ﺣﻘﯿﻘﻲ ﻣﻮﺟﺐ ﻏﯿﺮ ﻣﻌﺪوم ﺑﺤﯿﺚ ‪x   1 :‬‬
‫‪x‬‬
‫‪2‬‬
‫‪1‬‬ ‫‪‬‬ ‫‪1‬‬ ‫‪1‬‬
‫‪x2‬‬ ‫و ﻣﻨﮫ ﺑﺎﻟﺘﺮﺑﯿﻊ ﻧﺠﺪ ‪  x    1 :‬أي أن ‪ 3 :‬‬ ‫‪x‬‬ ‫ﻟﺪﯾﻨﺎ ‪ 1 :‬‬
‫‪‬‬ ‫‪x ‬‬
‫‪2‬‬
‫‪x‬‬ ‫‪x‬‬
‫‪2‬‬
‫‪1‬‬ ‫‪‬‬ ‫‪1‬‬ ‫‪1‬‬
‫‪x  5‬‬ ‫إذن ‪:‬‬ ‫ﺑﺈﺿﺎﻓﺔ ‪ 2‬و ﻃﺮﺣﮭﺎ ﻧﺠﺪ ‪ x  2  2  5 :‬أي ‪ x    5 :‬‬
‫‪2‬‬

‫‪x‬‬ ‫‪‬‬ ‫‪x‬‬ ‫‪x‬‬


‫‪1 5‬‬ ‫‪1‬‬ ‫‪1‬‬
‫‪x‬‬ ‫اﻹﺳﺘﻨﺘﺎج‪ x   x   2x  1 5 :‬و ﻣﻨﮫ ‪:‬‬
‫‪2‬‬ ‫‪x‬‬ ‫‪x‬‬
; ;;;ÏËe¬ç’\;ÏËö\ÖŒ¥Å’\;ÏÁÖ\á°\;ÏÁÑÊ‚⁄°\
; ;;;Ï÷çfi|;ÏzzzzzzzzÁˆÊ’;;ÏËd2’\;ÏÁÖÁÅŸ;;;;;;;;;;;;;;;;;;;;;;;;;;;;;;;;;;;;;;;;;;;;;;;;;;;;;;;;;;;;;;;;;;;;;;;;;;;;;;;;;;;;;;;;;;;ÏzzzzzzzËfiöÊ’\;ÏzzzzËd2’\;ÎÑ\ÜÂ;;;;
; ;;2019—2018;;U;;;;ÏzzzzzzËà\ÑÅ’\;Ïfiâ’\;;;;;;;;;;;;;;;;;;;;;;;;;;;;;;;;;;;;;;;;;;;;;;;;;;;;;;;;;;;;;;;;;;;;;;;;;;;;;;;;;;;;;;;;;;;ÿˆ\;Èl˜zzzzzzzzm’\;‡]uzzzziŸ\;;;;
; ;;H;Ñ]zzzzzzzzzzçå;;HHH;;;;{]dÑ;fld;ÿ˜d;ÏÁÊ›]l;;;;;;;;;;;;;;;;;;;;;;;;;;;;;;;;;;;;;;;;;;;;;;;;;;;;;;;;;;;;;]ËpÊ’Êfi“hÂ;;‹Ê÷¡;‘2çŸ;ƒÉp;∞ˆ\;Ïfiâ’\;;;;
;
; ;‡]i¡]zzzzzzzzà;U;;ÎÅzzzzzzzzzzzzzzzzzzzzzzzzz∏\;;;;;;;;;;;;;;;;;;;;;;;;;;;;;;;;;;;;;;;;;;;;;;;;;;;;;;;;;;;;;;;;;;;;;;;;;;;;;;;;;;;;;k]Ëî]ÁÖ’\;ÎÄ]Ÿ;;∫;Ñ]zzzzei|\;;;;
; C;ÏŒ›; 07,5 D;U;ÿˆ\;;flÁÖ⁄i’\
ÁÖ⁄i’\
⁄i’\;;;;;;

s
; U;;;◊Ë÷¬i’\;;√Ÿ;Ït2Œ∏\;;;k]d]pˆ\;Ød;flŸ;;;;ÏuËuë’\;;;;Ïd]pˆ\;2|\
Ïd]pˆ\;2|\
pˆ\;2|\;;;
n’]m’\;;;g\Ê°\ È›]m’\;;g\Ê°\ ÿˆ\;g\Ê°\ ; ;;ÎÑ]e¬’\
ÎÑ]e¬’\
¬’\;;;;;;;;;;
;;;;;

m
; ;;€ê^ ; ;œö]› ; ÍÖç¡ 4 4 1
; Ê·;;; 1  u 1  ;;;ÄŬ’\’\
5 5

xa
; ;4 ; 16 ; 2 810  410 2
;
84  411
1,2 u104 0,12 u103 12 u105 È·;; 4 u103 u 3 u102 U;;ÄŬ÷’;ÏË⁄÷¬’\;Ïd]i“’ 3

/e
;;ÄŬ÷’;ÏË⁄÷¬’\;Ïd]i“
¬÷’;ÏË⁄÷¬’\;Ï \
S @3; 2> S @2;3> S > 2; 3@
; È·;; x 
5 1
d ;Ïup\2∏\;ÿÊ÷t;Ï¡Ê
;Ïup\2∏\;ÿÊ÷t;Ï¡Ê⁄§
Ïup\2∏\;ÿÊ÷
4
2 2

m
x 1  8 9 7 7 9 ; :¬Á;;; x  @1;8> 5
x  ; x 2  2
2 2 co ; ; ( ;ÏŒ›
Ï ; 05,5 ;; ) ;;È›]m’\;flÁÖ⁄i’\;;;
;;U;;;;nËt;;;; B Â;; A ;;;Øˌˌ¢\;;;;flÁÄŬ’\;1i¬› II
; B 2 7 ;;Â;;;; A 3 3 ;;U;;;;nËt
1
; A B ;;;‡\;;Ød C1
n
3 32 7
;;; B Â;; A ;flÁÄŬ’\;Ød;;Ï›Ñ]ŒŸ;rifiià\ C2
;; 3 3  2 7 ;ÄŬ’\;;;;ªâdÂ;Öç›\;C1
2
III
at

; x
;;;;; 5  12 21 ;U;;;nËt;; x ;ÄŬ÷’;;ÏâeŸ;Ïd]i—;;;rifiià\ ¾
55
; ; 1,7 d 3 d 1,8 ;;;;;;;;Â;;;;;; 2,6 d 7 d 2,7 ;U;;;;‡\;;j⁄÷¡;;;\Ç\C2
; ; x ;ÄŬ÷’;\Öët;;ª¡^;;;HH;^;;;;;;;;;
2018 3 2
§3 · 3 3 3 3
¨  2x ¸ G §¨  22xx ·¸ G §¨  2 x ·¸ ;G; §¨  2 x ·¸ U;;Ä\Å¡ˆ\;‡Ñ]ŒŸ;rifiià\Â;; 0 d  2 x d 1 ;U;;‡\;;Ød;;;HH;g;;;;;;;
ed

;
©5 ¹ ©5 ¹ ©5 ¹ ©5 ¹ 5
; ; ( ;;;º]Œ›;; 07 ;; ) ;U;;;n’]m’\;;;;flÁÖ⁄i’\;;;;
;ÅŸ]¬iŸ;€÷¬Ÿ;; f ;;Ï’\Å’; ( C f ) ;;◊d]Œ∏\;;;È›]Ëe’\;◊Ëm⁄i’\;;;;;
; ( o;i;
i; j ) ;;ä›]qiŸÂ;;;;;;;;
; f ;;Ï’\Å’\;ÀÁÖ¬h;Ï¡Ê⁄§;;;Ø¡ C1
f ;Ï’\Å’]d;; 5; 2;1 ;;;;flŸ;;◊—;ÎÑÊê;Ø¡ C2
f ;Ï’\Å’]d;;; 0 ;ÄŬ’\;;œd\Êà;Ø¡ C3
;;]‚ ÁÖ¬h;Ï¡Ê⁄§;;;Ì÷¡; f ;;Ï’\Å÷’;;ÏÁÅ¢\;€ËŒ’\;;Ø¡ C4
; f ;Ï’\Å’\;;3∆h;;;;‰]û\;Ø¡ C5
; f ;Ï’\Å’\;;;k\3∆h;ÿÂÅp;◊“å C6
; f ( 1,5 ) ;Â;; f ( 2 ) ;;flÁÄŬ’\;;‡Ñ]Õ C7
f ( x ) t 0 ;;Ïup\2∏\;ÿÊ÷t;;;rifiià\;;;€l; f ( x ) ÎÑ]å\;;;Ø¡ C8

1;flŸ1;Ïzzzzzzu ê
‫‪)0‬أ ‪ ---‬اجياد حصرا لـــ ‪ x‬حًح ‪x  2 7  3 3 :‬‬ ‫الــتــاريخ ‪1328/21/30 :‬‬ ‫ثانىيُ بالل بن رباح ‪/‬ششـــــار ‪/‬‬
‫احلل الننىرجٌ إلختبار الفصل االول مادَ الرياضًات‬
‫‪1,7  3  1,8‬‬ ‫‪، 2 , 6  7  2 ,7‬‬
‫االستــــــــــــــار ‪ :‬مخًشُ اجلنعٌ‬
‫لذينا ‪ 2,6  7  2,7‬ومنه ‪)2(... 5,2  2 7  5,4‬‬
‫لذينا ‪ 1,7  3  1,8‬ومنه ‪5,1  3 3  5,4‬‬ ‫التنرين االول ‪ 7 ,5 ( :‬نكطُ )‬
‫وبالتالٌ ‪)1(................... 5,4  3 3  5,1 :‬‬ ‫العبارَ اجلىاب التعلـــــــــــــــًل‬
‫جبنع (‪ )2‬و (‪ )1‬طرف لطرف جنذ‬ ‫‪4‬‬ ‫‪4‬‬ ‫‪16‬‬ ‫‪2‬‬ ‫‪2‬‬
‫‪1  1  1‬‬
‫‪5‬‬ ‫‪5‬‬ ‫‪25‬‬
‫‪ 0,2  2 7  3 3  0,3‬ومنه ‪0,2  x  0,3‬‬
‫‪25  16 3‬‬ ‫‪3‬‬
‫‪3‬‬ ‫‪‬‬ ‫‪  1 0‬‬
‫ب ‪ --‬بًان ان ‪0   2 x  1 :‬‬ ‫‪25‬‬ ‫‪5 5 2‬‬
‫‪5‬‬
‫لذينا ‪ 0,2  x  0,3 :‬ومنه ‪0,6  2 x  0,4‬‬ ‫‪810  410‬‬ ‫‪230  220‬‬ ‫‪1‬‬ ‫‪1‬‬
‫‪‬‬
‫‪3‬‬ ‫‪84  411‬‬ ‫‪212  222‬‬
‫بإضافُ جلنًع االطراف جنذ ‪:‬‬
‫‪5‬‬ ‫‪230  220‬‬ ‫‪220  210  1‬‬
‫‪ 12‬‬ ‫‪ 12‬‬
‫‪2 1  210 ‬‬
‫‪3‬‬ ‫‪3‬‬ ‫‪3‬‬
‫‪  0,6   2 x   0,4‬ينتج ‪:‬‬ ‫‪2  222‬‬
‫‪5‬‬ ‫‪5‬‬ ‫‪5‬‬
‫‪33 3‬‬ ‫‪3 2‬‬
‫‪3‬‬
‫اٍ ‪0   2 x  1‬‬
‫‪5‬‬ ‫‪5‬‬
‫‪  2x ‬‬
‫‪5‬‬ ‫‪5‬‬
‫‪ 28 ‬‬ ‫‪2 ‬‬ ‫‪4 2‬‬
‫‪ 24  16‬‬

‫استنتاج املكارنُ ‪:‬‬ ‫‪4 103  3 102  12 105‬‬ ‫‪0‬‬ ‫‪0‬‬


‫‪2‬‬ ‫‪3‬‬ ‫‪2018‬‬ ‫‪1,2 104‬‬
‫‪3‬‬ ‫‪ 3‬‬ ‫‪ 3‬‬ ‫‪ 3‬‬ ‫‪‬‬
‫‪  2x     2x     2x     2x ‬‬ ‫‪5 1 5 1‬‬ ‫‪2‬‬ ‫‪4‬‬
‫‪5‬‬ ‫‪ 5‬‬ ‫‪ 5‬‬ ‫‪ 5‬‬ ‫‪‬‬ ‫معناه ‪x    ;  ‬‬
‫التنرين الجالح ( ‪ 7‬نكاط )‬ ‫‪2 2 2 2‬‬
‫‪)2‬جمنىعُ تعريف ‪D f   3; 7 :‬‬ ‫اٍ ‪ x   2; 3‬ومنه ‪S   2; 3‬‬
‫‪8 1 7‬‬ ‫‪8 1 9‬‬ ‫‪1‬‬ ‫‪5‬‬
‫‪f (5)  3 ،‬‬ ‫‪f ( 2 )  0 ، f ( 1 )  3 )1‬‬ ‫‪r‬‬ ‫‪ c‬و ‪‬‬ ‫‪‬‬
‫‪2‬‬ ‫‪2‬‬ ‫‪2‬‬ ‫‪2‬‬
‫‪ f ( x )  0 )0‬معناه ) ‪( x  7 ) ، ( x  2 ) ، ( x  3‬‬ ‫‪9 7‬‬
‫ومنه ‪x  ‬‬
‫‪)4‬قًنُ حذيُ عظنِ ‪ 3‬من اجل ‪x  5‬‬ ‫‪2 2‬‬
‫قًنُ حذيُ صغرّ ‪ 3‬من اجل ‪x  1‬‬ ‫التنرين الجانٌ ‪ 05,5 ( :‬نكطُ)‬
‫‪)5‬الذالُ ‪ f‬متناقصُ متاما علِ ‪ 3; 1 5;7‬‬ ‫‪B2 7 ،‬‬ ‫‪A3 3‬‬ ‫‪(I‬‬
‫الذالُ ‪ f‬متزايذَ متاما علِ ‪ 1;5‬‬ ‫‪A B ‬‬
‫‪1‬‬
‫‪)2‬بًان ان‬
‫‪)6‬جذول التغريات‬ ‫‪3 32 7‬‬
‫لذينا ‪:‬‬
‫‪x‬‬ ‫‪3‬‬ ‫‪1‬‬ ‫‪5‬‬ ‫‪7‬‬
‫‪A B  3 3 2 7 ‬‬
‫‪3‬‬ ‫‪‬‬
‫‪32 7 3 32 7‬‬ ‫‪‬‬
‫‪3‬‬ ‫‪‬‬
‫‪0‬‬ ‫‪3‬‬
‫‪32 7‬‬
‫)‪f(x‬‬ ‫‪3‬‬ ‫‪0‬‬ ‫‪27  28‬‬ ‫‪1‬‬
‫‪‬‬ ‫‪‬‬
‫‪)7‬لذينا ‪ 2  1,5‬و ‪ f‬متناقصُ متاما علِ‬ ‫‪3 32 7 3 32 7‬‬
‫‪)1‬املكارنُ ‪ A  B  0 :‬ينتج ‪A  B‬‬
‫‪  3; 1‬ومنه ‪f ( 2 )  f ( 1,5 ) :‬‬
‫‪3‬‬ ‫‪‬‬
‫‪2‬‬
‫‪)8‬اشارَ ) ‪f ( x‬‬ ‫‪)2 ( II‬نشر وتبشًط العذد ‪3  2 7 :‬‬
‫‪3‬‬ ‫‪3‬‬ ‫‪‬‬
‫‪2‬‬
‫‪x‬‬ ‫‪2‬‬ ‫‪7‬‬ ‫‪32 7‬‬ ‫‪ 27  28  12 21  55  12 21‬‬
‫)‪f(x‬‬ ‫‪0 --------‬‬ ‫‪0‬‬ ‫‪+++‬‬ ‫‪0‬‬
‫‪ f ( x )  0‬معناه ‪x   2; 7‬‬ ‫حلىل املرتاجخُ ‪:‬‬ ‫‪ ---‬استنتاج كتابُ مبشطُ ‪x  55  12 21 :‬‬

‫‪S   2; 7 ‬‬ ‫‪3‬‬ ‫‪‬‬


‫‪2‬‬
‫ومنه‬ ‫‪x  55  12 21 ‬‬ ‫‪32 7‬‬ ‫لذينا‬
‫‪ 3 3  2 7  2 7 3 3‬‬
‫الن ‪3 3  2 7  0‬‬
‫امجلِورًة اجلزائرًة ادلميلراطَة امشؼبَة‬
‫وزارة امرتبَة اموطيَة‬
‫امس ية ادلراس َة ‪2020/2019‬‬ ‫مدٍرًة امرتبَة موالًة مترناست‪ -‬اثهوًة امش َخ أمود‬
‫امـمــدة ‪:‬ساغــــــتان‬ ‫املس توى ‪:‬الاوىل جذع مشرتك ػووم وتكٌووجِا‬
‫ادخبار املثال ااول يف مادة امرايضَات‬
‫اهمترٍن ااول( ‪ 05‬هلاط ) ‪:‬‬
‫أذرت االإجابة امصحَحة مع امتربٍر‪:‬‬
‫اجلواب (‪)3‬‬ ‫اجلواب (‪)2‬‬ ‫اجلواب (‪)1‬‬ ‫امسؤال‬
‫‪2  52‬‬ ‫‪23  5‬‬ ‫‪2 5‬‬ ‫)‪ُ PGCD (120;250‬و‪:‬‬
‫‪2  10 4‬‬ ‫‪0,18  105‬‬ ‫‪18  105‬‬ ‫رثبة ملدار امؼدد ‪:‬‬
‫‪4  10   4,5  10  10‬‬
‫‪2 3‬‬ ‫‪8‬‬ ‫‪7‬‬

‫‪S   2 ; 3 ‬‬ ‫‪S   1 ; 5 ‬‬ ‫‪S  0 ; 2 ‬‬ ‫مجموػة حوول املؼادةل ‪x  2  3‬‬
‫يه‪:‬‬
‫‪k   2 ;3‬‬ ‫‪‬‬ ‫‪k  2; 3 ‬‬ ‫‪k   3;  2 ‬‬ ‫‪x‬‬ ‫جمموعة األعداد احلقيقية‬ ‫‪k‬‬

‫هي‬ ‫‪x‬‬
‫‪5‬‬
‫حتقق ‪‬‬
‫‪1‬‬
‫اليت‬
‫‪2‬‬ ‫‪2‬‬

‫‪‬‬ ‫‪  ; 0    0; ‬‬ ‫‪  ;  2     2; ‬‬ ‫مجموػة ثؼرًف ادلاةل ‪ h‬حِث‪:‬‬
‫‪3‬‬
‫‪2‬‬
‫‪ hx   x‬يه‪:‬‬
‫‪x2‬‬

‫اهمترٍن املاين(‪ 04‬هلاط ) ‪:‬‬


‫‪.‬‬
‫‪d  50  8‬‬ ‫‪‬‬ ‫‪2 1‬‬ ‫حِث ‪‬‬ ‫‪ - I‬هؼترب امؼدد احللِلي‬
‫‪d‬‬

‫‪ .1‬بني أن ‪d  3 2  4 .‬‬

‫‪- .2‬كارن بني امؼددٍن ‪ 3 2‬و ‪4‬‬


‫‪ .3‬اس تًخج اإشارة ‪. d‬‬
‫‪ - II‬مس تطَل طوهل ‪ l‬حمصور بني ‪ 4‬و ‪ 5‬وغرضَ ‪ k‬حمصور بني ‪ 2‬و ‪.3‬‬
‫اوجد حرصا ملك من حمَط ومساحة املس تطَل‪.‬‬ ‫‪-‬‬
‫اهمترٍن املامث( ‪ 04‬هلاط ) ‪:‬‬
‫‪g x   3x 2  6 x‬‬ ‫حِث ‪:‬‬ ‫‪‬‬ ‫ادلاةل امؼددًة املؼرفة ػىل‬ ‫‪g‬‬

‫‪ g   1 ‬و ‪. g  2 ‬‬ ‫أحسب ‪ g 2‬و ‪ g  3‬و‬ ‫‪-1‬‬


‫‪ 2‬‬

‫‪1‬‬
‫احسب سوابق امؼدد ‪.0‬‬ ‫‪-2‬‬
‫أدرس شفؼَة ادلاةل ‪. g‬‬ ‫‪-3‬‬

‫اهمترٍن امرابع( ‪ 07‬هلاط ) ‪:‬‬


‫‪ C ‬اهمتلَل امبَاين دلاةل ‪ f‬يف مس توي مًسوب اإىل مؼمل مخؼامد ومخجاوس ‪.‬‬ ‫‪f‬‬

‫بلراءة بَاهَة اجب ػىل ما ًيل‪:‬‬


‫‪ - 1‬ػني مجموػة ثؼرًف ادلاةل ‪. f‬‬
‫‪ - 2‬ػني صور ااػداد ‪ 7 ; 2 ; 0 ;  3 :‬ابدلاةل ‪. f‬‬
‫‪ - 3‬ػني سوابق امؼدد ‪ 4‬ابدلاةل ‪. f‬‬
‫‪ - 4‬ادرس اجتاٍ ثغري ادلاةل ‪ f‬مث شلك جدول ثغرياهتا‪.‬‬
‫‪ - 5‬شلك جدول اإشارة ادلاةل ‪f‬‬

‫‪ - 6‬أمكل ما ًيل‪:‬‬

‫‪. x2  ..........‬‬ ‫و‬ ‫‪x1  .........‬‬ ‫ثلبل كمية حدًة غظمى يه ‪ ... :‬غيد‬ ‫‪f‬‬ ‫ادلاةل‬
‫‪. x3‬‬ ‫ثلبل كمية حدًة صغرى يه‪ ... :‬غيد ‪ .........‬‬ ‫‪f‬‬ ‫‪ -‬ادلاةل‬

‫ابمتوفِق نوجمَع ‪ -‬أساثذة املادة‬

‫‪2‬‬
‫امتصحَح املفصل الدخبار املثال ااول يف مادة امرايضَات‬
‫اهمترٍن ااول( ‪ 05‬هلاط ) ‪:‬‬
‫أذرت االإجابة امصحَحة مع امتربٍر‪:‬‬
‫‪ ‬حنول امؼددٍن ‪ 250‬و ‪ 120‬جند ‪ 250  2  5‬و ‪ 120  2  3  5‬و مٌَ املامس املشرتك ااكرب ُو جداء امؼوامل املشرتكة يف‬
‫‪3‬‬ ‫‪3‬‬

‫حتوَل امؼددٍن و بأصغر أس أي أن ‪ PGCD (120;250)  2  5‬اإذن االإجابة امصحَحة االإجابة (‪.)1‬‬
‫‪ ‬دلبيا ‪ 4  10   4,5  10  10  4  4,5  10  10  10‬أي أن ‪ 4  10   4,5  10  10  18  10‬ىكخهبا‬
‫‪2 3‬‬ ‫‪8‬‬ ‫‪7‬‬ ‫‪5‬‬ ‫‪2 3‬‬ ‫‪8‬‬ ‫‪7‬‬ ‫‪6‬‬ ‫‪8‬‬ ‫‪7‬‬

‫ػىل امشلك امؼومي أي أن ‪ 4  10   4,5  10  10  1,8  10‬و رثبة ملدارٍ يه ‪ 2  10‬اإذن االإجابة امصحَحة يه‬
‫‪4‬‬ ‫‪2 3‬‬ ‫‪8‬‬ ‫‪7‬‬ ‫‪4‬‬

‫االإجابة (‪. )2‬‬


‫‪S   1 ; 5 ‬‬ ‫‪ x  2  3 ‬ابس تخدام املس تلمي امؼددي جند أن‬
‫االإجابة امصحَحة (‪)2‬‬
‫و مٌَ ‪ x   2; 3‬االإجابة امصحَحة يه رمق (‪)2‬‬ ‫‪‬‬
‫‪5 1‬‬ ‫‪5 1‬‬ ‫‪5‬‬ ‫‪1‬‬
‫‪2 x3‬‬ ‫أي أن‬ ‫‪ x ‬‬ ‫ٍاكئف‬ ‫‪x‬‬ ‫‪‬‬
‫‪2 2‬‬ ‫‪2 2‬‬ ‫‪2‬‬ ‫‪2‬‬
‫و مٌَ يه ‪   ;  2     2; ‬مجموػة امتؼرًف‬ ‫‪x  2‬‬ ‫أي أن‬ ‫أن ‪x  2  0‬‬
‫دلًيا ‪ h‬مؼرفة ٍاكئف‬
‫و مٌَ االإجابة امصحَحة يه (‪)1‬‬
‫اهمترٍن املاين(‪ 04‬هلاط ) ‪:‬‬
‫هؼترب امؼدد احللِلي ‪ d‬حِث ‪d  50  8  2  1‬‬ ‫‪-I‬‬
‫‪d  2  25  4  2  4‬‬ ‫و مٌَ‬ ‫‪d  50  16 ‬‬ ‫‪ .1‬اإثبات أن ‪ d  50  8  2  1 : d  3 2  4 .‬أي أن ‪8‬‬

‫اإذن ‪ d  5 2  4  2 2‬أي أن ‪ d  3 2  4‬و ُو املطووب ‪.‬‬


‫‪.3‬‬ ‫‪ .2‬امللارهة بني امؼددٍن ‪ 3 2‬و ‪ 4‬دلًيا ‪ 3 2   18‬و ‪ 4  16‬و مٌَ‬
‫‪2 4‬‬ ‫‪2‬‬ ‫‪2‬‬

‫‪ .3‬اس تًخاج اإشارة ‪ : d‬مبا أن ‪ 3 2  4‬فاإن ‪ d  3 2  4‬ػدد موجب متام ًا ‪.‬‬


‫‪ - II‬مس تطَل طوهل ‪ l‬حمصور بني ‪ 4‬و ‪ 5‬وغرضَ ‪ k‬حمصور بني ‪ 2‬و ‪3‬‬
‫حرص حمَط املس تطَل دلًيا ‪ 4  l  5‬و ‪ 2  k  3‬ابمجلع جند ‪ 2  4  k  l  3  5‬أي أن ‪ 6  k  l  8‬ابمرضب يف‬
‫‪ 2‬جند ‪ 12  2 k  l   16‬و مٌَ حمَط املس تطَل حمصور بني ‪ 12‬و ‪16‬‬
‫حرص مساحة املس تطَل دلًيا ‪ 4  l  5‬و ‪ 2  k  3‬ابمرضب جند ‪ 8  k  l  15‬أي مساحة املس تطَل حمصور بني ‪8‬‬
‫و ‪.. 15‬‬
‫اهمترٍن املامث( ‪ 04‬هلاط ) ‪:‬‬
‫‪g x   3x 2  6 x‬‬ ‫حِث ‪:‬‬ ‫‪‬‬ ‫ادلاةل امؼددًة املؼرفة ػىل‬ ‫‪g‬‬

‫و‬ ‫‪g  3  3 3  6 3  45‬‬ ‫‪ g 2  3  2‬و‬ ‫‪ 6 2  0‬‬ ‫حساب‬
‫‪2‬‬ ‫‪2‬‬
‫‪-4‬‬

‫‪. g 2   3 2  6 2  6  6 2‬‬
‫‪2‬‬
‫‪2‬‬ ‫‪ 1  1‬‬ ‫‪ 1  3 6 3 12 15‬‬
‫و‬ ‫‪g     3    6      ‬‬ ‫‪‬‬
‫‪ 2  2‬‬ ‫‪ 2 4 2 4 4‬‬ ‫‪4‬‬

‫‪3‬‬
‫أو‬ ‫‪x0‬‬ ‫ٍاكئف‬ ‫أ ن ‪3 x x  2   0‬‬ ‫‪ 3x‬اإي‬ ‫‪2‬‬
‫‪ 6x  0‬‬‫‪ - 5‬حساب سوابق امؼدد ‪ 0‬أي حوول املؼادةل ‪ٍ g x  0‬اكئف‬
‫‪ً x  2  0‬ؼين أن ‪ x  2‬امسوابق املمكٌة نوؼدد ‪ 0‬يه ‪ 0‬و ‪. 2‬‬
‫‪ - 6‬دراسة شفؼَة ادلاةل ‪ : g‬مخياظرة ابمًس بة اىل ‪ 0‬بؼدُا حنسب ‪ g x  3 x  6 x  3x  6x‬و‬
‫‪2‬‬ ‫‪2‬‬

‫‪  g x   3x  6 x‬و ‪ g x   3x  6 x‬و مٌَ ‪ g  x  g x ; g  x   g x‬اإذن ‪ g‬مُس مِا شفؼَة ‪.‬‬
‫‪2‬‬ ‫‪2‬‬

‫اهمترٍن امرابع( ‪ 07‬هلاط ) ‪:‬‬


‫‪ .1‬مجموػة ثؼرًف ادلاةل ‪ f‬يه ‪.  3; 7 ‬‬
‫ابدلاةل ‪ f  3  4 : f‬و ‪ f 0  3‬و ‪ f 2  0‬و ‪f 7  2‬‬ ‫غَني صور ااػداد ‪7 ; 2 ; 0 ;  3 :‬‬ ‫ت‬
‫‪ .2‬ثؼَني سوابق امؼدد ‪ 4‬ابدلاةل ‪ f‬سوابق ‪ 4‬يه ‪ 4‬و ‪ 3‬‬

‫‪ .3‬دراسة اجتاٍ ثغري ادلاةل ‪ : f‬يه داةل مخياكصة ػىل اجملامني ‪  3; 0 ‬و ‪ 4 ; 7 ‬و مزتاًدة ػىل اجملال ‪0; 4 ‬‬
‫جدول ثغريات ُو ٌش بَ امليحىن امبَاين مِا‬

‫‪:‬‬ ‫‪f‬‬ ‫‪ .4‬جدول اإشارة ادلاةل‬

‫‪ .5‬اإمتام املامتني ‪:‬‬


‫‪. x2  4‬‬ ‫و‬ ‫‪x1  3‬‬ ‫ثلبل كمية حدًة غظمى يه ‪ 4 :‬غيد‬ ‫‪f‬‬ ‫ادلاةل‬
‫‪. x3  0‬‬ ‫ثلبل كمية حدًة صغرى يه‪  3 :‬غيد‬ ‫‪f‬‬ ‫‪ -‬ادلاةل‬

‫‪4‬‬
7102 ‫ديسمرب‬ )TCST( ) ‫ ا ألوىل اثنوي(جذع مشرتك علوم‬:‫املس توى‬
11‫سا‬3 :‫املدة‬ ‫اختبار الفصل ا ألول يف مادة الرايضيات‬

2 10 3 19,74 10 4 1


13  225 2
15 2  33  23
27  36  52 3
125
  ; 1 2 x  1  1 4

B A x I
B  9 x 2  12 x  2 A  3x  2
A2  B 2 1
B A 2
x  0 ; 1 x II
Ax   x 2  4 x 2  2 x  1 1
x2  4 IR 2
Ax   4 3

h S
8cm  h  9cm 36,8cm 2  S  36,9cm 2
1000cm 3 1
I h 2
2/1 ‫الصفحة‬

Web site : www.ets-salim.com /Fax023.94.83.37 - Tel : 0560.94.88.02/05.60.91.22.41/05.60.94.88.05 :


J x 3  5 3
I J I J 4

o, i , j 
 
C  x f I

f D 1
f D 2
f 4 0 3
f 4
f 2,7  f 2,5 5
D f 6
f 7
f ( x)  2 f ( x)  0 8
g ( x)  f ( x ) 0 , 3 x  3 , 3 g II
 3 , 3 g C 
g g 1
C  g 2

2/2 ‫الصفحة‬

Web site : www.ets-salim.com /Fax023.94.83.37 - Tel : 0560.94.88.02/05.60.91.22.41/05.60.94.88.05 :


:


2x10⁻³ , 079.1 01 :


01 01 2


2²x15²x3⁻³
27  36  52
12⁵


x 2 ≥1

A²-B² =2 0 I
2 1 2 II
A >B B A : A² > B²

-0≥ -1≥ 1 1≥ ≥ 0 III


𝐴(𝑥) = √𝑥² + 4│𝑥 − 1│
= √𝑥² + 4(1 − 𝑥)
= √(𝑥 − 2)²

x 2 ≤1 2

x є -2, 6 ≥1

Web site : www.ets-salim.com /Fax023.94.83.37 - Tel : 0560.94.88.02/05.60.91.22.41/05.60.94.88.05 :


є -2; 6 П 1 ; 0 𝐴(𝑥)≥1 1

S 1;0 є 1; 0

V=Sxh 0

8 ; h ; 9 ; 36.8 ; S ; 36.9

1 29171 ≥ S x h ≥ 11270

00..71 ≥ 4V ≥ 012271

1000 cm³ 1V

𝐼 2;9 2

є ;-2 2 𝑥 ∈ [3 − +𝑟, 3𝑟] │ 1│≥1 1

𝐼 -2 ; 9 𝐼⋂𝐽 2 1

𝐷 = [−4,3] 0
𝑓(1) = 2 𝑓(0) = 0 2
𝑓(3) = 0 𝑓(2) = 4
1 1 1 1 -
2 1 -
[−2,2] 𝑓 -
[2,3] [−4, −2] 𝑓 -
-
𝑥 -4 −2 2 3
0 1
𝑓(𝑥)
2 1
2 1
2 2

Web site : www.ets-salim.com /Fax023.94.83.37 - Tel : 0560.94.88.02/05.60.91.22.41/05.60.94.88.05 :


‫اﳌﺴﺘﻮى‪ :‬ا ٔوﱃ ﻧﻮي ) ﺬع ﻣﺸﱰك ﻠﻮم و ﻜ ﻮﻟﻮﺟ ﺎ( د&ﺴﻤﱪ ‪2018‬‬
‫اﳌﺪة‪03 :‬ﺳﺎ‪00‬‬ ‫اﻣ‪12‬ﺎن اﻟﻔﺼﻞ ا ٔول ﰲ ﻣﺎدة اﻟﺮ;ﺿﻴﺎت‬

‫اﻝﺘﻤرﻴن اﻷول)‪05‬ن(‪:‬‬
‫اﺨﺘر اﻹﺠﺎﺒﺔ اﻝﺼﺤﻴﺤﺔ ﻤﻊ اﻝﺘﻌﻠﻴل‬
‫‪3− x‬‬
‫ﻫﻲ ‪D f = IR D f = IR − {−2, 2} :‬‬ ‫= ) ‪f (x‬‬ ‫اﻝﻤﻌرﻓﺔ ﺒـ‪:‬‬ ‫‪f‬‬ ‫‪ (1‬ﻤﺠﻤوﻋﺔ ﺘﻌرﻴف اﻝداﻝﺔ‬
‫‪x +2‬‬

‫}‪D f = IR − {−2‬‬

‫‪ (2‬اﻝﻔرق ﺒﻴن اﻻﻋداد اﻝﻨﺎطﻘﺔ و اﻻﻋداد اﻝﺤﻘﻴﻘﻴﺔ ﻫﻲ‪:‬‬


‫ج( اﻻﻋداد اﻝﻌﺸرﻴﺔ‬ ‫ب( اﻻﻋداد اﻝﺼﻤﺎء‬ ‫ا( اﻻﻋداد اﻝطﺒﻴﻌﻴﺔ‬
‫) ‪f ( x ) = f ( −x‬‬ ‫) ‪f ( − x ) = −f ( x‬‬ ‫‪ f‬داﻝﺔ زوﺠﻴﺔ ﻴﻌﻨﻲ ‪f ( x ) = −f ( x ) :‬‬ ‫‪(3‬‬
‫‪،‬‬ ‫‪،‬‬
‫‪ x ∈ [ 2, 4] (4‬ﻤﻌﻨﺎﻩ ‪x ≺ x 2 ≺ x 3 ، x ≤ x 2 ≤ x 3 ، x ≤ x 2 ≺ x 3 :‬‬

‫‪Φ‬‬
‫} {‪،{ } ،‬‬
‫‪4 3‬‬ ‫‪ (5‬ﺘﻘﺎطﻊ اﻝﻤﺠﺎﻝﻴن ‪ [ 2, 4[ :‬و ]‪ ]4, 6‬ﻫو ‪:‬‬

‫اﻝﺘﻤرﻴن اﻝﺜﺎﻨﻲ)‪05‬ن(‪:‬‬
‫‪ x‬ﻋدد ﺤﻘﻴﻘﻲ ﻨﻀﻊ ‪A( x) = x − 1 − 2 :‬‬

‫‪ (1‬ﻋﻴن اﻝﻤﺠﺎل ‪ J‬ﻝﻘﻴم ‪ x‬ﺤﻴث ‪A( x) ≺ 0 :‬‬

‫‪ (2‬ﻋﺒر ﻋن اﻝﻤﺠﺎل ‪ I‬اﻝﻤﻌرف ﺒـ‪ I = [ 2, 4] :‬ﺒﺎﻝﺤﺼر و اﻝﻘﻴﻤﺔ اﻝﻤطﻠﻘﺔ و اﻝﻤﺴﺎﻓﺔ‬


‫‪ (3‬ﻨﻔرض أن ‪J = ]−1,3] :‬‬

‫‪I ∪J‬‬ ‫‪ I ∩J‬و‬ ‫ﻋﻴن‬


‫‪ (4‬ﺒﺎﺴﺘﻌﻤﺎل ﻤﻔﻬوم اﻝﻤﺴﺎﻓﺔ ﺤل ﻓﻲ ‪ R‬اﻝﻤﻌﺎدﻝﺔ ‪A( x) + 2 = x + 4 :‬‬

‫‪2/1‬‬ ‫ا‬
‫ﺣﻲ ﻗﻌﻠﻮل –ﺑﺮج اﻟﺒﺤﺮي‪ -‬اﻟﺠﺰاﺋﺮ‬
‫℡‪- Tel : 0560.94.88.02/05.60.91.22.41/05.60.94.88.05 :‬اﻟﻔﺎﻛﺲ ‪Web site : www.ets-salim.com /Fax023.94.83.37 :‬‬
‫اﻝﺘﻤرﻴن اﻝﺜﺎﻝث)‪10‬ن(‪:‬‬
‫ﻤﻌرﻓﺔ ﺒﺘﻤﺜﻴﻠﻬﺎ اﻝﺒﻴﺎﻨﻲ ) ‪(C‬‬ ‫ب إﻝﻰ ﻤﻌﻠم ) ‪ f . (o, i , j‬داﻝﺔ ﻋددﻴﺔ ﻝﻠﻤﺘﻐﻴر ﻘﻴﻘﻲ‬
‫اﻝﺤﻘﻴﻘﻲ ‪x‬‬ ‫اﻝﺠزء اﻻول‪:‬اﻝﻤﺴﺘوي ﻤﻨﺴوب‬
‫ﺒﺎﻻﻋﺘﻤﺎد ﻋﻠﻰ اﻝﺸﻜل ‪:‬‬
‫ﻋﻴن ‪ D‬ﻤﺠﻤوﻋﺔ ﺘﻌرﻴف اﻝداﻝﺔ ‪. f‬‬ ‫‪(1‬‬
‫اﻝطﺒﻴﻌﻴﺔ اﻝﺘﻲ ﺘﻨﺘﻤﻲ إﻝﻰ ‪ D‬ﺒﺎﻝداﻝﺔ ‪. f‬‬
‫اد اﻝطﺒﻴﻌ‬
‫‪ (2‬ﻋﻴن ﺼور اﻷﻋداد‬
‫‪1‬‬
‫ﺒﺎﻝداﻝﺔ ‪. f‬‬ ‫ﻋداد ‪ 1‬و ‪ 5‬و‬
‫ﻋﻴن ﺴواﺒق اﻷ د‬ ‫‪(3‬‬
‫‪2‬‬
‫اﻝداﻝﺔ ‪ f‬ﺜم ﺸﻜل ﺠدول ﺘﻐﻴراﺘﻬﺎ‪.‬‬
‫‪ (4‬ﻋﻴن اﺘﺠﺎﻩ ﺘﻐﻴر ﻝداﻝﺔ‬
‫‪ (5‬ﻋﻴن اﻝﻘﻴم اﻝﺤدﻴﺔ ﻝﻠداﻝﺔ ‪ f‬ﻋﻠﻰ ‪. D‬‬
‫زوﺠﻴﺔ ؟ ‪ ،‬ﻓردﻴﺔ ؟ ‪ ،‬ﻻ زوﺠﻴﺔ و ﻻ ﻓردﻴﺔ ؟ﻋﻠل‪.‬‬
‫‪ (6‬ﻫل اﻝداﻝﺔ ‪ f‬ﺠﻴﺔ‬
‫اﻝﻤﺘراﺠﺤﺔ اﻝﺘﺎﻝﻴﺘﻴن ‪. f (x ) ≥ 1 ، f ( x) = 0 :‬‬
‫‪ (7‬ﺤل اﻝﻤﻌﺎدﻝﺔ و ﻤﺘ‬

‫اﻝﺠزء اﻝﺜﺎﻨﻲ‪:‬‬
‫اﻝﻤﺠﺎل ]‪ [ −2 , 2‬ﺒﺤﻴث ﻤن أﺠل ﻜل ‪ x‬ﻤن ]‪. g ( x) = f ( x) : [0 , 2‬‬
‫‪ g‬داﻝﺔ ﻋددﻴﺔ ﻤﻌرﻓﺔ ﻋﻠﻰ اﻝﻤ‬
‫ﻝﻠداﻝﺔ ‪ g‬ﻋﻠﻰ ]‪. [ −2 , 2‬‬ ‫‪ /1‬إذا ﻋﻠﻤت أن ‪ g‬داﻝﺔ ﻓردﻴﺔ‪ ،‬اﺸرح ﻜﻴف ﻴﻤﻜن رﺴم ) ‪ (C g‬اﻝﻤﻨﺤﻨﻰ اﻝﻤﻤﺜل‬
‫ﻤﺜل ﻝﻠداﻝ‬

‫‪ /2‬أرﺴم ) ‪. (C g‬‬

‫ﺒﺎﻝﺘوﻓﻴق‬

‫‪2/2‬‬ ‫ا‬
‫ﺣﻲ ﻗﻌﻠﻮل –ﺑﺮج اﻟﺒﺤﺮي‪ -‬اﻟﺠﺰاﺋﺮ‬
‫‪Web site : www.ets-salim.com /Fax023.94.83.37‬‬
‫‪/Fax023.‬‬ ‫‪- Tel : 0560.94.88.02/05.60.91.22.41/05.‬اﻟﻔﺎﻛﺲ ‪:‬‬
‫℡‪91.22.41/05.60.94.88.05 :‬‬
‫اﻝﺘﺼﺤﻴﺢ اﻝﻨﻤوذﺠﻲ ‪TCST‬‬
‫اﻝﺘﻤرﻴن اﻻول‪5/5:‬‬
‫اﺨﺘر اﻻﺠﺎﺒﺔ اﻝﺼﺤﻴﺤﺔ ﻤﻊ اﻝﺘﻌﻠﻴل‬
‫‪3− x‬‬
‫‪1‬ن‬ ‫= ) ‪ f ( x‬ﻫﻲ ‪D f = IR :‬‬ ‫‪ (4‬ﻤﺠﻤوﻋﺔ ﺘﻌرﻴف اﻝداﻝﺔ‬
‫‪x +2‬‬

‫‪1‬ن‬ ‫‪ (5‬اﻝﻔرق ﺒﻴن اﻻﻋداد اﻝﻨﺎطﻘﺔ و اﻻﻋداد اﻝﺤﻘﻴﻘﻴﺔ ﻫﻲ‪:‬ب( اﻻﻋداد اﻝﺼﻤﺎء‬
‫‪1‬ن‬ ‫‪ f‬داﻝﺔ زوﺠﻴﺔ ﻴﻌﻨﻲ ‪f ( x ) = f ( −x ) :‬‬ ‫‪(6‬‬
‫‪1‬ن‬ ‫‪x ≺x2≺x3‬‬ ‫‪ x ∈ [ 2, 4] (4‬ﻤﻌﻨﺎﻩ ‪:‬‬
‫‪1‬ن‬ ‫‪Φ‬‬ ‫‪ (5‬ﺘﻘﺎطﻊ اﻝﻤﺠﺎﻝﻴن ‪ [ 2, 4[ :‬و ]‪ ]4, 6‬ﻫو ‪:‬‬

‫اﻝﺘﻤرﻴن اﻝﺜﺎﻨﻲ‪:5/5‬‬
‫‪ x‬ﻋدد ﺤﻘﻴﻘﻲ ﻨﻀﻊ ‪A = x − 1 − 2 :‬‬

‫‪ (1‬ﻋﻴن اﻝﻤﺠﺎل ‪ J‬ﻝﻘﻴم ‪ x‬ﻫو ‪J = ]−1,3] :‬‬

‫‪ (2‬ﻋﺒر ﻋن اﻝﻤﺠﺎل ‪ I‬اﻝﻤﻌرف ب ]‪ I = [ 2, 4‬ﺒﺎﻝﺤﺼر‪ 2 ≤ x ≤ 4 :‬و اﻝﻘﻴﻤﺔ اﻝﻤطﻠﻘﺔ ‪x − 3 ≤ 1‬‬

‫و اﻝﻤﺴﺎﻓﺔ ‪d ( x ,3) ≤ 1 :‬‬

‫‪ (3‬ﻨﻔرض ان ‪J = ]−1,3] :‬‬

‫]‪ I ∩ J = [ 2,3‬و ]‪I ∪ J = ]−1, 4‬‬ ‫ﻋﻴن‬


‫‪ 3‬‬
‫‪ (4‬ﺒﺎﺴﺘﻌﻤﺎل ﻤﻔﻬوم اﻝﻤﺴﺎﻓﺔ ﺤل ﻓﻲ اﻝﻤﻌﺎدﻝﺔ ‪ A + 2 = x + 4 :‬ﻫﻲ ‪S =  −  :‬‬
‫‪ 2‬‬

‫اﻟﺘﻤﺮﻳﻦ اﻟﺜﺎﻟﺚ ‪10/10:‬‬


‫‪= −1,2 -/1‬‬
‫‪1 = 1 f ( 2) = 5 ،‬‬ ‫‪0 = 1 -/2‬‬

‫‪ -‬ﺳﻮاﺑﻖ ‪ 1‬ﻫﻲ ‪0 ، 1‬‬


‫‪ -‬ﺳﻮاﺑﻖ ‪5‬ﻫﻲ ‪ -1‬و ‪2‬‬
‫‪1‬‬ ‫‪1‬‬
‫ﺳﻮاﺑﻖ ﻫﻲ‬ ‫‪-‬‬
‫‪2‬‬ ‫‪2‬‬

‫ﺣﻲ ﻗﻌﻠﻮل –ﺑﺮج اﻟﺒﺤﺮي‪ -‬اﻟﺠﺰاﺋﺮ‬


‫℡‪- Tel : 0560.94.88.02/05.60.91.22.41/05.60.94.88.05 :‬اﻟﻔﺎﻛﺲ ‪Web site : www.ets-salim.com /Fax023.94.83.37 :‬‬
‫ﻣﺘﺰاﻳﺪة ﰲ ‪, 2‬‬
‫‪1‬‬
‫‪-‬‬
‫‪2‬‬

‫‪−1,‬‬
‫‪1‬‬
‫ﻣﺘﻨﺎﻗﺼﺔ ﲤﺎﻣﺎ ﰲ‬ ‫‪-‬‬
‫‪2‬‬
‫‪ -‬ﺟﺪول اﻟﺘﻐﲑات ‪:‬‬

‫ ‪-1 2‬‬
‫‪1‬‬
‫‪2‬‬
‫‪5‬‬ ‫‪5‬‬

‫‪1‬‬
‫‪2‬‬
‫اﻟﻘﻴﻤﺔ اﳊﺪﻳﺔ اﻟﻌﻈﻤﻰ ‪ 5‬ﻋﻨﺪ ‪ 2‬و ‪-1‬‬
‫‪1‬‬ ‫‪1‬‬
‫اﻟﻘﻴﻤﺔ اﳊﺪﻳﺔ اﻟﺼﻐﺮى ﻋﻨﺪ‬
‫‪2‬‬ ‫‪2‬‬

‫ﺣﻲ ﻗﻌﻠﻮل –ﺑﺮج اﻟﺒﺤﺮي‪ -‬اﻟﺠﺰاﺋﺮ‬


‫℡‪- Tel : 0560.94.88.02/05.60.91.22.41/05.60.94.88.05 :‬اﻟﻔﺎﻛﺲ ‪Web site : www.ets-salim.com /Fax023.94.83.37 :‬‬
 "#$'&"#$%!     
 
./$"#! )))) +-+,+*$!())

  

   012 
 
 
 

           


  


s
     
$#  
    
 
 !" 
!

m
      *#      %%&'()   
%%&'()

      $# 
  !" 

x
 ./  +,-"$#                 !" 

/e
           
   43 
   
  01203405 (6# '
 
         (90):87) 

      (340;<'(& 

 (=2 @  D.0  >@ABC ?  ((=>%


2@
2 ! 
n
     'G 
 5FEB& 

 
 5FH)  (+,-"'(& 


 B
B
    ?   C'(7   ?   C'(7,@ 
   
c

 
OKP  (  > M:NIGA:NGE  (BLG   K.HIJIG>/, 
 
 5FH)


OKP
O
du

 B   (Q-'(& 

   5 3 

              70170,.C
 

   N& 
 
e

        JRGB& 

        SK 

 % G%U , '     T

      'GB&TM

 
 





 AL M NO*J> 


K

         

7"              $56          J  


s
      

 NO*  


m

  
NO  
    
    8
"        NO*
   

         $Q0    $
$
P56 J  

/e
                                     8
"   
  $

$
P56 J  
      
 ,
R


m


 
     
   F#=@S 
c
  
  
 

 >
"  

<=

;>
"   (
<
((
<=

;9::   A  '#8
T 
  
on
             < @#A  UVW# 
<?=

     
         
 !"#$%&' (()*
!"#$%&'
"#$%& A     
Y 
 (
X 

    
 
 (
<=  9Z
[6C 
          
     

 
     
 ,#
,#-  
 (
<=

;>
"  $B      A \Z
W  
## + #
+ 
 

 ,#-DEC 
 (
<=

;9:  $B
e


 
 (
<=   \
G<"]"YVRZ 
  
  
en




 
 

  4./ 01230

I6G2"
H
" 12      F =@SA  E[ 


                  
 
 

   
                                             
   
 
 
    
     

   
          
            
    


s

 &2,341 +/&0$%&  +,-(.$%& '() *"# 
     
     
!

am
 ,9:;: <3= %& '> +
-(. +8&0$%&  +,-(.$%& 
$%& 
& ''(  
(7
77  56
56&+

  
   
?.;
?.;0.@A
0.@A/&+
/&+1 
   
          BC 

/e
           %
%;$@% D+(E
%;$@% D
;$     > 

m
          
          
BC
 
      
co    
   
 
        
  
         
n.
             
 
         
 
tio

            
 
       

          
       

      E  
       
ed

 
     (34                          
  
y-

      
     (34+"#          "F8&0
      
en

  
    (34                    "F8&0
  

          I#?.;J0.@A:GH
 

 
 



ϰδϴϋϱΪϴγ -ΪϤΤϣϕϭήδϣϦΑΔϳϮϧΎΛΔϴϨρϮϟ΍ΔϴΑήΘϟ΍Γέ΍ίϭ
2019 / 2018 :Δϴγ΍έΪϟ΍ΔϨδϟ΍ ϝϭϷ΍Ϟμϔϟ΍ϥΎΤΘϣ·
ϲϤϠόϟ΍ϙήΘθϤϟ΍ωάΠϟ΍ :ΔΒόθϟ΍ ϱϮϧΎΛ ϰϟϭϷ΍ΔϨδϟ΍ :ϯϮΘδϤϟ΍
ϥΎΘϋΎγΕΎϴοΎ˴ϳή˷ ϟ΍ :ΓΩ˷ ΎϣϲϓέΎΒΘΧ΍
(ϥ 4) :1ϦϳήϤΗ
ଵ ଵ
. 0,9ˬξ3ˬ ˬ ˬ1ˬ0 :ΔϴϟΎΘϟ΍Ω΍ΪϋϷ΍ϦϴΑϦϣ˯ΎϤμϟ΍Ω΍ΪϋϷ΍ϭΔϘρΎϨϟ΍Ω΍ΪϋϷ΍ϭΔϳήθόϟ΍Ω΍ΪϋϷ΍ΩΪΣ /1
ଷ ଶ
. 1 – ξ2 = –ඥ͵ െ ξ8ϥ΃ΖΒΛ΃΃ΖΒΛ΃Λ΃ /2
/2
(ΔΑήϘϣΔϤϴϗϡΪΨΘδΗϻ) .ΔϘϠτϤϟ΍ΔϤϴϘϟ΍ΰϣέϥϭΪΑ |1 – ξ2|ΩΪόϟ΍ΐΘϛ΃
ΩΪόϟ΍ΐΘϛ΃
ΐΘϛ΃
ϛ΃ /3 /3

s
ଶ଴ଵଽ
ˮ൫1 – ξ2൯ ΩΪόϟ΍ΓέΎη·Ύϣ
όϟ΍ΓέΎη·Ύϣ
έΎη
Ύη /4
Ύη·Ύ

m
(ϥ 6)
6) :2ϦϳήϤ
ϦϳήϤΗ

xa
. 980ˬ 560ϦϣϼϛΔϴϟϭ΃Ϟϣ΍Ϯϋ˯΍ΪΟ
ϴϟϭ΃Ϟϣ΍Ϯϋ˯΍ΪΟ
ϣ΍Ϯϋ˯΍ΪΟ
ϣ΍Ϯϋ
Ϯ ΍ΪΟ ϰϟ·Ϟ
ϰϟ·ϞϠΣ /1
ϰ
.ppcm(560,980)˰ϟ΍ϭ pgcd(560,980)
60,980)
60 980)0)˰ϟ΍ϖΒγΎϤϣ
˰ϟ΍ϖΒγΎϤϣΞΘϨΘγ·
˰ϟ΍ϖΒγ
˰ϟ΍ϖ
ϟ΍ ϖ /2
ହହଷ
ହ ଶସସ

. + ΐδΣ΃ /3

/e
ଽ଼଴
ଽ ହ଺଴
ଽ଼଴
ଽ଼
.ϝ΍ΰΘΧϼϟϞΑΎϗήϴϏήδϛϰϟ·
ϼϟϞΑΎϗήϴϏήδϛϰϟ·
ΎϗήϴϏ
ΎϗήϴϏήδ
ϗήϴϏ
Ϗήδ
Ϗήδ ήδϜϟ΍ϝΰΘΧ· /4
ହ଺଴

m
(ϥ 4) :3ϦϳήϤΗ
(–ξB”$”ξB)ΊϓΎϜϳ A ”%) :ΎΤϴΤλϲϟΎΘϟ΍ΆϓΎϜΘϟ΍ϥϮϜϳΐΟϮϣ
co 2
ΆϓΎϜΘϟ΍ϥϮϜϳΐΟϮϣ
΍ϥϮϜϳΐΟϮϣ
ϥϮ ϳΐΟϮ
ϥϮϜϳ ΐΟ BϲϘϴϘΣΩΪϋϞϛϞΟ΃ϦϣϪϧ΃ήϛάΗ
((xx – 1)
1 ±”
2
±”
± ϥ΃νήϔϧϦϳήϤΘϟ΍΍άϫϞϣΎϛϲϓ
. x – 1 ΩΪόϟ΍Ϫϴϟ·ϲϤΘϨϳϝΎΠϣήϐλ΃ΩΪΣ /1
. xΩΪόϟ΍ήμΣ΃ /2
n.
ଵ ଵ
. a = x + ΚϴΣ aΩΪόϠϟ΍ήμΣΞΘϨΘγ· /3
ସ ସ
. a3ˬ a2ˬ aΩ΍ΪϋϷ΍ΐΗέ /4
tio

ϠϟϲϧΎϴΒϟ΍ Ϟ Μ (Cf)ϢϠόϣϰϟ·ΏϮδϨϤϟ΍ϱϮΘδϤϟ΍ϲϓ (ϥ 6) :4ϦϳήϤΗ


ˬ f(x) = x2 – 2x – 3˰Α5ϰϠϋΔϓήόϤϟ΍ f Δϟ΍ΪϠϟϲϧΎϴΒϟ΍Ϟ
Δϟ΍ΪϠϟϲϧΎϴΒϟ΍ϞϴΜϤΘϟ
Δϟ΍ΪϠϟϲϧΎϴΒϟ΍ϞϴΜϤΘϟ΍
Δϟ΍ΪϠ
ϟ΍ΪϠϠϟϲϧΎϴΒϟ
:ϖϓήϤϟ΍ϞϜθϟ΍ϲ
ϖϓήϤϟ΍ϞϜθϟ΍ϲϓϰτόϣ
ϖϓήϤϟ΍
ϖϓήϤϟ
ϖϓήϤϟ΍ϞϜθ
ϞϜθ
Ϝθ @±’@ϰϠϋΔϓήόϤϟ΍ gΔϟ΍ΪϠϟϲϧΎϴΒϟ΍ϞϴΜϤΘϟ΍ (Cg)ϭ
ca

ϦϣϼϛΪ˸ ΟΏΎδΤϟΎΑ
˶ -1
. f(3)ˬf(–1)ˬf(–2) (΃
ˮfΔτγ΍ϮΑ –4ϖΑ΍Ϯγϭ –3ϖΑ΍Ϯγ (Ώ
du

:ΞΘϨΘγ΍ϰτόϤϟ΍ϞϜθϟ΍Ϧϣ -2
. gΔτγ΍ϮΑ 1ΓέϮλ (΃
.gΔτγ΍ϮΑ –1ϖΑ΍Ϯγ (Ώ
-e

.(Cf)Ίθϧ΃ -3
.fΕ΍ήϴϐΗϝϭΪΟϞϜ͋ η -4
. [0,4]ϝΎΠϤϟ΍ϰϠϋ f˰ϟΔϳΪΤϟ΍ϢϴϘϟ΍ΞΘϨΘγ· -5
cy
en

.......................................................................................................................................
ϲϤϴ˰˰ϦϳΪϟ΍έϮϧ˰˰ϫΩΫΎΘγϷ΍ϦϋϖϴϓϮΘϟΎΑϦϣιϰϬΘϧ΍
.......................................................................................................................................
2019/2018 ωϡΝ 1 1Ϟμϔϟ΍ΕΎϴοΎϳήϟ΍έΎΒΘΧ·ςϴϘϨΗϭ΢ϴΤμΗ ϰδϴϋϱΪϴγ -ΪϤΤϣϕϭήδϣϦΑΔϳϮϧΎΛ
f(x) = x2 – 2x – 3˰Α5ϰϠϋΔϓήόϣf (ϥ 6) :4ϦϳήϤΗ (ϥ 4) :1ϦϳήϤΘϟ΍
.ϰτόϤϟ΍ΎϬϠϴΜϤΘΑ@±’@ϰϠϋΔϓήόϣJϭ ଵ
0,9 ˬ ˬ 1 ˬ 0 :Δϳήθόϟ΍ :Ω΍ΪϋϷ΍ϒϴϨμΗ /1

: f(3)ˬf(–1)ˬf(–2) (΃ΏΎδΣ -1 ଵ ଵ
f(–2) = 5 ϱ΃ f(–2) = (–2)2 – 2(–2) – 3  ςϘϓ ξ3 :˯ΎϤμϟ΍ 0,9 ˬ ˬ ˬ 1 ˬ 0 :ΔϘρΎϨϟ΍
ଷ ଶ
f(–1) = 0 ϱ΃ f(–1) = (–1)2 – 2(–1) – 3
f(3) = 0 ϱ΃ f(3) = (3)2 – 2(3) – 3 = 0 :1 – ξ૛ = –ඥ૜ െ ξૡϥ΃ΕΎΒΛ· ΕΎΒΛ· /2
/2
ϥ΃ΪΠϧΏΎδΤϟΎΑϭ (΃  ΎόϣϥΎΒϟΎγ ΎϤϬϧ΃΃ ΢ο΍
΢ο΍ϭ
ο

s
:f˰Α±ϖΑ΍Ϯγϭ –3ϖΑ΍Ϯγ (Ώ ଶ ଶ
(Ώ)... ൫1 – ξ2൯ = ቀ– ඥ͵ െ ξ8 ቁ

m
2 ϭ 0 ΎϤϫ ϥΎΘϘΑΎγ ΎϬϟ –4 ϥ΃ΪΠϧ f(x) = –3ΔϟΩΎόϤϟ΍ϞΤΑ
 ςϘϓ  ϲϫ ± ΔϘΑΎγ ϥ΃ΪΠϧ f(x) = –4ΔϟΩΎόϤϟ΍ϞΤΑϭ ΏϮϠτϤϟ΍΢πΘϳ
΢πΘϳ
πΘϳ (Ώ)ϭ (΃
΃ Ϧϣ
Ϧ 

xa
g(1) = –1 :J˰ΑΓέϮλ (΃ :ϞϜθϟ΍Ϧϣ -2 :ΔϘϠτϤϟ΍ΔϤϴϘϟ΍ΰϣέϥϭΪΑ
ϣέϥϭΪΑ
ϥϭΪΑΑ |1
| –ξ ૛_ΔΑ
ξ૛ ΔΑΎΘϛ
|1 – ξ2| = ξ2 – 1 ϥΈϓ
 Έϓ 1 – ξ 2 < 0ϥ΃ΎϤΑ
ξ2
૛૙૚ૢ
૛૙
  ςϘϓϲϫ±  ΔϘΑΎγ :J˰Α±ϖΑ΍Ϯγ (Ώ

/e
ˬΐϟΎγ 1 – ξ2αΎγϷ΍
αΎγϷ΍΍ :൫൫૚
૚– ξ ૛൯
ξ૛൯ ΓέΎη· /4
ଶ଴ଵଽ
ଶ଴
଴ଵ
଴ଵ
(ϞϜθϟ΍) :(Cf)˯Ύθϧ· -3  ΐϟΎγ ൫1 – ξ2൯
ξ2 ൯ ϥΫ·ϱΩήϓ 2019αϷ΍ϭ
ϥΫ·

m
(ϥ 6) :2ϦϳήϤΘϟ΍
co 980 = 2 ×5×7

5×72 2
5 = 2 ×5×7 :ϲϟϭϷ΍ϞϴϠΤΘϟ΍ /1
560 4

:p
ppcm(560,980)
ppcm(560,98
pppcm(56
p m(5
5 ϭ pgcd(560,980)ΝΎΘϨΘγ· /2
ppcm(560,980)=
ppc
ppcm(560,
ppcm(560,980
cm(
m(56
560
60 24×5×72ϭ pgcd(560,980)= 22×5×7
n.
:ϱ΃
ppcm
ppcm(560
ppcm(560,980) = 3920 ϭ pgcd(560,980) = 140
tio

૞૞૜ ૛૝૝
: + ΏΎδΣ /3
ૢૡ૙ ૞૟૙
ହହଷ ଶସସ ହହଷ×ସ ଶସସ×଻ ଶଶଵଶ ଵ଻଴଼ ଷଽଶ଴
+ = + = + =
ଽ଼଴ ହ଺଴ ଷଽଶ଴ ଷଽଶ଴ ଷଽଶ଴ ଷଽଶ଴ ଷଽଶ଴
ca

ହହଷ ଶସସ
. + = 1 :ϱ΃
ଽ଼଴ ହ଺଴
ଽ଼଴ ଻ ଽ଼଴ ଽ଼଴÷ଵସ଴ ଻ ૢૡ૙
= ସ :ϱ΃ = ହ଺଴÷ଵସ଴ = ସ : ૞૟૙ϝ΍ΰΘΧ· /4
du

ହ଺଴ ହ଺଴
:fΕ΍ήϴϐΗϝϭΪΟ
:f Ε -4
:ήϴϛάΗϭ(x – 1)2±”νήϔϧ (ϥ 4) :3ϦϳήϤΘϟ΍
(–ξ۰”$”ξ۰)ΊϓΎϜϳ A2”%)
ΪΠϧ x – 1)2±”Ϧϣ:x – 1 ˯ΎϤΘϧ·ϝΎΠϣ /1
ˬ±”x±”ϱ΃(x – 1)2”
[–2,2] Ϯϫ ΏϮϠτϤϟ΍ ϝΎΠϤϟ΍
c

ΪΠϨϓ±”x±”ΎϨϳΪϟϖΒγΎϤϣ : xήμΣ
en

:>@ϰϠϋf˰ϟΔϳΪΤϟ΍ϢϴϘϟ΍ -5  ±”x”  


x = 4 ϞΟ΃Ϧϣ  ϲϫ ϯήΒϜϟ΍ ૚ ૚
x = 1 ϞΟ΃Ϧϣ±  ϲϫ ϯήϐμϟ΍ ΪΠϧ±”x”Ϧϣ:a = ૝x + ૝ΚϴΣD˰ϟήμΣ
ଵ ଵ ଵ ଵ ଵ ଵ ଵ ଵ ଵ
–1 + ” x + ” + :ϱ΃ –1 ” x”
ସ ସ ସ ସ ସ ସ ସ ସ ସ

 ”D”  ϥΫ·
:ϥΈϓ”D”ϥ΃ΎϤΑ :a3ˬ a2ˬ aΐϴΗήΗ /4
ϲϤ˰ϦϳΪϟ΍έϮϧ˰ϴϫΩΫΎΘγϷ΍Ϧϋ ϰϬΘϧ· a3”D2”D  
‫اﻟﻴﻮم‪ :‬ا ٔرﺑﻌﺎء ‪ 5‬دﺴﻤﱪ ‪2018‬‬ ‫ﻧﻮﻳﺔ‪ :‬ﺳ
ﻴﺪي اﻋﺒﺎز‬
‫اﻟﺮﻳـﺎﺿﻴﺎت‪2‬‬ ‫‪3‬ااﻻﺧﺘﺒﺎر اﻷول ﻓﻲ ﻣﺎدة‬
‫اﳌﺪة‪ :‬ﺳﺎﻋﺘﺎن‬ ‫اﳌﺴ
ﺘﻮى‪"ٔ :‬وﱃ ﻧﻮي 'ﻠﻮم وﺗﻚ‬

‫اﻟﺘﻤﺮﻳﻦ اﻷول‪6) :‬ن(‬


‫اﺟﺟﺐ ﺑﺼﺤﻴﺢ ٔ"و ﺧﻄ‪ ٔ/‬ﻣﻊ اﻟﺘّﻌﻠﻴﻞ اﳊﺴﺎﰊ ﰲ ﳇﺘﺎ اﳊﺎﻟﺘﲔ ﻣﺎ ﻳﲇ‪:‬‬ ‫‪9‬‬
‫‪ A‬ﻫﻮ 'ﺪد ﻃﺒﻴﻌﻲ‪.‬‬ ‫ ‬ ‫ﻣﻣﺮﺮﺑ ّﻊ ااﻟﻌﺪد‬
‫‪3 2 2  3 2 2‬‬ ‫ ‬ ‫‪-1‬‬
‫) ‪.PGCD( B; C‬‬ ‫اﻟﻌﺪد‪N‬ﻦ‪ B 2 u 3 u 14 ; C 6 u 3 u 7 :‬ﻓﺎٕن‪2 u 3 u 7 :‬‬
‫‪2‬‬ ‫‪4‬‬ ‫‪2‬‬ ‫‪3‬‬
‫ٕاذإذا ﰷن اﻟﻌﺪ‬
‫‪2‬‬ ‫‪4‬‬ ‫‪2‬‬
‫‪-2‬‬
‫ٕاذإذا ﰷن ‪ ABC‬ﻣ ﺚ‬
‫ﻣ‪U‬ﻠﺚ واﻟﻨﻘﻄﺔ ‪ I‬ﻣ‪W‬ﺘﺼﻒ @ ‪ > AB‬ﻓﺎٕن‪. AI  IB 0 :‬‬ ‫‪-3‬‬
‫)‪ f ( x‬ﻓﺎٕن‪.D >1; f> :‬‬
‫‪f‬‬ ‫ﻣﻌﺮﻓﺔ ﻟ ﺒﺒﺎر‬
‫[ﻟﻌﺒﺎرة‪x  1 :‬‬ ‫إذا ﰷﻧﺖ ‪ f‬دا‪ّ Z‬ﺮ‬ ‫‪-4‬‬
‫‪ 3, 445 u10‬ﱔ‪.9 u 10 :‬‬
‫‪4‬‬
‫رﺗﺒﺔ ﻣﻘﺪارر اﻟﻌﺪد ‪0 u 28,1‬‬
‫‪5‬‬
‫‪-5‬‬
‫‪.‬‬
‫‪133‬‬
‫ﻛﺘﺎﺑﺔ _ﴪﻳ ّﺔ ﻣﻦ ااﻟﺸّﺸ‬
‫ﻟﺸﲁ‪:‬‬ ‫ﻳﻘ‪a‬ﻞ ﻛﺘﺘﺎﺑﺔ‬
‫‪ -6‬اﻟﻌﺪد ‪1,3434...‬ﻳﻘ‪aa‬ﻞ‬
‫‪99‬‬

‫اﻟﺘﻤﺮﻳﻦ اﻟﺜﺎﻧﻲ‪4) :‬ن(‬

‫‪.E‬‬ ‫‪co‬‬
‫‪5 3، D‬‬
‫‪2 3‬‬
‫‪3‬‬
‫ﺣ‪d‬ﺚ ‪:‬‬ ‫‪E ;D‬‬ ‫ﻧﻌﺘﱪ اﻟﻌﺪد‪N‬ﻦ اﳊﻘ‪d‬ﻘ‪d‬ﲔ‬
‫‪ -1‬اﻛﺘﺐ اﻟﻌﺪد ‪' D‬ﲆ ﺷﲁ _ﴪ ﻣﻘﺎﻣﻪ 'ﺪد ‪h‬ﻃﻖ‪.‬‬
‫ﲁ ﻣﻦ اﻟﻌﺪد‪N‬ﻦ اﳊﻘ‪d d‬‬
‫‪d‬ﻘ‪d‬ﲔ ‪ D‬و ‪.E‬‬
‫اﳊﻘ‪d‬ﻘ‪d‬ﲔ‬ ‫‪ -2‬إذا 'ﻠﻤﺖ ٔ" ّن ‪ 1,73 3 1,74 :‬ﻓ‪ٔ/‬و‪l‬ﺪ ﴫ‬
‫ﺣﴫ ‪m‬ﲁ‬
‫‪ -‬اﺳ
ﺘ‪qr‬ﺞ ﺣﴫا ‪o‬ﻠﻌﺪد ‪.D u E‬‬

‫اﻟﺘﻤﺮﻳﻦ اﻟﺜﺎﻟﺚ‪5) :‬ن(‬


‫‪ (I‬اﻧﻘﻞ ﰒ ٔ"ﳈﻞ اﳉﺪول اﻟﺘﺎﱄ‪:‬‬
‫‪uc‬‬

‫ا‪y‬ﺎل‬ ‫اﳊﴫ‬ ‫اﳌﺴﺎﻓﺔ‬ ‫اﻟﻘﳰﺔ اﳌﻄﻠﻘﺔ‬


‫‪x2 d 2‬‬
‫‪ed‬‬

‫)‪d ( x; 1‬‬ ‫‪4‬‬


‫@‪x  > 3;1‬‬

‫ﰻ ﻣﻦ اﳌﻌﺎد‪ Z‬واﳌﱰاﲧﺔ اﻟﺘﺎﻟﻴﺔ ‪:‬‬ ‫‪z -1 (II‬ﻞ ﰲ‬


‫‪y‬‬

‫‪R‬‬

‫‪1 x  1‬‬ ‫‪x3‬‬ ‫)‪،x 2‬‬ ‫‪ 4 t1‬‬

‫‪ -2‬اﻛﺘﺐ دون رﻣﺰ اﻟﻘﳰﺔ اﳌﻄﻠﻘﺔ اﻟﻌﺒﺎرﺗﲔ ‪ A‬و ‪ B‬ﺣ‪d‬ﺚ ‪:‬‬


‫‪A‬‬ ‫‪x 2 4 x 3‬‬ ‫‪،B‬‬ ‫‪1 2 2  2 3  2‬‬

‫اﻟﺼﻔﺔ ‪2/1‬‬
‫اﻟﺘﻤﺮﻳﻦ اﻟﺮاﺑﻊ‪5) :‬ن(‬
‫)‪f ( x‬‬ ‫‪x2  2 x 1‬‬ ‫دا‪ Z‬ﻣﻌﺮﻓﺔ 'ﲆ ‪[ R‬ﻟﻌﺒﺎرة‪:‬‬ ‫‪f‬‬ ‫ﻟﺘﻜﻦ‬
‫‪f ( x) ( x  1) 2  2‬‬ ‫ﲢﻘّﻖ ٔ"ﻧ ّﻪ ﻣﻦ ٔ"‪l‬ﻞ ﰻ ‪ x‬ﻣﻦ ‪ R‬ﻓﺎٕن‪:‬‬ ‫‪-1‬‬
‫ﺗﻐﲑ ا‰ّ ا‪' f Z‬ﲆ ﰻ ﻣﻦ ا‪y‬ﺎﻟﲔ @‪ @f; 1‬و >‪. >1; f‬‬ ‫ادرس اﲡﺎﻩ ّ‬ ‫‪-2‬‬
‫ﺗﻐﲑات ا‰ّ ا‪ ; f Z‬ﰒ ّ'ﲔ اﻟﻘﳰﺔ اﳊﺪﻳ ّﺔ ّﻟ‹ا‪ f Z‬ﻣ‪ّ a‬ﻴِﻨﺎ ﻃﺒﻴﻌﳤﺎ‪.‬‬
‫ﺷﲁ ‪l‬ﺪول ّ‬ ‫ّ‬ ‫‪-3‬‬
‫ﻟ‹ا‪ f Z‬ﰒ ‪z‬ﻞ ﰲ ‪ R‬اﳌﻌﺎد‪. f ( x) 1 ; f ( x) 7 :Z‬‬ ‫ﺸﲁ اﳌﻨﺎﺳﺐ ّ‬ ‫ﻟﺸﲁ‬‫اŽﱰ ااﻟﺸّﺸ‬ ‫‪-4‬‬
‫‪x‬‬ ‫‪-3 -2 -1 0 1 2‬‬ ‫اﳉﺪول اﻟﺘﺎﱄ‪:‬‬‫اﻧﻘﻞ ﰒ ٔ"ﳈﳈﻞ اﳉ‬ ‫‪-5‬‬
‫ ‪f x‬‬

‫ ‬
‫‪. O; i , j‬‬ ‫ﻟﺴﺎﺑﻖ ٔ"ﺸﺊ ‪ C‬ﻣ‪W‬ﺤﲎ ا‰ّ ا‪ f Z‬ﰲ اﳌﻌﲅ اﳌﺘﻌﺎﻣﺪ ‬
‫‪f‬‬ ‫‪[ -6‬ﻻﻻﺳ

ﺘﻌﺘﻌﺎﻧﺔ [ﳉﺪول ا ﺴ‬
‫اﻟﺴﺎﺑﻖ‬

‫‪‬ﺑﺎﻟﺘﻮﻓﻴﻖ و اﻟﻨﺠﺎح ﻟﻠﺠﻤﻴﻊ‪‬‬ ‫اﻧﺘﻬﻰ‬

‫اﻟﺼﻔﺔ ‪2/2‬‬
‫اﻷول‪2‬‬ ‫‪3‬اﻟﺤﻞ اﻟﻨﻤﻮذﺟﻲ ﻟﻼﺧﺘﺒﺎر‬
‫‪3, 26 u 0,15 D u E‬‬ ‫‪3, 27 u 0,16‬‬
‫‪0,1‬‬ ‫‪D uE‬‬ ‫اﺳ
ﺘ‪qr‬ﺎج ﺣﴫ ﻟـ‪:‬‬ ‫اﻟﺴ
ﻨﺔ ا‰راﺳ
ﻴﺔ‪2019/2018:‬‬ ‫ٔ"وﱃ ﻧﻮي 'ﻠﻤﻲ‬
‫‪0, 49 D u E‬‬ ‫‪0,52‬‬
‫‬
‫
  ٔ‪:‬‬
‫‬
‫
   ‪:‬‬
‫ ‬ ‫ ‬
‫‪2‬‬
‫‪A2‬‬ ‫‪3 2 2  3 2 2‬‬ ‫‪(1‬‬
‫‪ (I‬إﻛﲈل اﳉﺪول‬
‫ ‪ 3  2 2  3  2 2  2 3  2 2 3  2 2‬‬
‫‪2‬‬

‫اﻟﻘﳰﺔ اﳌﻄﻠﻘﺔ‬
‫‪2‬‬

‫ا‪y‬ﺎل‬ ‫اﳊﴫ‬ ‫اﳌﺴﺎﻓﺔ‬


‫@‪x  > 0; 4‬‬ ‫‪0d xd4‬‬ ‫‪d ( x; 2) d 2‬‬ ‫‪x2 d 2‬‬ ‫‪3  2 2  3  2 2  2 32  2 2‬‬ ‫ ‬
‫‪2‬‬
‫‪62 4‬‬

‫‪s‬‬
‫>‪x  @5;3‬‬ ‫‪5‬‬ ‫‪x 3‬‬ ‫)‪d ( x; 1‬‬ ‫‪4‬‬ ‫‪x 1‬‬ ‫‪4‬‬ ‫وﻣ‪W‬ﻪ‪W‬ﻪ ﳌاﳌﺮﺑﻊ اﻟﻌﺪد ﻫﻮ ''ﺪد ﻃﻃﺒﻴﻌﻲ‪.‬‬
‫@‪x  > 3;1‬‬ ‫‪3 d x d 1‬‬ ‫‪d ( x; 1) d 2‬‬ ‫‪x 1 d 2‬‬
‫‪C‬‬ ‫‪62 u 34 u 7 2‬‬ ‫‪22 u 36 u 7 2‬‬ ‫‪2‬‬
‫‪(2‬‬
‫‪z -1‬ﻞ ﰲ ‪ R‬اﳌﻌﺎد‪ Z‬واﳌﱰاﲧﺔ‪:‬‬ ‫‪(II‬‬ ‫‪B‬‬ ‫‪2 u 3 u 14‬‬
‫‪2‬‬ ‫‪4‬‬
‫‪2 u3 u7‬‬
‫‪3‬‬ ‫‪4‬‬

‫ﻳﺐ‬ ‫ ‪PGCD C ; B‬‬ ‫‪22 u 34 u 7‬‬


‫ب‬ ‫وﻣ‪W‬ﻪ اﳉﻮاب ﺧﻄ‪.ٔ/‬‬
‫ب‬ ‫ﺘﺼﻒ @ ‪ > AB‬ﻓﺎٕن‪:‬‬
‫ﻘﻄﺔ ﻣ‪W‬ﺘﺼﻒ‬
‫‪ ABC (3‬ﻣ‪U‬ﻠﺚ و ‪ I‬اﻟﻨﻘﻄﺔ‬
‫‪S‬‬ ‫وﻣ‪W‬ﻪ ‪z‬ﻞ اﳌﱰاﲧﺔ >‪@f;3@ ‰ >5; f‬‬ ‫‪ AI‬و ‪ BI‬ﻣ‪q‬ﻌﺎ_ﺴﺎن وﻣ‪W‬ﻪ ‪AI  BI 0‬‬
‫‪z x  1 x  3‬ﻞ ﰲ ‪ R‬اﳌﻌﺎد‪Z‬‬ ‫إذن اﳉﻮاب ﺧﻄ‪.ٔ/‬‬
‫‪x f‬‬ ‫‪3‬‬ ‫‪1‬‬ ‫‪f‬‬ ‫)‪f ( x‬‬ ‫‪ (4‬ﶍﻮ'ﺔ ﺗﻌﺮﻳﻒ ا‰ا‪ f Z‬ﺣ‪d‬ﺚ‪x  1 :‬‬
‫)‪x  1 ( x  1) ( x  3‬‬ ‫)‪( x  1‬‬ ‫‪x3‬‬ ‫‪x 1 x  3‬‬ ‫‪ x 1 t 0‬وﻣ‪W‬ﻪ ‪N x t 1‬ﲀ— >‪D >1; f‬‬
‫‪f‬‬
‫‪x  3 x 1 x  3‬‬
‫‪1 3‬‬
‫‪x 1 x  3‬‬ ‫‪1 3‬‬
‫‪ (5‬رﺗﺒﺔ ﻣﻘﺪار اﻟﻌﺪد ‪ 3, 445 u10 u 28.1‬‬
‫‪5‬‬

‫‪x‬‬ ‫‪1‬‬
‫‪tio‬‬

‫‪3, 445 u105 u 2,81u10‬‬


‫‪S‬‬ ‫اﳌﻌﺎد‪^1` Z‬‬ ‫وﻣ‪WW‬ﻪ‬
‫ﻣ‪W‬ﻪ ‪z‬ﻞ اﳌﻌﺎ‬
‫وﻣ‪W‬ﻪ‬ ‫‪3 u 3 u104‬‬

‫‪q‬ﺎﺑﺔ دون رﻣرﻣﺰ اﻟﻘﳰﺔ اﳌﻄﻠﻘﺔ‪:‬‬


‫‪ -2‬اﻟﻜ‪q‬‬
‫ﻟﻜ‪q‬ﺎﺑﺔ‬
‫اﻟﻜ‪q‬ﺎﺑﺔ‬ ‫‪9 u104‬‬
‫‪ca‬‬

‫وﻣ‪W‬ﻪ اﳉﻮاب ﲱﻴﺢ‪.‬‬


‫‪B‬‬ ‫ ‬
‫‪ 1 2 2  2 3  2‬‬‫ ‬ ‫ ‬ ‫‪ (6‬اﻟﻜ‪q‬ﺎﺑﺔ اﻟﻜﴪﻳﺔ ‪o‬ﻠﻌﺪد ‪1,3434...‬‬
‫‪1  2 2  6  2 2‬‬ ‫‪7  4 2‬‬
‫‪du‬‬

‫‪34‬‬ ‫‪99  34‬‬ ‫‪133‬‬


‫ ‪1  0,3434 1‬‬
‫‪A‬‬ ‫‪x2 4 x3‬‬ ‫‪99‬‬ ‫‪99‬‬ ‫‪99‬‬
‫‪x f‬‬ ‫‪3‬‬ ‫‪2‬‬ ‫‪f‬‬ ‫وﻣ‪W‬ﻪ اﳉﻮاب ﲱﻴﺢ‪.‬‬
‫‪-e‬‬

‫‬
‫
  
‪:‬‬
‫))))‪x  2 ( x  2)  4(( x  3‬‬ ‫)‪x  2  4( x  3‬‬
‫)‪( x  2)  4( x  3‬‬
‫‪x  3  x  2  4 x  12‬‬ ‫‪3x  14‬‬
‫‪ x  2  4 x  12‬‬
‫‪y‬‬

‫‪3x  10‬‬ ‫‪ -1‬ﻛﺘﺎﺑﺔ ‪' D‬ﲆ ﺷﲁ _ﴪ ﻣﻘﺎﻣﻪ 'ﺪد ‪h‬ﻃﻖ‬


‫‪5x  10‬‬
‫‪nc‬‬

‫‪D‬‬
‫‪2 3‬‬ ‫‪ 2  3 u‬‬ ‫‪3‬‬ ‫‪2 3 3‬‬
‫‪3‬‬ ‫‪3u 3‬‬ ‫‪3‬‬
‫‪E ,D‬‬ ‫‪ -2‬ﺣﴫ ﰻ ﻣﻦ‬
‫‪1,73‬‬ ‫‪3 1,74‬‬ ‫‪1, 73‬‬ ‫‪3 1, 74‬‬
‫‪3, 46‬‬ ‫‪2 3‬‬ ‫‪3, 48‬‬
‫‪1,74  3 1,73‬‬
‫‪0, 46‬‬ ‫‪2 3 3‬‬ ‫‪0, 48‬‬
‫‪3, 26 5  3 3, 27‬‬
‫‪2 3 3‬‬
‫‪0,15‬‬ ‫‪0,16‬‬
‫‪3, 26‬‬ ‫‪E‬‬ ‫‪3, 27‬‬ ‫وﻣ‪W‬ﻪ‬ ‫‪3‬‬
‫وﻣ‪W‬ﻪ ‪0,15 D 0,16‬‬
‫‬
‫‪f‬‬ ‫ا‪m‬ﳣﺜﻴﻞ اﻟﺒﻴﺎﱐ ﻟ‹ا‪Z‬‬ ‫
   ‪:‬‬
‫)‪f ( x‬‬ ‫‪ f‬دا‪ Z‬ﻣﻌﺮﻓﺔ 'ﲆ ‪[ R‬ﻟﻌﺒﺎرة‪x  2 x  1 :‬‬
‫‪2‬‬

‫)‪f ( x‬‬ ‫‪ -1‬اﻟﺘﺤﻘﻖ ‪x  2 x  1 x  2 x  1  2‬‬


‫‪2‬‬ ‫‪2‬‬

‫ ‪ x  1‬‬
‫‪2‬‬
‫‪2‬‬
‫‪ -2‬دراﺳﺔ اﲡﺎﻩ اﻟﺘﻐﲑ ا‰ا‪' f Z‬ﲆ ﰻ ﻣﻦ ا‪y‬ﺎﻟﲔ @‪ @f; 1‬و >‪>1; f‬‬
‫‪ -‬ﻧﻔﺮض ‪' b , a‬ﺪدان ﺣﻘ‪d‬ﻘ‪d‬ﺎن ﻣﻦ ا‪y‬ﺎل @‪ @f; 1‬ﺣ‪d‬ﺚ ‪a b‬‬
‫‪a‬‬ ‫‪b d 1‬‬
‫‪a 1 b 1 d 0‬‬
‫وﻣ‪W‬ﻪ ا‰ا‪ f Z‬ﻣ‪q‬ﻨﺎﻗﺼﺔ ﲤﺎﻣﺎ 'ﲆ @‪@f; 1‬‬ ‫ ‪ a  1 b  1‬‬
‫‪2‬‬ ‫‪2‬‬

‫‪ a  1  2 b  1  2‬‬
‫‪2‬‬ ‫‪2‬‬

‫)‪f (a‬‬ ‫)‪f (b‬‬

‫&‬ ‫‪ > 1; f‬ﺣ‪d‬ﺚ ‪a b‬‬


‫>‪f‬‬ ‫ا‪y‬ﺎل‬ ‫ﻧﻔﺮض ‪' b , a‬ﺪدانن ﺣﻘ‪dd‬ﻘ‪d‬‬
‫‪d‬ﺎن ﻣﻦ ا‪y‬ﺎ‬
‫ﺣﻘ‪d‬ﻘ‪d‬ﺎن‬
‫‪d‬ﻘ‪d‬ﺎن‬ ‫‪-‬‬
‫"‬‫!  ‪$
%  (   "
* "  ,‬‬
‫‪#‬‬
‫    ‪ '   ) +‬‬ ‫‪1d a‬‬ ‫‪b‬‬
‫‬
‫ ٔ ‪/01 23 * :- .‬‬ ‫‪0 d a 1 b 1‬‬
‫ا‪ f Z‬ﻣﱱاﻳﺪة ﲤﲤﺎﻣﺎ 'ﲆ >‪f‬‬
‫‪>1;;f‬‬ ‫وﻣ‪W‬ﻪ ا‰ا‪Z‬‬ ‫ ‪ a  1 b  1‬‬
‫‪2‬‬ ‫‪2‬‬

‫‪ a  1  2 b  1  2‬‬
‫‪2‬‬ ‫‪2‬‬

‫)‪f (a‬‬ ‫)‪f (b‬‬


‫‪f‬‬ ‫‪l -3‬ﺪول اﻟﺘﻐﲑات ﻟ‹ا‪Z‬‬
‫‪on‬‬

‫‪ f‬ﺗﻘ‪a‬ﻞ ﻗﳰﺔ ‪z‬ﺪﻳّﺔ ﺻﻐﺮى ﱔ ‪ 2‬ﳌﺎ‪.x 1‬‬ ‫ﻣﻦ ‪l‬ﺪول اﻟﺘﻐﲑات‪،‬ا‰ا‪Z‬‬


‫‪z -4‬ﻞ ﰲ ‪ R‬اﳌﻌﺎد‪Z‬‬
‫)‪f ( x‬‬ ‫‪1‬‬ ‫)‪f ( x‬‬ ‫‪7‬‬
‫‪x 2  2 x  1 1‬‬ ‫‪( x  1) 2  2‬‬ ‫‪7‬‬
‫‪x2  2 x‬‬ ‫‪ x  1  9 0‬‬
‫‪2‬‬
‫‪0‬‬
‫)‪x( x  2‬‬ ‫‪0‬‬ ‫ ‪ x  1  3 x  1  3‬‬ ‫‪0‬‬
‫ﻭﻣﻨﻪ ‪ x 0‬ﺃﻭ ‪x 2‬‬ ‫)‪( x  4)( x  2‬‬ ‫‪0‬‬
‫‪x 2‬‬ ‫‪"ٔ x‬و‬ ‫‪4‬‬ ‫وﻣ‪W‬ﻪ‬
‫‪s‬‬ ‫`‪^0; 2‬‬
‫‪s‬‬ ‫`‪^2; 4‬‬
‫‪ -5‬إﻛﲈل اﳉﺪول‬
‫‪x‬‬ ‫‪-3 -2 -1‬‬ ‫‪ 2‬يءء‪0 1‬‬
‫‪ff‬‬ ‫‪2 -1 -2 -1 2‬‬ ‫‪7‬‬
‫)‪f ( x‬‬
‫ ‪f x‬‬
‫اﻟﺠﻤﮭﻮرﯾﺔ اﻟﺠﺰاﺋﺮﯾﺔ اﻟﺪﯾﻤﻘﺮاطﯿﺔ اﻟﺸﻌﺒﯿﺔ‬
‫ﺛﺎﻧﻮﯾﺔ ﺑﻦ ﻣﺴﺮوق ﻣﺤﻤﺪ ‪ -‬ﺳﯿﺪي ﻋﯿﺴﻰ‬ ‫وزارة اﻟﺘﺮﺑﯿﺔ اﻟﻮطﻨﯿﺔ‬
‫اﻟﺴﻨﺔ اﻟﺪراﺳﯿﺔ‪2020 / 2019 :‬‬ ‫إﻣﺘﺤﺎن اﻟﻔﺼﻞ اﻷول‬
‫اﻟﺠﺬع اﻟﻤﺸﺘﺮك اﻟﻌﻠﻤﻲ‬ ‫اﻟﻤﺴﺘﻮى‪ :‬اﻟﺴﻨﺔ اﻷوﻟﻰ ﺛﺎﻧﻮي‬
‫ــــــــــــــــــــــــــــــــــــــــــــــــــــــــــــــــــــــــــــــــــــــــــــــــــــــــــــــــــــــــــــــــــــــــــــــــــــــــــــــــــــــــــــــــــــــــــــــــــــــــــــــــــــــــــــــــــــــــــــــــــــــــــــــــــــــــــــــــــــــــــــــــــــــــــــــــــــــــــــــــــــ‬
‫‪ 02‬ﺳﺎ‬ ‫اﺧﺘﺒﺎر ﻓﻲ ﻣﺎدّة‪ :‬اﻟ ّﺮﯾَﺎﺿﯿﺎت‬
‫ــــــــــــــــــــــــــــــــــــــــــــــــــــــــــــــــــــــــــــــــــــــــــــــــــــــــــــــــــــــــــــــــــــــــــــــــــــــــــــــــــــــــــــــــــــــــــــــــــــــــــــــــــــــــــــــــــــــــــــــــــــــــــــــــــــــــــــــــــــــــــــــــــــــــــــــــــــــــــــــــــــ‬

‫اﻟﺘﻤﺮﯾﻦ اﻷول‪ 6) :‬ن(‬


‫إﺧﺘﺮ اﻟﻘﻀﯿﺔ أو اﻟﻘﻀﺎﯾﺎ اﻟﺼﺤﯿﺤﺔ إن وﺟﺪت ﻣﻊ اﻟﺘﻌﻠﯿﻞ ﻓﻲ ﻛﻞ ﺣﺎﻟﺔ ﻣﻤﺎ ﯾﻠﻲ‪:‬‬
‫‪ /1‬إذا ﻛﺎن اﻟﻌﺪد اﻟﺤﻘﯿﻘﻲ ‪ x‬ﯾﺤﻘﻖ‪ |x + 4| < 6 :‬ﻓﺈن‪:‬‬
‫ج( ‪– 10 ≤ x ≤ 2‬‬ ‫ب( ‪4 ≤ x ≤ 6‬‬ ‫أ( ‪–4 < x < –6‬‬
‫‪ଶ‬‬
‫‪ /2‬إذا ﻛﺎن ‪ A = |√3 – 1| + |2 – 2√3| – 3ට ൫1 − √3൯‬ﻓﺈن‪:‬‬
‫ج( ‪A = 2√3 –2‬‬ ‫ب( ‪A = 1 – √ 3‬‬ ‫أ( ‪A = √ 3 – 1‬‬
‫‪ /3‬إذا ﻛﺎن [∞‪ I = [0,+‬و [‪ J = ]–5,5‬ﻓﺈن‪:‬‬
‫ج( [‪I∩J = ]0,5‬‬ ‫ب( ]‪I∩J = [0,5‬‬ ‫أ( [‪I∩J = [0,5‬‬
‫‪ /4‬إذا ﻛﺎن ‪ x = 0,1443‬و ‪ y = 14×103‬ﻓﺈن رﺗﺒﺔ ﻣﻘﺪار اﻟﺠﺪاء ‪ xy‬ھﻲ‪:‬‬
‫ج( ‪2×103‬‬ ‫ب( ‪14×14,43‬‬ ‫أ( ‪2000‬‬
‫‪ /5‬إذا ﻛﺎن اﻟﻌﺪد اﻟﺤﻘﯿﻘﻲ ‪ x‬أﻛﺒﺮ أو ﯾﺴﺎوي ‪ 1‬ﻓﺈن‪:‬‬
‫‪2‬‬ ‫‪2‬‬
‫ج( ‪(1 – 5x) ≥ –16‬‬ ‫ب( ‪(1 – 5x) ≥16‬‬ ‫أ( ‪(1 – 5x)2 ≤ –16‬‬

‫اﻟﺘﻤﺮﯾﻦ اﻟﺜﺎﻧﻲ‪ 6) :‬ن(‬

‫‪ /1‬أﻧﻘﻞ اﻟﺸﻜﻞ اﻟﻤﺮﻓﻖ ﺛﻢ ﻋﻠﱢﻢ اﻟﻨﻘﻄﺘﯿﻦ ‪ B ،A‬اﻟﻠﺘﯿﻦ ﻓﺎﺻﻠﺘﺎھﻤﺎ ﻋﻠﻰ اﻟﺘﻮاﻟﻲ ھﻤﺎ ‪ ، –4 ،2‬وﻋﻠﱢﻢ ‪ C‬ﻣﻨﺘﺼﻒ ]‪.[AB‬‬
‫‪ /2‬ﺣﻞ ﻓﻲ ‪ R‬اﻟﻤﻌﺎدﻟﺘﯿﻦ واﻟﻤﺘﺮاﺟﺤﺔ اﻟﺘﺎﻟﯿﺔ‪) :‬ﯾﻤﻜﻦ اﻻﻋﺘﻤﺎد ﻋﻠﻰ ﺷﻜﻞ ﺟﻮاب اﻟﺴﺆال اﻟﺴﺎﺑﻖ(‬
‫)‪|x + 4| ≤ |x – 2| ... (3‬‬ ‫)‪|x + 4| + |x – 2| = 6 ... (2‬‬ ‫)‪|x – 2| = 3 ... (1‬‬

‫اﻟﺘﻤﺮﯾﻦ اﻟﺜﺎﻟﺚ‪ 8) :‬ن(‬


‫اﻟﺪاﻟﺘﺎن ‪ f‬و ‪ g‬ﻣﻌﺮﻓﺘﺎن ﻋﻠﻰ [∞‪[–3,+‬‬
‫ﺑﺘﻤﺜﯿﻠﯿﮭﻤﺎ اﻟﺒﯿﺎﻧﯿﯿﻦ ﻓﻲ اﻟﺸﻜﻞ اﻟﻤﺮﻓﻖ‪ ،‬إﻋﺘﻤﺎدا‬
‫ﻋﻠﯿﮫ ﺟﺪ ﻛﻼ ﻣﻤﺎ ﯾﻠﻲ‪:‬‬

‫‪ /1‬اﻟﺼﻮر )‪ g(0) ، f(1) ، f(0‬و)‪.g(1‬‬


‫‪ /2‬ﺳﻮاﺑﻖ ‪ –1‬ﺑﻮاﺳﻄﺔ ‪.g‬‬
‫‪ /3‬ﺟﺪول ﺗﻐﯿﺮات ‪.g‬‬
‫‪ /4‬إﺷﺎرة )‪ f(x) – g(x‬ﻣﻦ أﺟﻞ ‪ x‬ﻣﻦ [∞‪.[–3,+‬‬
‫‪ /5‬ﺣﻠﻮل اﻟﻤﻌﺎدﻟﺔ )‪ f(x) = 0 … (1‬وﻣﺠﻤﻮﻋﺔ‬
‫ﺣﻠﻮل اﻟﻤﺘﺮاﺟﺤﺔ )‪.g(x) ≤ f(x) … (2‬‬
‫‪ /6‬اﻟﻘﯿﻢ اﻟﺤﺪﯾﺔ ﻟﻠﺪاﻟﺔ ‪ g‬ﻋﻠﻰ [∞‪.[–3,+‬‬
‫‪ /7‬ﻣﺠﺎﻻ ﺗﻜﻮن ﻓﯿﮫ ‪ g‬رﺗﯿﺒﺔ ﻋﻠﯿﮫ‪.‬‬
‫‪ /8‬اﻟﺪاﻟﺔ اﻟﺰوﺟﯿﺔ ﻣﻦ ﺑﯿﻦ ھﺘﯿﻦ اﻟﺪاﻟﺘﯿﻦ‪.‬‬

‫ــــــــــــــــــــــــــــــــــــــــــــــــــــــــــــــــــــــــــــــــــــــــــــــــــــــــــــــــــــــــــــــــــــــــــــــــــــــــــــــــــــــــــــــــــــــــــــــــــــــــــــــــــــــــــــــــــــــــــــــــــــــــــــــــــــــــــــــــــــــــــــــــــــــــــــــــــــــــــــــــــــ‬
‫ص ‪1/1‬‬
‫ــــــــــــــــــــــــــــــــــــــــــــــــــــــــــــــــــــــــــــــــــــــــــــــــــــــــــــــــــــــــــــــــــــــــــــــــــــــــــــــــــــــــــــــــــــــــــــــــــــــــــــــــــــــــــــــــــــــــــــــــــــــــــــــــــــــــــــــــــــــــــــــــــــــــــــــــــــــــــــــــــــ‬
‫أوﻟﻰ ج م ع‬ ‫ﺗﺼﺤﯿﺢ وﺗﻨﻘﯿﻂ اﺧﺘﺒﺎر اﻟﻔﺼﻞ اﻷول ﻓﻲ اﻟ ّﺮﯾَﺎﺿﯿﺎت ‪2020/2019 -‬‬ ‫ﺛﺎﻧﻮﯾﺔ ﺑﻦ ﻣﺴﺮوق ﻣﺤﻤﺪ ﺳﯿﺪي ﻋﯿﺴﻰ‬
‫اﻟﺘﻤﺮﯾﻦ اﻟﺜﺎﻟﺚ‪ 8) :‬ن( ‪ f‬و ‪ g‬ﻣﻌﺮﻓﺘﺎن ﻋﻠﻰ [∞‪[–3,+‬‬

‫اﻟﺘﻤﺮﯾﻦ اﻷول‪ 6) :‬ن(‬


‫‪ /1‬إذا ﻛﺎن اﻟﻌﺪد اﻟﺤﻘﯿﻘﻲ ‪ x‬ﯾﺤﻘﻖ‪ |x + 4| < 6 :‬ﻓﺈن‪:‬‬
‫‪ –6 < x + 4 < 6‬أي ‪ –10 < x < 2‬وﻣﻨﮫ ج( ﺻﺤﯿﺤﺔ ‪ 1‬ن‬

‫‪૛‬‬
‫‪ /2‬إذا ﻛﺎن ‪A = |√૜ – 1| + |2 – 2√૜| – 3ට ൫૚ − √૜൯‬‬
‫ﻓﺈن‪ A = √3 – 1 + 2√3 – 2 – 3|1 – √3| :‬أي‪:‬‬
‫)‪A = 3√3 – 3 – 3(√3 –1‬أي‪ A =0 :‬ﻛﻠﮭﺎ ﺧﻄﺄ ‪ 1,5‬ن‬

‫‪ /3‬إذا ﻛﺎن [∞‪ I = [0,+‬و [‪ J = ]–5,5‬ﻓﺈن‪I∩J = [0,5[ :‬‬


‫أ( ﺻﺤﯿﺤﺔ ‪ 1‬ن‬

‫‪ /4‬إذا ﻛﺎن ‪ x = 0,1443‬و ‪ y = 14×103‬ﻓﺈن‪:‬‬


‫‪ 0,5‬ن× ‪4‬‬ ‫‪ /1‬اﻟﺼﻮر )‪ g(0) ، f(1) ، f(0‬و)‪:g(1‬‬ ‫‪ xy = 0,1443×14×103 = 2,0202×103‬إذن ر ﺗﺒﺔ‬
‫‪. g(1) = 1 g(0) = 3 f(1) = 1 f(0) = 0‬‬ ‫ﻣﻘﺪار ‪ xy‬ھﻲ ‪ ،2000‬وﻣﻨﮫ أ( و ج( ﺻﺤﯿﺤﺘﺎن ‪ 1,5‬ن‬

‫‪ /2‬ﺳﻮاﺑﻖ ‪ –1‬ﺑﻮاﺳﻄﺔ ‪ – 1 :g‬ﻟﮫ ﺳﺎﺑﻘﺔ واﺣﺪة ھﻲ ‪ 0,5 – 3‬ن‬ ‫‪ /5‬إذا ﻛﺎن اﻟﻌﺪد اﻟﺤﻘﯿﻘﻲ ‪ x ≥ 1‬ﻓﺈن‪ –5x ≤ –5 :‬إذن‬
‫‪ 1 – 5x ≤ –4‬إذن ‪(1 – 5x)2 ≥16‬‬
‫‪1‬ن‬ ‫وﻣﻨﮫ ب( و ج( ﺻﺤﯿﺤﺘﺎن‬
‫‪1‬ن‬ ‫‪ /3‬ﺟﺪول ﺗﻐﯿﺮات ‪:g‬‬
‫‪x‬‬ ‫‪–3‬‬ ‫‪0‬‬ ‫‪2,2‬‬ ‫∞‪+‬‬
‫‪3‬‬
‫‪g‬‬
‫‪–1‬‬ ‫‪–0,5‬‬
‫‪ /4‬إﺷﺎرة )‪ f(x) – g(x‬ﻣﻦ أﺟﻞ ‪ x‬ﻣﻦ [∞‪ 1 :[–3,+‬ن‬
‫ﻣﻮﺟﺒﺔ ﻋﻠﻰ ﻛﻞ ﻣﻦ اﻟﻤﺠﺎﻟﯿﻦ ]‪، [1,+∞[ ، [–3;–1,2‬‬ ‫اﻟﺘﻤﺮﯾﻦ اﻟﺜﺎﻧﻲ‪ 6) :‬ن(‬
‫‪ /1‬ﻧﻘﻞ اﻟﺸﻜﻞ وﺗﻌﻠﯿﻢ ‪ B ،A‬ﻓﺎﺻﻠﺘﺎھﻤﺎ ‪ –4 ،2‬و‪ 1,5 :C‬ن و ﺳﺎﻟﺒﺔ ﻋﻠﻰ اﻟﻤﺠﺎل ]‪. [–1,2;1‬‬

‫‪ /5‬ﺣﻠﻮل اﻟﻤﻌﺎدﻟﺔ )‪:f(x) = 0 … (1‬‬


‫ﻟﮭﺎ ﺣﻞ واﺣﺪ ھﻮ ‪ 0,5 0‬ن‬
‫ﻣﺠﻤﻮﻋﺔ ﺣﻠﻮل )‪:g(x) ≤ f(x) … (2‬‬
‫ﻣﺠﻤﻮﻋﺔ ﺣﻠﻮل ) ‪ ( 2‬ھﻲ [∞‪ 1 . [–3;–1,2]U[1,+‬ن‬ ‫‪ /2‬ﺣﻞ )‪ 1,5 :|x – 2| = 3 ... (1‬ن‬
‫)‪ (1‬ﺗﻌﻨﻲ ‪ d(x,2) = 3‬إذن ﻟـ) ‪ ( 1‬ﺣﻼن ﻓﻲ ‪ R‬ھﻤﺎ ‪ 5‬و ‪- 1‬‬
‫‪1‬ن‬ ‫‪ /6‬اﻟﻘﯿﻢ اﻟﺤﺪﯾﺔ ﻟـ ‪ g‬ﻋﻠﻰ [∞‪:[–3,+‬‬
‫ﻻ ﺗﻘﺒﻞ ﻗﯿﻤﺔ ﺣﺪﯾﺔ ﻛﺒﺮى‬ ‫‪ 1,5‬ن‬ ‫ﺣﻞ )‪:|x + 4| + |x – 2| = 6 ... (2‬‬
‫ﻟﮭﺎ ﻗﯿﻤﺔ ﺣﺪﯾﺔ ﺻﻐﺮى ھﻲ ‪ –1‬ﻣﻦ أﺟﻞ ‪، x = –3‬‬ ‫)‪ (2‬ﻣﻌﻨﺎه ‪d(x,–4) + d(x,2) = 6‬‬
‫ﻟﮭﺎ ﻗﯿﻤﺔ ﺣﺪﯾﺔ ﺻﻐﺮى ﻣﺤﻠﯿﺔ ھﻲ ‪ –0,5‬ﻣﻦ أﺟﻞ ‪. x = 2,2‬‬ ‫إذن ﻣﺠﻤﻮﻋﺔ ﺣﻠﻮل ) ‪ ( 2‬ﻓﻲ ‪ R‬ھﻲ ]‪. [–4,2‬‬
‫ﻟﮭﺎ ﻗﯿﻤﺔ ﺣﺪﯾﺔ ﻛﺒﺮى ﻣﺤﻠﯿﺔ ھﻲ ‪ 3‬ﻣﻦ أﺟﻞ ‪. x = 0‬‬
‫‪ 1,5‬ن‬ ‫ﺣﻞ )‪:|x + 4| ≤ |x – 2| ... (3‬‬
‫‪ 0,5‬ن‬ ‫‪ /7‬ﻣﺠﺎل ﺗﻜﻮن ‪ g‬رﺗﯿﺒﺔ ﻋﻠﯿﮫ‪ :‬ﻣﺜﻼ ]‪. [–3,0‬‬ ‫)‪ (3‬ﻣﻌﻨﺎه )‪d(x,–4) ≤ d(x,2‬‬
‫إذن ﻣﺠﻤﻮﻋﺔ ﺣﻠﻮل ) ‪ ( 3‬ﻓﻲ ‪ R‬ھﻲ ]‪. ]–∞,–1‬‬
‫‪ /8‬ﺷﻔﻌﯿﺔ اﻟﺪاﻟﺘﯿﻦ‪ 0,5 :‬ن‬
‫ﻟﯿﺴﺘﺎ زوﺟﯿﺘﯿﻦ وﻻ ﻓﺮدﯾﺘﯿﻦ ﻷن ﻣﺠﻤﻮﻋﺔ ﺗﻌﺮﯾﻔﯿﮭﻤﺎ ﻟﯿﺴﺖ‬
‫ﻣﺘﻨﺎظﺮة ﺑﺎﻟﻨﺴﺒﺔ ﻟﻠﺼﻔﺮ ‪.‬‬
‫ﻋﻦ اﻷﺳﺘﺎذ دھﯿﻤﻲ ﻧﻮراﻟﺪﯾﻦ‬
‫ص‪1/1‬‬
‫السنة الدراسية ‪9191/9102 :‬‬ ‫ثانوية سيدي اعباز‬
‫االختبار األول في مادة الرياضيات‬
‫المدة‪ :‬ساعتان‬ ‫المستوى‪ :‬أولى علوم تك‬
‫التمرين األول‪6:‬ن‬
‫‪4 3, 2,1‬‬
‫‪b  5n1  5n a  7n1  7n‬‬ ‫‪b a‬‬
‫‪n‬‬ ‫‪b a‬‬
‫‪ppcm  a; b   7n  5n  3  23‬‬
‫‪d  4, 4747...‬‬ ‫‪d‬‬
‫‪103‬‬ ‫‪d‬‬
‫‪.d‬‬
‫‪2‬‬ ‫‪2‬‬ ‫‪2‬‬
‫‪.x   2;3‬‬ ‫‪‬‬ ‫‪‬‬ ‫‪x‬‬
‫‪5 2x 1 3‬‬
‫‪.x 1; x 2 ; x 3‬‬ ‫‪-‬‬
‫‪x‬‬
‫‪f  x ‬‬ ‫‪f‬‬
‫‪x2  1‬‬
‫‪f‬‬
‫التمرين الثاني‪7:‬ن‬
‫)‪E ( x‬‬ ‫‪x 3‬‬ ‫‪x 1‬‬ ‫‪1‬‬
‫‪E  x‬‬
‫‪E‬‬ ‫‪ 3  5  2‬‬ ‫‪3‬‬ ‫‪E  2‬‬
‫‪2‬‬
‫‪1,7‬‬ ‫‪3‬‬ ‫‪1,8‬‬
‫)‪E ( 3‬‬
‫‪E x‬‬ ‫‪1 ‬‬
‫‪E x‬‬ ‫‪x 3‬‬ ‫‪‬‬
‫‪E x‬‬ ‫‪1‬‬ ‫‪‬‬
‫الصفحة ‪1/2‬‬
‫ن‬7:‫التمرين الثالث‬
:‫الجزء األول‬
f  x   4x 2  12x  5 ℝ f
2
f  x    2x  3  4
f  x  0 f  0  ; f (1)
 3 3 
 ; 2  ;   f
 2 
.f
3
f  x  f    0 :‫فإن‬ℝ x
2
‫الجزء الثاني‬
C  f f
f
7,  2
f 0, f  1, f 2
f
f
‫انتهى بالتوفيق للجميع‬
E  x ‫ﺍﻟﺤﻞ ﺍﻟﻨﻤﻮﺫﺟﻲ ﻟﻺﺧﺘﺒﺎﺭ ﺍﻷﻭﻝ ﻓﻲ ﻣﺎﺩﺓ ﺍﻟﺮﻳﺎﺿﻴﺎﺕ‬
x  1 3 
‫اﻟﺘﻤﺮﻳﻦ اﻷول‬
x 3    b  5n 1  5n a  7 n 1  7 n
n b a
x 1   
b  5n 1  5n  5n  5  5n  5n  5  1  5n  2  3
E  x x  3 x  3 x 3 a  7 n 1  7 n  7 n  7  7 n  7 n  7  1  7 n  23
 x 1  1 x 1 1  x 1 1
ppcm  a; b   7 n  5n  3  23
3  2x  5  1
ppcm a; b  7n 5n 3 23 a  7 n  23 b  5n  2  3
E  3  5  2 3 E  2
d  4, 4747... d
1;3 3 2
4 ; 4, 475 103 d
E  3  5  2 3; E  2   2  2  5  1
1 .....d 102  d  443
1, 7  3  1,8  2 .....d 102  d  99d
2 443
d 443  99d  2  1
E ( 3) 99
1, 7  3  1,8 2 2 2
.x   2;3   x
5 2x 1 3
21, 7  2 3  21,8 1 1 1 2 2 2
   
3, 4  2 3  3, 6 5 2x 1 3 5 2x 1 3
3, 4  5  2 3  5  3, 6  5 6  2x  4 5  2x 1  3
1, 6  2 3  5  1, 4
.x   2;3 3 x 2
1, 6  E  x  1, 4
2 2 2 1 1 1 .x 1 ; x 2 ; x 3 -
   
1, 6 E  x  1, 4 1, 6 E  x  1, 4 1 1 1 1 1
 x 1    3 x 2
2 3 2 3 x 2
1, 25   1, 43
E  x .x 1  x 2  x 3 0  x 1  1
 x
f  x   f
x2  1
x2 s  2 ‫ وﻣﻧﮫ‬2 x  5  1 ‫ وﻣﻧﮫ‬E  x   1 -1
f
x  3  x 1  1  x  3
‫ ﯾﻛﺎﻓﺊ‬E x  x  3 -2
Df
x  2 x 1  1
s  0; 2   1  x 1
x  0  x  1  1 x x
f  x  2
  f  x
 x 1 x2 1
x  3  x 1 1  1 ‫ وﻣﻧﮫ‬E  x   1 ‫ﺣل اﻟﻣﻌﺎدﻟﺔ‬
f
d  x;3  d  x;1 ‫ وﻣﻧﮫ‬x  3  x 1 ‫ﺗﻛﺎﻓﺊ‬
:‫اﻟﺘﻤﺮﻳﻦ اﻟﺜﺎﻧﻲ‬
s   2;  E ( x )  x  3  x 1  1
3 3
‫ﻗﻴﻤﺔ‬ f  ‫ﻷن‬ f  x  f   ‫ﻟﺪﻳﻨﺎ ﻣﻦ ﺟﺪول اﻟﺘﻐ ات‬
2 2
:‫اﻟﺘﻤﺮﻳﻦ اﻟﺜﺎﻟﺚ‬
3
f  x  f    0 ‫ﺣﺪﻳﺔ ﺻﻐﺮى وﻣﻨﮫ‬
2 :‫اﻟﺠﺰء اﻷول‬
:‫ا ﺰء اﻟﺜﺎ ﻲ‬ f  x   4 x 2  12x  5 ℝ f
2
f D f   2; 2 f  x    2x  3  4
2
. 1 1  2   2x  3  4  4x2  9 12x  4
. 2 2 7  4x2 12x  5  f ( x)
f  0  1, f  1  2, f  2   7 f  0   5; f (1)  21
2 2
f  2 x  3  22  0  2 x  3 40 f  x  0
 2 x  1 2 x  7   0  2 x  3  4  2 x  3  4   0
7 1
x x 2x  7  0 2x 1  0
2 2
 3 3 
 ; 2   ;   f
f   2 
 3
x  2, x  2 7 x1  x2  ; 2  x2 x1
 
x  1, x  1 2 3
2 x1  3  2 x2  3  0 2 x1  2 x2  3 x1  x2 
2
2 2
 2 x1  3   2 x2  3  0
2 2
f  x1   f  x2   2 x1  3  4   2 x2  3  4
 3
 ; 2  f
 
3 
x1  x2  ;   x2 x1
2 
3
0  2 x1  3  2 x2  3 3  2 x1  2 x2  x1  x2
2
2 2
 2 x1  3   2 x2  3
2 2
f  x1   f  x2   2 x1  3  4   2 x2  3  4
3 
 ;   f
2 
‫ﺍﻧﺘﻬﻰ ﺑﺎﻟﺘﻮﻓﻴﻖ ﻟﻠﺠﻤﻴﻊ‬
3
f  x  f    0 :‫ﻓﺈن‬ℝ x
2
‫الجمهورية الجزائرية الديمقراطية الشعبية‬
‫وزارة التربية الوطنية‬
‫السنة الدراسية ‪9191/9102 :‬‬ ‫ثانوية ‪:‬وادي الماء *باتنة*‬
‫التاريخ ‪9102/12/ 01 :‬‬ ‫المستوى ‪ :‬السنة االولى جذع مشترك علوم‬
‫المدة الزمنية ‪:‬ساعتان‬

‫‪‬‬
‫اخرت الاجابة الصحيحة الوحيدة ‪ ،‬مع التعليل ‪ ،‬للك سؤال من الاس ئةل التالية‪:‬‬
‫‪ )1‬من بني الاعداد التالية العدد الطبيعي هو ‪:‬‬
‫‪1‬‬
‫‪ ‬‬ ‫‪8  2 15 ‬‬ ‫‪ a   b ‬‬
‫‪2 4‬‬ ‫‪4‬‬
‫‪3‬‬
‫‪‬‬ ‫‪B‬‬ ‫‪ a 1 ‬‬ ‫‪a 8‬‬
‫‪0‬‬

‫‪2019‬‬ ‫‪A‬‬ ‫‪ 2   7‬‬


‫‪C ‬‬
‫‪  ‬‬
‫‪0‬‬ ‫‪1‬‬ ‫‪a  b ‬‬
‫‪6‬‬ ‫‪2 2‬‬
‫‪b ‬‬ ‫‪b‬‬
‫‪2019‬‬ ‫حيث ‪ a‬و ‪ b‬عددان حقيقيان موجبان‬

‫‪ )2‬رتبة مقدار العدد ‪  75 10    4 10 ‬يه ‪:‬‬


‫‪3‬‬ ‫‪2‬‬

‫‪3  10 3‬‬ ‫‪4  10 4‬‬ ‫‪3  10 2‬‬


‫‪ )3‬القامس املشرتك الاكرب للعددين ‪ n  840‬و ‪ m  270‬هو ‪:‬‬
‫‪PGCD (n , m )  7561‬‬ ‫‪PGCD (n , m )  7560‬‬ ‫‪PGCD (n , m )  30‬‬
‫‪x 1440‬‬
‫يه‪:‬‬ ‫‪f  x   2019‬‬
‫‪x‬‬ ‫‪‬‬
‫‪ )4‬شفعية ادلاةل ‪ f‬املعرفة عىل ‪ R‬ابلشلك ‪:‬‬
‫‪‬‬

‫الزوجية وال فردية‬ ‫فردية‬ ‫زوجية‬


‫‪ N‬فان‪:‬‬ ‫‪‬‬ ‫‪ )5‬العدد احلقيقي ‪ x‬حيث ‪، x  ‬اذا اكن دلينا ‪ x   ‬‬
‫‪2‬‬

‫‪N  x  ‬‬ ‫‪N  x  ‬‬ ‫‪N  x ‬‬

‫‪ )6‬اذا اكن ‪ x  2‬فان ‪:‬‬


‫‪1‬‬ ‫‪1‬‬ ‫‪1‬‬ ‫‪1‬‬ ‫‪1‬‬ ‫‪1‬‬
‫‪‬‬ ‫‪‬‬
‫‪3x  4‬‬ ‫‪2‬‬ ‫‪3x  4 2‬‬ ‫‪3x  4 2‬‬

‫‪ x )7‬عدد حقيقي اذا اكن ‪ x  0;1‬فان ‪:‬‬


‫‪1  x ‬‬ ‫‪ 1  x ‬‬ ‫‪1  x ‬‬ ‫‪1  x ‬‬ ‫‪1  x ‬‬ ‫‪1  x ‬‬
‫‪1962‬‬ ‫‪1954‬‬ ‫‪1962‬‬ ‫‪1954‬‬ ‫‪1962‬‬ ‫‪1954‬‬

‫اقلب الصفحة‬
‫‪‬‬
‫حيث ‪:‬‬ ‫‪I‬‬ ‫اجملاالت ‪، L‬‬ ‫نعترب يف‬
‫‪I  y  R /‬‬ ‫‪d  2 y ; 5 ‬‬ ‫‪7‬‬
‫‪‬‬ ‫‪12‬‬ ‫‪‬‬
‫‪L  x  R  8 / 3 ‬‬ ‫‪ 6‬‬
‫‪‬‬ ‫‪x  8‬‬ ‫‪‬‬
‫حل يف املرتاحجة‪ ، 2 y  5 7 :‬مث اكتب ‪ I‬عىل شلك جمال ‪.‬‬ ‫‪.1‬‬
‫بني أنه من أجل لك عدد حقيقي ‪ x‬أن‪ ، 4  x  6 :‬مث اكتب ‪ L‬عىل شلك جمال‪.‬‬ ‫‪.2‬‬
‫عني ‪ I L‬و ‪. I L‬‬ ‫‪.3‬‬
‫حل يف املعادةل واملرتاحجة التالية ‪:‬‬ ‫‪.4‬‬

‫‪. 9  3x 15 ، x  8   x  2 ‬‬ ‫‪2‬‬

‫‪‬‬
‫لتكن ادلاةل ‪ f‬املعرفة جبدول تغيااها (الشلك املقابل ) ‪،‬‬ ‫‪.I‬‬

‫‪x‬‬ ‫‪2‬‬ ‫‪1‬‬ ‫‪0‬‬ ‫‪1‬‬ ‫‪5‬‬

‫‪f x‬‬ ‫‪5‬‬ ‫‪2‬‬ ‫‪4‬‬

‫‪1‬‬ ‫‪0‬‬

‫من خالل جدول التغيات أجب عىل ماييل‪:‬‬


‫‪ .1‬عني مجموعة تعريا ادلاةل ‪. D f‬‬
‫‪ .2‬عني العداد احلقيقية‪ f  0 ، f  1 ، f 5 ، f  4 :‬و ‪. f 1‬‬
‫‪ .3‬عني سابقة ‪ 5‬و ‪ 0‬العدد ابدلاةل ‪. f‬‬
‫‪ .4‬صا‪ ،‬ابس تعامل عبارات مناس بة تغيات ادلاةل ‪. f‬‬
‫‪ .5‬عني القمي احلدية لدلاةل ‪ f‬عىل اجملال ‪ D f‬معينا القمي اليت تبلغها من أجلها‪.‬‬
‫‪ .6‬أنشئ املنحين ) ‪ (C‬املمثل لدلاةل ‪ f‬يف معمل متعامد متجانس ‪. O ; i ; j ‬‬
‫‪f‬‬

‫بالتوفيق‬ ‫الصفحة ‪2/2‬‬ ‫انتهى‬


‫التنقيط‬
‫‪‬‬
‫‪A‬‬
‫‪ a   b    a ‬‬
‫‪2 4‬‬ ‫‪5‬‬
‫‪1‬‬ ‫‪4‬‬

‫‪‬‬
‫‪a 8‬‬
‫العدد الطبيعي هو ‪:‬‬
‫‪‬‬ ‫‪‬‬
‫‪a  b   b ‬‬ ‫‪2 2‬‬ ‫‪2‬‬
‫‪6‬‬ ‫‪b7‬‬

‫‪A‬‬
‫‪ a   b    a   a‬‬
‫‪2 4‬‬ ‫‪3‬‬
‫‪1‬‬ ‫‪4‬‬ ‫‪8‬‬
‫‪‬‬
‫‪a 8  b 3  a 4  a 8‬‬
‫‪‬‬ ‫‪‬‬
‫‪a  b   b  b‬‬ ‫‪2 2‬‬ ‫‪a 12  b 4  b 8  b 7‬‬
‫لن ‪:‬‬
‫‪2‬‬ ‫‪7‬‬

‫‪ 0,5‬ن‬
‫‪6‬‬

‫‪a 12  b 3‬‬


‫‪A‬‬ ‫‪1‬‬
‫‪a 12  b 3‬‬

‫رتبة مقدار العدد ‪  75 10    4 10 ‬يه ‪:‬‬


‫‪3  10 4‬‬
‫‪3‬‬ ‫‪2‬‬

‫‪ 75 10    4 10   8 10    4 10 ‬‬


‫‪3‬‬ ‫‪2‬‬ ‫‪2‬‬ ‫‪2‬‬

‫‪ 5,5‬ن‬ ‫لن ‪:‬‬


‫‪  32 10   3, 2 10 10  3 10‬‬
‫‪4‬‬ ‫‪1‬‬ ‫‪4‬‬ ‫‪3‬‬

‫‪2‬ن‬
‫‪PGCD (n , m )  30‬‬ ‫القامس املشرتك الاكرب للعددين ‪ n  840‬و ‪ m  270‬هو ‪:‬‬
‫لن ‪ n  2  3  5  7 :‬و ‪ m  2  3  5‬ومنه ‪PGCD (n , m )  30 :‬‬ ‫‪3‬‬ ‫‪3‬‬

‫‪x 1440‬‬
‫ليست‬ ‫‪f  x   2019‬‬
‫‪x‬‬ ‫‪‬‬
‫ال ميكن دراسة الشفعية لن ادلاةل ‪ f‬املعرفة عىل ‪ 0 ;   ‬ابلعبارة‬
‫‪1‬ن‬ ‫زوجية وال فردية ‪ ،‬ل ّن اجملال ‪ 0 ;   ‬غي متناظر ابلنس بة اىل ‪. 0‬‬

‫‪ N‬فان‪:‬‬ ‫‪‬‬ ‫‪x  ‬‬


‫‪2‬‬
‫‪،‬اذا اكن دلينا‬ ‫‪x ‬‬
‫العدد احلقيقي ‪ x‬حيث‪:‬‬
‫‪1‬ن‬
‫‪x   ; x  ‬‬
‫‪x  ‬‬
‫‪2‬‬
‫‪ x   ‬‬
‫‪x   ; x  ‬‬
‫‪ N  x  ‬لن ‪:‬‬
‫‪‬‬

‫اذا اكن ‪ x  2‬فان ‪:‬‬


‫‪1‬‬
‫‪‬‬
‫‪1‬‬
‫‪3x  4 2‬‬
‫دلنيا ‪ x  2‬نرضب الطريف يف ‪ -3‬املتباينة جند ‪3x  6 :‬‬
‫نضيا ‪ +4‬للطريف املتباينة جند ‪3x  4  2 :‬‬
‫‪1‬ن‬ ‫‪1‬‬ ‫‪1‬‬
‫‪‬‬ ‫ومنه ‪:‬‬
‫‪3x  4 2‬‬

‫‪1  x ‬‬
‫‪1962‬‬
‫‪1  x ‬‬
‫‪1954‬‬
‫‪ x‬عدد حقيقي اذا اكن ‪ x  0;1‬فان ‪:‬‬
‫‪1‬ن‬
‫‪. 1 x ‬‬‫‪1962‬‬
‫‪1  x ‬‬
‫‪1954‬‬
‫دلينا ‪ 0 x 1 :‬معناه ‪ 0 1 x 1 :‬ومنه ‪:‬‬
‫‪‬‬
‫‪2y  5‬‬ ‫‪7‬‬ ‫املرتاحجة‪:‬‬
‫حل يف‬
‫دلينا ‪2 y  5 7 :‬‬
‫معناه ‪7 2 y  5 7 :‬‬
‫‪7  5‬‬ ‫أي ‪2 y  5  5 7  5 :‬‬
‫اذن ‪12 2 y 2 :‬‬
‫‪1‬ن‬
‫‪. 6‬‬ ‫‪y‬‬ ‫‪1‬‬ ‫ومنه ‪:‬‬

‫‪I  6;1‬‬ ‫‪ ‬كتابة ‪ I‬عىل شلك جمال ‪:‬‬


‫‪:‬‬ ‫‪x‬‬ ‫من أجل لك عدد حقيقي‬
‫دلنيا ‪ P  -------- 4  x  6‬‬
‫‪ ‬نرضب طريف ‪  P ‬يف ‪ 1‬جند ‪ Z  -------- 6  x  4 :‬‬
‫‪ ‬نضيا العدد ‪ 8‬لطريف ‪  Z ‬املتباينة جند ‪2  x  8  4 :‬‬
‫‪1‬‬ ‫‪1‬‬ ‫‪1‬‬
‫‪‬‬ ‫‪‬‬ ‫ومنه ‪:‬‬
‫‪4 x  8 2‬‬

‫‪2‬ن‬ ‫‪3‬‬
‫‪12‬‬
‫‪6‬‬ ‫اذن ‪:‬‬
‫‪x  8‬‬
‫‪.‬‬
‫‪L   4;6 ‬‬ ‫‪ ‬كتابة ‪ L‬عىل شلك جمال ‪:‬‬
‫تعني تقاطع وااحاد لك من اجملالني ‪ I‬و ‪: L‬‬
‫‪2‬ن‬
‫‪.I‬‬ ‫‪L  6;1‬‬ ‫‪4;6‬‬ ‫‪I‬‬ ‫‪L ‬‬

‫‪x 8  x 2‬‬ ‫حل يف املعادةل ‪ x  8   x  2  :‬معناه ‪:‬‬


‫‪2‬‬

‫عىل مس تقمي مدرج‪ ،‬نسمي ‪ M‬النقطة اليت فاصلهتا ‪.x‬‬


‫‪ x  8‬يه املسافة من النقطة ‪ M‬اىل النقطة ‪ A‬ذات الفاصةل ‪8‬‬
‫‪ x  2‬يه املسافة من النقطة ‪ M‬اىل النقطة ‪ B‬ذات الفاصةل ‪2‬‬
‫‪ x  8  x  2‬تاكئف ‪ MA  MB‬و ‪M   AB ‬‬
‫‪8  2‬‬
‫هذا يعين أ ّن ‪ M‬منتصا ‪ . AB‬فاصةل ‪ M‬يه ‪. 2  3‬‬
‫اذن مجموعة حلول املرتاحجة يه ‪S  3 :‬‬
‫‪1‬ن‬
‫‪9  3x‬‬ ‫‪15‬‬ ‫‪ ‬حل يف املرتاحجة التالية ‪:‬‬
‫‪x 2‬‬ ‫‪3x 6‬‬ ‫‪9  3x 15‬‬
‫‪‬‬ ‫‪ 9  3x 15‬معناه ‪ 3x 24‬ومنه‬
‫‪‬‬
‫‪1‬ن‬ ‫‪ x 8‬‬ ‫‪‬‬ ‫‪‬‬
‫اذن مجموعة حلول املرتاحجة يه ‪S  ; 8 2;  :‬‬

‫‪‬‬
‫‪1‬ن‬ ‫مجموعة تعريا ادلاةل ‪ f‬يه اجملال ‪. 2;5‬‬
‫صورة العدد ‪ 4‬ابدلاةل ‪ f‬ال توجد‪.‬‬
‫صورة العدد ‪ 5‬ابدلاةل ‪ f‬يه العدد احلقيقي ‪. 4‬‬
‫‪1‬ن‬
‫صورة العدد ‪ 1‬ابدلاةل ‪ f‬يه العدد احلقيقي ‪.1‬‬
‫صورة العدد ‪ 0‬ابدلاةل ‪ f‬يه العدد احلقيقي ‪. 2‬‬
‫صورة العدد ‪ 1‬ابدلاةل ‪ f‬يه العدد احلقيقي ‪. 0‬‬
‫العدد ‪ 5‬يقبل ‪ 2‬كسابقة وحيدة ابدلاةل ‪. f‬‬
‫‪1‬ن‬
‫العدد ‪ 0‬يقبل ‪ 1‬كسابقة وحيدة ابدلاةل ‪. f‬‬
‫لك من اجملالني ‪ 1;0 ، 1;5‬ومتناقصة متاما عىل ّ‬
‫لك من اجملالني‬ ‫ادلاةل ‪ f‬مزتايدة متاما عىل ّ‬
‫‪1‬ن‬ ‫‪. 0 :1 ، 2; 1‬‬
‫‪ f‬تقبل قمية حدية كربي عىل اجملال ‪ 2;5‬عند ‪ ، x  2‬تساوي ‪. 5‬‬
‫‪1‬ن‬
‫‪ f‬تقبل قمية حدية كربي عىل اجملال ‪ 2;5‬عند ‪ ، x  1‬تساوي ‪. 0‬‬
‫‪f‬‬ ‫انشاء املنحين ) ‪ (C‬املمثل لدلاةل‬
‫‪f‬‬

‫‪1‬ن‬

You might also like